You are on page 1of 434

Pharmacology in older patients

Perhaps the most common intervention performed by physicians is to write a


prescription.

Older patients will have more conditions requiring medication;


polypharmacy is common.

In the developed world:

• The over 65s typically make up around 14% of the population yet
consume 40% of the drug budget

• 66% 2/3 of the over 65s

• 87% 5/6 of the over 75s are on regular medication

• 34% 1/3 of the over 75s are on three or more drugs.

• Care home patients are on an average of eight medications.

• ‘Have you managed to take all those tablets I suggested?’


• ‘Which tablets do you find useful?’
• ‘Which tablets disagree with you?’— if yes, then ‘Do you manage to take them and
how often?’
• ‘What triggers you to remember?’ (e.g. take with each meal, leave by
toothbrush, etc.)
Differentiate normal age-related cognitive changes from mild
cognitive impairment/dementia:

The ability to perform activities of daily living ((eg, cooking, cleaning, using
transportation)) is one factor that can help differentiate normal age-related cognitive
changes from mild cognitive impairment/dementia, but it does not limit the validity of
the MMSE.

Initial evaluation of suspected dementia:

No matter which subtype is suspected, the initial evaluation of suspected dementia is


similar.

Initial workup of suspected cognitive impairment Cognitive testing


• MMSE (score <24/30 suggestive of MCI/dementia)
• Montreal Cognitive Assessment (score <26/30)
• Mini-Cog (abnormal 3-word recall &/or clock-drawing test)

Orientation to time, place and person


How many items of 3 items can the patient recall in 5 minutes
Stating the days of the week forward and backward

Interpretation of cognitive screening tests such as the Mini-Mental State Examination


(MMSE) or Montreal Cognitive Assessment should take educational level into
account.

The MMSE is a 30-point scale that assesses orientation, memory, concentration,


language, and visuospatial function.
A cutoff score of ≤23 is most commonly used as an indicator of cognitive impairment.

Individuals with high educational levels may score in the unimpaired range despite
experiencing clinically significant cognitive decline.
Interpretation of cognitive screening tests such as the Mini-Mental State Examination
(MMSE) or Montreal Cognitive Assessment should take educational level into
account.

The MMSE is a 30-point scale that assesses orientation, memory, concentration,


language, and visuospatial function.
A cutoff score of ≤23 is most commonly used as an indicator of cognitive impairment.

Individuals with high educational levels may score in the unimpaired range despite
experiencing clinically significant cognitive decline.

Laboratory testing
• Routine: CBC, vitamin B12, TSH, CMP
• Selective (specific risk factors): folate, syphilis, vitamin D level
• Atypical (early onset): CSF (for CJD)

Imaging
• Routine: CT scan or MRI of the brain
• Atypical: EEG

Evaluation involves neuropsychological testing (eg, Montreal Cognitive


Assessment), laboratory testing (complete blood count, vitamin B12, TSH, complete
metabolic panel), and neuroimaging.

Folate and vitamin D levels are not routinely obtained; they are indicated only in
specific risk groups.

For example, patients with alcohol use disorder may have folate deficiencies that can
contribute to cognitive impairment.

Some conditions may place a patient at risk for vitamin D deficiency (eg, celiac
disease, chronic kidney disease) and may also justify targeted testing.

Syphilis testing is indicated only in patients with specific risk factors such as those
with a known exposure.

Cerebrospinal fluid testing and electroencephalogram are only indicated in patients


with specific risk factors.

Cerebrospinal fluid testing may be indicated when a suspected underlying infection


(CJD, Syphilis, NPH) or malignancy is believed to be contributing to the presentation.

Electroencephalogram may be considered in patients with an underlying


seizure disorder or in those exhibiting seizure like activity (CJD).

The apolipoprotein E epsilon 4 allele is associated with an increased risk for


AD. However, genetic testing is contraindicated for the routine evaluation of
suspected dementia because it exposes patients to the potential
psychological harms of false positives and false negatives without providing
sufficient clinical utility.
Differential diagnosis of dementia subtypes

Differential diagnosis of dementia subtypes


Alzheimer disease
• Early, insidious short-term memory loss
• Language deficits & visuospatial dysfunction & disorientation
• Later personality changes

Vascular dementia
• Stepwise decline
• ASCVD: association
• Early executive dysfunction
• Cerebral infarction due to large artery (usually cortical) or smaller artery (subcortical
or lacunar) infarction &/or deep white matter changes on neuroimaging
• Typically associated with focal neurologic findings specific to the infarct location.

Frontotemporal dementia
• Early personality changes
• Apathy, disinhibition & compulsive behavior
• Frontotemporal atrophy on neuroimaging

Dementia with Lewy bodies


• Visual hallucinations
• Spontaneous parkinsonism
• Fluctuating cognition
• Rapid eye movement behavior disorder

Normal pressure hydrocephalus


• gait disturbance early in disease
• Dementia
• Urinary incontinence
• Dilated ventricles on neuroimaging

Prion disease
• Behavioral changes
• Rapid progression
• Myoclonus &/or seizures

Alzheimer disease

Pathophysiology:
Amyloid deposition occurs in Alzheimer disease.
Cognitive deficits present early in the disease

Gait impairment and urinary incontinence may occur but typically much later in the
disease course. Urinary urgency is not associated with Alzheimer disease.

• Early and prominent, insidious onset progressive short-term memory loss


• Language deficits (difficulty with word finding), visuospatial disorientation (eg,
getting lost in familiar surroundings)(agnosia) & motor skills (apraxia).
• Followed by personality & behavioral changes (eg, apathy, disinhibition,
suspiciousness).
• Psychotic symptoms (delusions, hallucinations) may develop later in the course of
the disease.

Prominent, progressive and irreversible memory symptoms, insidious disease course,


and lack of associated neurological findings make Alzheimer disease (AD) more likely
than other subtypes of dementia (eg, vascular dementia, frontotemporal dementia,
dementia with Lewy bodies).

The early insidious onset of short-term memory impairment/loss (eg, poor short-term
memory, naming current president) followed by the development of language
difficulties (difficulty with word finding), visuospatial disorientation (agnosia), motor
skills (apraxia) and late behavioral changes (eg, apathy, disinhibition, suspiciousness)
in a previously high functioning patient is characteristic of Alzheimer disease (AD).

AD, the most common cause of dementia, typically begins with memory loss for
recent events, executive dysfunction, and impaired visuospatial abilities and motor
skills (apraxia).

Alzheimer disease presents with:


greater deficits in episodic memory
Visuospatial abilities (disorientation, agnosia)
Motor skills (apraxia)
without early prominent behavioral symptoms.

Alzheimer disease can be difficult to differentiate from depression-related cognitive


impairment because depression can occur in patients with Alzheimer disease.
However, this patient does not have other dementia symptoms, such as apraxia,
agnosia, and aphasia.
Patients with Alzheimer disease are typically less concerned about their memory loss
and are brought in for evaluation by family members.

In contrast to this patient who shows poor effort, they are more likely to try harder but
to give incorrect answers or confabulate to compensate for poor memory on mental
status testing.

Psychotic features
(delusions, hallucinations) may develop in the middle and late course of the disease.
Delusions with paranoid themes are common and can be particularly distressing to
both patients and caregivers.

Common disorders in the differential diagnosis of AD include vascular


dementia, pseudodementia (cognitive changes due to depression), and other
neurodegenerative dementias (eg, dementia with Lewy bodies,
frontotemporal dementia).

Although the diagnosis of AD is largely clinical, neuropsychological testing, brain


imaging, and laboratory testing are typically ordered to rule out reversible causes of
dementia.

Neuroimaging:
Neuroimaging is typically performed to exclude alternate diagnoses such as
a lesion (eg, tumor, subdural hematoma, infarction) or normal-pressure
hydrocephalus (NPH). In patients with early AD, MRI may be normal.
However, temporal lobe atrophy, which is most prominent in the
hippocampi and surrounding medial temporal lobes, is typically seen in later
stages.

///////////////////////////////////////////////////////////////////

Vascular dementia

• Significant atherosclerotic cardiovascular risk factors


• Course: Stepwise neurocognitive decline (deterioration)
Patients with overt strokes (eg, large vessel distribution) often display a classic
stepwise patten (ie, discrete, identifiable episodes of worsening that follow a stroke),
whereas some patients with VaD due to small-vessel disease have a smoother decline
due to the accumulation of smaller, "silent" insults.
• Early prominent executive dysfunction (eg, difficulty planning meetings, •
organization, and problem-solving)
• Subcortical symptoms due to small-artery disease can lead to a subcortical-type
VaD, which can include symptoms (such as eg, urinary frequency/incontinence, gait
disturbances, psychiatric symptoms such as depressed mood)
• Focal neurologic deficits (eg, asymmetric reflexes, gait abnormalities). (eg, middle
cerebral artery infarction causing contralateral weakness and sensory impairment).
• Cerebral cortical infarction &/or Subcortical lacunar infarcts, deep white matter
changes & hyperintensities on neuroimaging

Vascular dementia is caused by cerebral ischemia or infarction and


commonly presents with executive dysfunction, focal neurologic findings, and
subcortical symptoms/signs. It typically follows a progressively declining
course.

Pathophysiology
Ischemia and/or infarcts due to:
• Large-vessel atherosclerosis
• Small-vessel disease (eg, arteriolosclerosis)
• Cerebral amyloid angiopathy

Clinical manifestations
• Early, prominent executive dysfunction
• Patients tend to have a narrow-based, shuffling gait.
• Subtypes:
◦ Multi-infarct: often with stepwise decline
◦ Strategic cortical infarct (Large-vessel): associated with localizing cortical deficits
◦ Subcortical vascular encephalopathy: associated with subcortical signs (eg, urinary
incontinence, gait disturbances)

Imaging findings
• Cortical infarcts
• Subcortical lacunar infarcts
• White matter hyperintensities

Multiple hypointense cortical and subcortical lesions corresponding to old infarcts


may be seen on T1-weighted MRI sequences.

Treatment
• Risk factor modification (eg, hypertension, diabetes)
• •} Antiplatelet therapy
• Cholinesterase inhibitor

Vascular dementia is associated with stroke or small-vessel cerebrovascular disease.

The dementia is typically characterized by stepwise cognitive impairments associated


with focal neurologic deficits.

It is less likely in patients with no significant atherosclerotic cardiovascular risk


factors and a normal examination.

D.D: AD
Verbal learning and recall are often relatively preserved in patients with VaD
compared to those with AD.

urinary symptoms and abnormal reflexes could occur at late stages, this is more
consistent with VaD.

///////////////////////////////////////////////////////

Frontotemporal dementia

Pathophysiology:
Frontotemporal dementia (loss of frontotemporal neurons)

causes progressive dementia, but it is characterized by both prominent


changes in personality/behavior and aphasia.

Gait abnormalities and urinary urgency are uncommon.

Frontotemporal dementia is characterized by:

• Early personality & behavioral changes (eg, disinhibition, apathy), compulsive


behaviors (eg, peculiar eating habits, hyperorality), and impaired memory.
• Apathy, loss of empathy, disinhibition & compulsive behavior
• Visual-spatial functions usually remain intact.
• Frontotemporal atrophy on neuroimaging

Early personality (eg, euphoria, disinhibition, apathy) & behavioral changes


(compulsive behaviors (eg, peculiar eating habits, hyperorality)) that often precede
memory impairments (in contrast to AD).
Frontotemporal atrophy demonstrated on brain imaging.

Frontotemporal dementia (FTD),

a neurocognitive disorder that commonly presents at a relatively early age (50s-60s).

In many cases, FTD involves abnormal inclusions of


hyperphosphorylated tau protein or the 43 kD TAR (ie, transactive response) DNA-
binding protein (TDP-43). Neuronal cell death leads to characteristic frontal and
temporal lobe atrophy

Pathogenesis • TDP-43 mutation


• Tau protein inclusions (eg, Pick bodies)

Clinical features
• Early onset (age 50s-60s)
• Initial behavior or personality changes (3 of 6 needed for diagnosis)
with resulting stereotypic personality and behavioral symptoms, including:
• Disinhibition (eg, scratching coworkers' cars)
• Apathy (eg, unconcerned by work deadlines, loss of interest uninterested in golf,
careless)
• Loss of empathy (eg, impolite to wife)
• Compulsions or rituals (eg, putting gum on nightstand)
• Hyperorality (eg, incessant gum chewing)
• Executive dysfunction (eg, Montreal Cognitive Assessment [MoCA]
score 25/30)

Diagnosis is primarily clinical, based on having ≥3 of 6 behavioral


Symptoms.

FTD has an earlier age of onset than AD (usually age 50-60) and typically presents
with personality changes and loss of social inhibition. Unlike in AD, memory
impairment is usually mild in the initial stages of frontotemporal dementia.

However, neuroimaging with either a CT scan or MRI is often


performed to exclude other etiologies (eg, mass, stroke) and look for the
expected focal white matter loss[1]. The prognosis is terminal within 8-10
years after disease onset, and there are no treatments to lessen disease
Progression.
• Neuroimaging (CT, MRI, PET)
◦ Rule out other disorders
◦ Look for frontal/anterior temporal lobe atrophy
Atrophy of the frontal lobes on neuroimaging is consistent with
frontotemporal dementia.

Atrophy of the parietal lobes may be seen in primary progressive


aphasia, a dementing syndrome that primarily affects language early in the
disease.

Management aims to reduce behavioral symptoms with both

• Nonpharmacologic: behavior modification (eg, exercise, trigger avoidance,


redirection)
pharmacologic (eg, selective serotonin reuptake inhibitors, atypical
antipsychotics) options.

///////////////////
Differential diagnosis of idiopathic Parkinson disease
Atypical parkinsonism* 1. Multiple system atrophy
(Parkinson-plus syndromes)2. Progressive supranuclear palsy
3. Corticobasal degeneration
4. Dementia with Lewy bodies
Secondary parkinsonism 5. Drug-induced (eg, antipsychotics)
6. Posttraumatic (eg, boxer)
7. Metabolic (eg, Wilson disease)
8. Toxic (eg, carbon monoxide)
9. Infectious (eg, prion disease)
10. Vascular (eg, lacunar strokes)
Mimics of parkinsonism 11. Normal pressure hydrocephalus
12. Essential tremor
13. Functional tremor
*Also called Parkinson-plus syndromes.

////////////////
Parkinson disease (PD)
Parkinson disease (PD), a neurodegenerative disorder associated with the
accumulation of alpha-synuclein within neurons of the substantia nigra pars compacta.

Low alpha-synuclein may be seen in Parkinson disease (PD) and Lewy body dementia
(LBD).

Diagnosis of idiopathic Parkinson disease*


Essential criterion
• Motor parkinsonism = bradykinesia (slowness in movements) + either rest tremor
(eg, pill rolling) or rigidity (stiffness, stepwise resistance to passive
movements)

Physical examination findings contributing to the clinical diagnosis include


(mnemonic - TRAP) the following:
T - Tremor
• Resting 4- to 6-Hz tremor with a pill-rolling quality
• Frequently first manifests in 1 hand, which may slowly generalize to the
other hand and lower extremities
R - Rigidity
• Baseline increased resistance to passive movement about a joint, which
may be oscillating (cogwheel) or uniform (lead pipe)
A - Akinesia/bradykinesia
• Difficulty initiating movements, as when starting to walk or rising from a
chair
• Narrow-based, shuffling gait with short strides and with decreased arm
swing (festination)
• Micrographia (small handwriting)
• Hypomimia (decreased facial expression ["masked facies"], lack of
blinking)
• Hypophonia (soft speech)
P - Postural instability
• Flexed axial posture
• Loss of balance during turning or stopping
• Loss of balance when pushed slightly from a stationary (pull test), bipedal
stance
• Frequent falls

Supportive features
• Excellent response to dopaminergic therapy (eg, levodopa)
• Asymmetric limb findings (eg, tremor, rigidity) at start
• Olfactory dysfunction
• Abnormal dopaminergic function on neuroimaging (SPECT, PET)

Exclusion criteria
• Vertical (supranuclear) gaze palsy
• Cerebellar signs (eg, limb ataxia, evoked nystagmus)
• Cortical sensory loss (eg, astereognosis)
• Progressive aphasia
• Antidopaminergic medication use (eg, haloperidol)

Red flags
• Early postural instability (>1/year) (eg, recurrent falls, loses balance when he tries to
turn or stop suddenly while walking or need for wheelchair)
Postural reflex dysfunction is typically a late finding (eg, >3 years after diagnosis).
This is why postural instability is not an essential criterion for PD diagnosis despite
being a cardinal manifestation.

Although frequent falls (>1/year) are suggestive of postural instability, postural


reflexes should be evaluated by a pull test[1] because recurrent falls can have several
other causes (eg, orthostasis, gait freezing).
Therefore, this patient's abnormal pull test (ie, ≥3 steps to recover when pulled)
confirming early-onset (eg, 12 months)

postural instability is a red flag that suggests a diagnosis other than PD (eg,
progressive supranuclear palsy).
• Early bulbar dysfunction (eg, dysarthria, dysphagia)
• Absence of nonmotor signs (eg, sleep disturbance, constipation)
• Severe orthostatic hypotension (eg, ≥30 mm Hg systolic drop)
• Symmetric symptoms, hyperreflexia

*Established Parkinson disease is parkinsonism + no exclusion criteria +


2 supportive criteria (no red flags).

Probable Parkinson disease is parkinsonism + no exclusion criteria + (sup


portive criteria ≥ red flags).

Although both can cause behavior changes and cognitive impairment, PD is typified
by bradykinesia, resting tremor, and rigidity.

Although idiopathic Parkinson disease most often presents with tremor,


it can also manifest with primarily bulbar symptoms, such as in this patient.
Specifically, this patient has the following:
• Hypophonia (a soft voice)
• Aspiration of liquids (coughing when drinking)
• Loss of olfactory function (often perceived by patients as change in taste)
• Rigidity of the upper limbs (increased resistance to passive movement
around a joint)
• Stooped posture and slow gait

Diagnostic features such as rigidity, postural instability, and gait changes are
often discovered on physical examination instead of through a patient's
report.

"Not being funny" can sometimes indicate a combination of


bradykinesia of the face (masked facies) and/or the voice (hypophonia).

In patients in whom Parkinson disease is being considered, bulbar symptoms


may be considered "red flags" because they can be seen with other related
conditions; however, the presence of red flags does not exclude the
diagnosis of Parkinson disease so long as other supportive features (eg,
olfactory dysfunction) are present.
///////////////////////

Dementia with Lewy Bodies (LBD)

dementia with Lewy bodies (DLB), a form of degenerative dementia characterized


by accumulations of alpha-synuclein protein (eosinophilic cytoplasmic inclusions:
Lewy bodies), may be seen in neurons of the substantia nigra, locus coeruleus, dorsal
raphe nucleus, and substantia innominata eosinophilic cytoplasmic.

DLB is characterized by alterations in consciousness, fluctuations in cognition,


visual hallucinations, parkinsonism, and relatively early compromise of executive
functions. Repeated falls and sleep disturbance are characteristic.

Dementia with Lewy bodies can present with parkinsonian features (eg, rigidity).

is characterized by:

• Early development of prominent detailed complex visual hallucinations ("a lion


was sleeping in the backyard", "strangers in the backyard," who are not there when
son looks for them)
• Spontaneous parkinsonism (ie, increased muscle tone, loss of mobility, coarse resting
tremor, early falls, significant orthostasis)
• Fluctuating cognition/periodic confusion (Fluctuating neurocognitive impairment
with alterations in consciousness and variations in attention and alertness)
• Rapid eye movement behavior disorder
• Dementia/cognitive impairment (memory loss and frequently gets lost in her
neighborhood)
• Can cause behavior changes
• Other supportive clinical features of DLB include Severe sensitivity to
antipsychotics, repeated falls, syncope, autonomic dysfunction (eg orthostatic
hypotension, heart rate variability), delusions, and depression.

Neurologic examination : Dysautonomia, Parkinsonism.

Dementia with Lewy bodies can present with parkinsonian features (eg, rigidity).
However, dementia, which is not seen in this patient,
must also be present. In addition, patients often have hallucinations and
fluctuating cognition.

Dementia with Lewy bodies

If The patient's early, prominent memory loss and unremarkable neurologic


examination are more consistent with AD.

Pharmacotherapy of DLB consists of carbidopa-levodopa for parkinsonism and


cholinesterase inhibitors for cognitive impairment.

If psychotic symptoms persist, a low-dose second-generation antipsychotic, rather than


a first-generation antipsychotic, is preferred due to the severe neuroleptic sensitivity
seen in these patients.

Geriatric Medicine SCE (Question Bank) (studyprn.com)

ijzkx@hi2.in
////////////////

Parkinson disease
Accumulation of alpha-synuclein within dopaminergic neurons is
consistent with Parkinson disease.

Parkinson can cause a shuffling gait but also typically causes bradykinesia, rigidity,
and a pill-rolling tremor.
Cognitive impairment is a late-stage finding, and urinary urgency would be
unexpected.

Parkinsonism

Multiple system atrophy (Shy-Drager syndrome), which is a degenerative disease


characterized by the following:
1. Parkinsonism (rigidity, bradykinesia)
2. Autonomic dysfunction (postural hypotension, dry mouth, dry skin, and erectile
dysfunction, abnormal sweating, disturbance of bowel or bladder control
(incontinence), abnormal salivation or lacrimation, impotence, gastroparesis, etc)
3. Widespread neurological signs (cerebellar, pyramidal or lower motor
neuron)

Always consider Shy-Drager syndrome when a patient with Parkinsonism


experiences orthostatic hypotension, impotence, incontinence, or other
autonomic symptoms. The accompanying bulbar dysfunction and laryngeal
stridor may be fatal. Anti-Parkinsonism drugs are generally ineffective, and
treatment is aimed at intravascular volume expansion with fludrocortisone,
salt supplementation, alpha-adrenergic agonists, and application of
constrictive garments to the lower body.

Progressive supranuclear palsy (PSP)


Vertical supranuclear palsy (ie, inability to elevate eye to look up)
is a feature of progressive supranuclear palsy (PSP), which presents with
dementia, postural instability, falls, dysautonomia, vertical supranuclear palsy, and
parkinsonism

///////////////////////////////////////////
Normal pressure hydrocephalus

NPH presents with the triad of cognitive impairment, gait disturbance,


and urinary urgency/incontinence.

Aetiology:
Most cases are idiopathic and occur in the elderly; however, secondary disease can
occur in those with prior neurologic insults (eg, subarachnoid hemorrhage [SAH],
trauma, meningitis) that result in scarring and destruction of the arachnoid
granulations responsible for CSF resorption.
Pathophysiology:
It is associated with reduced cerebrospinal fluid (CSF) reabsorption, which may
occur due to elevated central pressures or chronic periventricular ischemia.
Accumulation of cerebrospinal fluid (CSF) due to reduced absorption leading to
ventriculomegaly with normal opening pressure on lumbar puncture.

Clinical features
• Gait instability ( Shuffling slow, wide-based small steps gait, often described as
appearing that the feet are stuck to the ground is characteristic and typically appears
early in the disease process magnetic gait) with frequent falls.
• Cognitive dysfunction (eg, psychomotor slowness & retardation, apathy): typically
occurs slowly over a period of years and is associated with apathy and impaired
executive function.
• Urinary urgency (early)/incontinence with apathy occurs as the disease progresses.
• Depressed flat affect (frontal lobe compression)
• Upper motor neuron signs (in some cases hyperreflexia, upgoing plantar reflexes and
spasticity) in lower extremities.

NPH classically presents with a triad of urinary incontinence, cognitive impairment,


and gait abnormalities; however, all symptoms may not be present in early disease.

Diagnosis
• Lumbar puncture: normal opening pressure and analysis on lumbar puncture.
Rapid response to high-volume lumbar puncture is a confirmatory test for normal-
pressure hydrocephalus (NPH) in which gait function typically improves following
removal of 30-50 mL of cerebrospinal fluid.
• Marked improvement in gait with spinal fluid removal: Miller Fisher (lumbar tap)
test. The diagnosis is confirmed with improvement after high volume lumbar
puncture or lumbar drain placement.
• Enlarged ventricles out of proportion to the underlying brain atrophy on MRI/CT

Treatment
• Ventriculoperitoneal shunting

D.D:

Aqueductal stenosis can also occur after SAH and causes ventricular enlargement;
however, symptoms of elevated intracranial pressure (eg, headache, nausea/vomiting)
predominate.
Gait disturbances in parkinsonism are classically described as shuffling or
festinating (quick fast, short steps) (can rarely result from cerebrovascular
disease that interferes with the striatonigral pathway).
Rigidity, bradykinesia, and pill-rolling tremors are also expected, but ventricular
enlargement is not common.

///////////////////////////////////////////////////////

Creutzfeldt-Jakob disease (CJD)

• Rapidly progressive cognitive decline


• Behavioral/personality changes
• Mood changes (eg, depression)
• Neuropsychiatric decline: behavioral abnormalities, personality
changes, sleep disturbances (eg, insomnia, hypersomnia), and rapidly
progressive dementia. This may progress to a loss of speech and
inability to care for oneself.
• Myoclonus: classically provoked by a startle (eg, startle-induced arm jerking) &/or
seizures
• Cerebellar signs: (eg, falls, nystagmus, dysdiadochokinesia, dysmetria, ataxia
(wide-based gait))
• Upper motor neuron signs (eg, hyperreflexia, upgoing Babinski) due to
degeneration of the corticospinal tracts
• Extrapyramidal signs: hypokinesia, dystonia
• Rapid progression
• Myoclonus
• Cerebellar dysfunction
• Visual field defects (eg, sideswiping cars)

CJD is most commonly sporadic, but it can be hereditary, iatrogenic (due to


contaminated transplants or surgical instruments), or due to oral
consumption of contaminated neural tissue.

Clinical features
• Rapidly progressive neurocognitive deterioration (dementia).
• Mood/personality changes &/or sleep disturbances (hypersomnia)
• Myoclonus (eg, startle-induced & provoked involuntary muscle spasm)
• Cerebellar signs (eg, ataxia, falls, dysdiadochokinesia, dysmetria)
• Upper motor neuron signs (eg, hyperreflexia, extensor plantar reflexes)
• Visual field defects (eg, hemianopsia, sideswiping cars),
• Extrapyramidal signs (eg, hypokinesia)
Findings

Head CT is often normal, especially early in the disease course.

MRI is more sensitive and often shows hyperintensity in the cortex, caudate, and
anterior putamen early, with evidence of generalized atrophy and ventricular dilatation
becoming more apparent as the disease progresses.

• MRI: widespread generalized diffuse atrophy (cerebrum & cerebellum), cortical


enhancement (ie, cortical ribboning), enhancement of putamen & caudate head (ie,
hockey stick sign).

Cerebrospinal fluid (CSF) findings in patients with CJD may include:


• No white blood cells, normal glucose, normal or elevated protein (ie,
noninflammatory)
• Elevated 14-3-3 protein titers (a marker of neuronal cell death not
specific for CJD)
• Elevated tau protein (also seen in other neurodegenerative diseases)

• Real-time quaking-induced conversion (RT-QuIC) is a newer


assay performed on CSF in which the abnormally folded proteins are
isolated, amplified, and then incubated with normally folded proteins.

A prion-induced conformational change in the normal proteins can be observed in real


time via fluorescence detection. confirms a diagnosis of Creutzfeldt-Jakob disease .

The RT-QuIC test has greater sensitivity and specificity compared to 14-3-3 and tau
analyses, but it is not widely available.

• EEG: Periodic sharp-wavecomplexes (biphasic or triphasic sharp, synchronous


discharges) on an electroencephalogram (EEG).

• Neuropath: spongiform degeneration without inflammation

In contrast to CJD, Alzheimer disease has a more gradual onset of cognitive


dysfunction (years vs months). In addition, myoclonus, upper motor neuron signs, and
EEG periodic sharp waves are more characteristic of CJD.
Management & prognosis
• Symptomatic treatment, counseling, social services referral
• No effective disease-modifying therapy, fatal in <12 months

Although CJD has a long incubation period (estimated mean of 10 years), once
symptoms occur, patients experience an inexorable decline (eg, loss of speech, loss of
ability for self-care, coma) until death, which occurs less than 12 months after
diagnosis.

Only supportive treatment is available, which should include


symptom-based management, compassionate education, and referral to
social services.

///////////////
Anti-Hu encephalitis is a paraneoplastic syndrome most commonly associated with
small cell lung cancer (more common in patients who smoke).

It may present with rapid-onset dementia, including personality


and behavior changes.

however, patients also typically have seizures, hallucinations, and brainstem


symptoms (eg, dizziness, nausea).

Myoclonus is not usually present.

////////////////////////////////////

Anti-NMDA receptor encephalitis


Anti-NMDA receptor encephalitis can present with rapidly progressive behavior
changes, memory dysfunction, and abnormal movements (eg, dyskinesia, chorea,
facial twitching), it is also usually accompanied by hallucinations and autonomic
instability.

In addition, it is most commonly associated with ovarian teratomas.

///////////////////////////

Herpes simplex viral encephalitis

can present with rapidly worsening mental status. However, it typically manifests
much faster (ie, <1 week) than CJD and is usually accompanied by fever, headache,
seizures, and focal neurologic deficits.
//////////

Huntington disease

An autosomal dominant disease of CAG trinucleotide repeat expansion.

This trinucleotide expansion results in the formation of an abnormal huntingtin protein


that is particularly toxic to the caudate nucleus and putamen (ie, neostriatum).

The neostriatum is rich in GABA-producing neurons that are preferentially


destroyed due to HD.

Because trinucleotide repeats may expand between generations (particularly with


paternal transmission), HD is often associated with an anticipation effect in which
consecutive generations develop symptoms of the disease at progressively younger
ages.

Clinical features
• Motor: restless, chorea (ie, involuntary abrupt, twisting motions & contractions that
seem to flow from one muscle group to the next).
Early in the disease course, chorea may be mild and mistaken for restlessness, and
patients may unconsciously assimilate mild chorea into purposeful movements
(eg,smoothing one's hair).

Other neurological findings include hyperreflexia and on extraocular movement


testing shows delayed initiation of voluntary saccades.

Cognitive impairment: is most evident in the domain of executive function.


on the Montreal Cognitive Assessment (normal:≥26), losing points on measures of
executive function.

Psychiatric: depression, irritability, psychosis, obsessive-compulsive symptoms


• Cognitive: executive dysfunction

Findings
• Genetic: autosomal dominant CAG trinucleotide repeat exp
ansion disorder
• Neuropathology: loss of GABA-ergic neurons
• Imaging: caudate nucleus & putamen (ie, neostriatum) atrophy

Management/prognosis
• Treatment: supportive
• Survival: 10-20 years

///////////////
Vitamin B12 deficiency

Vitamin B12 deficiency can cause dementia and subacute combined degeneration with
ataxia and incontinence.

However, patients typically also develop bilateral lower extremity paresthesias,


glossitis, and macrocytic anemia.

///////////
Pseudodementia (also referred to as dementia syndrome of
Depression)

is seen in older depressed patients (history of depression or prominent mood


symptoms(anhedonia, and apathy)) who present with neurocognitive deficits similar to
neurodegenerative dementias.

Major depressive disorder


Irritability, neurocognitive changes, anhedonia, and apathy can be
seen in major depressive disorder.
however, disinhibitions, oral fixations, ritualistic behaviors, and mounting social
indiscretions are not characteristic of depression.

Major depressive disorder


Diagnosis  ≥5 of the following symptoms lasting ≥2
weeks (at least 1 symptom must be either
depressed mood or loss of interest/pleasure):
 Depressed mood (sadness)
Anhedonia: Loss of interest or pleasure (enjoyment)
of previously pleasurable social activities used to
love it causing apathy (social withdrawal,
withdrawn and less socially engaged
than normal).
Change in appetite or weight
 Sleep disturbance: Insomnia or
hypersomnia
Psychomotor retardation (eg, slowed speech,
thinking,Movements, impaired ability to think) or
agitation
 Low energy (malaise, tiredness)
 Poor concentration, attention or
indecisiveness
 Thoughts of worthlessness (feeling useless
and wortheless) or inappropriate guilt
 Recurrent thoughts of death or suicide
 No history of mania or hypomania
 Not due to substances or another medical
condition
Treatment  Psychotherapy
 Antidepressant medication

Decreased interest and enjoyment of social activities (eg, no longer spending time with
friends)
Impaired concentration (eg, difficulty focusing in church choir limiting participation).
Sleep disturbance
Fatigue (low energy)
Loss of appetite is consistent with Depression

Increasing forgetfulness, low energy and motivation, sleep disturbance, and


psychomotor retardation (eg, slowed speech, thinking,
movements) are consistent with major depressive disorder (MDD).

Older patients who are severely depressed may demonstrate cognitive impairment and
slowing rather than depressed mood (previously referred to as Pseudodementia).

During cognitive testing, patients with this condition typically exhibit deficits in
attention, concentration, memory, and executive function and are often compounded
by poor effort during testing.

They may have difficulty completing testing and frequently put forth
poor effort.

Depression-related cognitive impairment is largely reversible with the


treatment of the underlying depression using medications such as selective serotonin
reuptake inhibitors (eg, escitalopram) and/or psychotherapy.

As the patient's depression improves, cognitive impairment (and Montreal Cognitive


Assessment score) should return to premorbid baseline.
Although these patients are at higher risk for developing dementia later, their
depression-related cognitive impairment is potentially reversible and can improve
with treatment of the underlying depression.

Alzheimer disease can be difficult to differentiate from depression-related cognitive


impairment because depression can occur in patients with Alzheimer disease.
However, this patient does not have other dementia symptoms, such as apraxia,
agnosia, and aphasia.

Patients with Alzheimer disease are typically less concerned about their memory loss
and are brought in for evaluation by family members.

In contrast to this patient who shows poor effort, they are more likely to try harder but
to give incorrect answers or confabulate to compensate for poor memory on mental
status testing.

///////////////////////////////////////////

prior carbon monoxide (CO) poisoning

Can present with neuropsychiatric symptoms (cognitive


dysfunction, personality changes, movement disorders).

MRI can show hemorrhagic infarction of the basal ganglia, although this is typically
limited to the globus pallidus. Cerebellar signs (eg, nystagmus) are uncommon.

/////////////////////////////

Wernicke encephalopathy due to thiamine deficiency can also


present with confusion and ataxia (often with nystagmus). However,
myoclonus is less typical; and MRI classically reveals mamillary body atrophy
and periaqueductal gray matter hyperintensity.

Wilson disease can also cause subacute, progressive dementia


with movement disorders and MRI can show basal ganglia changes.
However, it typically presents in children or adolescents and is accompanied
by liver dysfunction.
Severe AS criteria
• Aortic jet velocity ≥4.0 m/sec, or
• Mean transvalvular pressure gradient ≥40 mm Hg
• Valve area usually ≤1.0 cm2 but not required

Indications for valve replacement


Severe AS & ≥1 of the following:
• Onset of symptoms (eg, angina, syncope)
• Left ventricular ejection fraction <50%
• Undergoing other cardiac surgery (eg, CABG)

///////////////////////////////////////////////////////////////////////////

1.A 75-year-old male is currently being assessed for an elective inguinal hernia repair.
Which frailty score is recommended to identify frailty in elective surgical patients?

1. Edmonton Frail Scale


2. Gait speed
3. PRISMA 7
4. Rockwood Clinical Frailty Scale
5. Timed Up and Go test

Which frailty score is recommended to identify frailty in elective surgical patients?

1. Edmonton Frail Scale Correct answer selected

Explanation:

In outpatient surgical settings, there is a lack of consensus on which tool should be


used to identify frailty.
Gait speed may help predict adverse outcomes; however, evidence is emerging for the
use of the Edmonton Frail Scale. The strengths of this tool include brevity, clinical
feasibility and identification of aspects of frailty amenable to preoperative
optimisation (e.g. cognition, nutrition). Furthermore, the association of pre-operative
gait velocity with postoperative morbidity and mortality makes this a potentially
useful frailty measure in the elective preoperative setting.

PRISMA 7 Questionnaire – A seven-item questionnaire to identify disability that has


been used in earlier frailty studies and is also suitable for postal completion. A score
of >3 is considered to identify frailty.

Walking speed (gait speed) - Gait speed is usually measured in m/s and has been
recorded over distances ranging from 2.4m to 6m in research studies.

Timed Up and Go test – The TUGT measures, in seconds, the time taken to stand up
from a standard chair, walk a distance of 3 metres, turn, walk back to the chair and sit
down.

It is inappropriate to use the (Rockwood) Clinical Frailty Scale (CFS) 7 as a method of


identifying frailty without a formal clinical assessment. The CFS was designed to be
used to measure the severity of frailty after a comprehensive geriatric assessment. It is
not validated for measuring improvement in individuals after an acute illness for
example.

Reference:

2. British Geriatrics Society (2014). Fit for Frailty – Consensus best practice guidance
for the care of older people living in community and outpatient settings.

A 77-year-old woman presents to the hospital with a fall and is diagnosed with an
acute fracture of her left neck of femur.

She is not on regular medication, with an unremarkable past medical history. Her
mother and sister both have suffered hip fractures.

The patient is concerned regarding the mortality following a hip fracture. What is the
approximate mortality rate at one year following a hip fracture?

1. 5%
2. 15%
3. 20%
4. 30%
5. 45%

What is the approximate mortality rate at one year following a hip fracture?
1.
2.
3. 30% Correct answer

Explanation:

28.7% of patients who suffer a hip fracture will die within 12 months of the fracture.

Reference:

4. NOGG Guideline 2017 Clinical Guideline for the Prevention and Treatment of
Osteoporosis.
5. National Hip Fracture Database 2016 Annual Report.
6. Neuburger J, Currie C, Wakeman R et al. The impact of a national clinician-led audit
initiative on care and mortality after hip fracture in England: an external evaluation
using time trends in non-audit data. Med Care 2015;53:686-91.

A 76-year-old woman had an elective hip replacement as a treatment for her


osteoarthritis. The patient’s risk of VTE outweighs her risk of bleeding. The team
started her on low molecular weight heparin (LMWH) for 28 days combined with anti-
embolism stockings until discharge.

In which one of these situations should anti-embolism stockings be used?

1. A known allergy to the material of manufacture


2. Mild leg oedema
3. Peripheral arterial bypass grafting
4. Peripheral neuropathy or other causes of sensory impairment
5. Suspected or proven peripheral arterial disease

When should anti-embolism stockings be used?

1. Mild leg oedema Correct answer selected


2.

Explanation:

NICE recommends not to offer anti-embolism stockings to people who have:

3. suspected or proven peripheral arterial disease


4. peripheral arterial bypass grafting
5. peripheral neuropathy or other causes of sensory impairment
6. any local conditions in which anti-embolism stockings may cause damage – for
example, fragile 'tissue paper' skin, dermatitis, gangrene or recent skin graft
7. known allergy to the material of manufacture
8. severe leg oedema
Reference:

9. National Institute for Health and Care Excellence, 2018. Venous Thromboembolism in
over 16s: reducing the risk of hospital-acquired deep vein thrombosis or pulmonary
embolism. NICE guideline [NG89].

A 68-year-old man had an elective knee replacement as a treatment for his


osteoarthritis. The patient’s risk of venous thromboembolism (VTE) outweighs their
risk of bleeding. The team started her on low molecular weight heparin (LMWH) for
14 days combined with anti-embolism stockings until discharge.

What calf pressure should anti-embolism stockings provide?

1. 10 – 11 mmHg
2. 11 – 12 mmHg
3. 12 – 13 mmHg
4. 13 – 14 mmHg
5. 14 – 15 mmHg

What calf pressure should anti-embolism stockings provide?

1. 14 – 15 mmHg Correct answer selected

Explanation:

NICE recommends using anti-embolism stockings that provide graduated compression


and produce a calf pressure of 14 mmHg to 15 mmHg.

Reference:

2. National Institute for Health and Care Excellence, 2018. Venous Thromboembolism in
over 16s: reducing the risk of hospital-acquired deep vein thrombosis or pulmonary
embolism. NICE guideline [NG89].

A 72-year-old man was admitted to the hospital after a fall. He had a neck of femur
fracture as a result of that fall. He has a past medical history of osteoporosis.

For how long should this patient with a neck of femur fracture be maintained on
venous thromboembolism (VTE) prophylaxis?

1. 1 month
2. 3 months
3. 4 months
4. 6 months
5. 12 months

1. 1 month Correct answer selected

Explanation:

NICE recommends offering VTE prophylaxis for one month to people with fragility
fractures of the pelvis, hip or proximal femur if the risk of VTE outweighs the risk of
bleeding. Choose either:

2. low molecular weight heparin (LMWH), starting 6 to12 hours after surgery or
3. fondaparinux sodium, starting 6 hours after surgery, providing there is a low risk of
bleeding.
Reference:

4. National Institute for Health and Care Excellence, 2018. Venous Thromboembolism in
over 16s: reducing the risk of hospital-acquired deep vein thrombosis or pulmonary
embolism. NICE guideline [NG89].

A 70-year-old woman was admitted for an elective hip replacement as a treatment for
her osteoarthritis. The patient’s risk of venous thromboembolism (VTE) outweighs
their risk of bleeding.

What is the VTE prophylaxis regimen for an elective hip replacement that should be
used?

1. Aspirin only (75 or 150 mg) for 14 days


2. Low molecular weight heparin (LMWH) for 10 days combined with anti-embolism
stockings (until discharge)
3. Low molecular weight heparin (LMWH) for 10 days followed by aspirin (75 or 150
mg) for a further 28 days
4. Low molecular weight heparin (LMWH) only for 28 days
5. Low molecular weight heparin (LMWH) for 28 days followed by aspirin (75 or 150
mg) for a further 28 days

1. Low molecular weight heparin (LMWH) for 10 days followed by aspirin (75 or 150
mg) for a further 28 days Correct answer

Explanation:

NICE recommends offering VTE prophylaxis to people undergoing elective hip


replacement surgery whose risk of VTE outweighs their risk of bleeding. Choose any
one of:

2. LMWH for 10 days followed by aspirin (75 or 150 mg) for a further 28 days.
3. LMWH for 28 days combined with anti-embolism stockings until discharge.
Reference:

4. National Institute for Health and Care Excellence, 2018. Venous Thromboembolism in
over 16s: reducing the risk of hospital-acquired deep vein thrombosis or pulmonary
embolism. NICE guideline [NG89].

A 74-year-old man was admitted for elective knee replacement as a treatment for his
osteoarthritis. The patient’s risk of venous thromboembolism (VTE) outweighs their
risk of bleeding.
What is the VTE prophylaxis for elective knee replacement regimen that should be
used?

1. Aspirin only (75 or 150 mg) for 14 days


2. Low molecular weight heparin (LMWH) for 10 days combined with anti-embolism
stockings (until discharge)
3. Low molecular weight heparin (LMWH) for 10 days followed by aspirin (75 or 150
mg) for a further 28 days
4. Low molecular weight heparin (LMWH) for 28 days followed by aspirin (75 or 150
mg) for a further 28 days
5. Low molecular weight heparin (LMWH) only for 28 days

1. Aspirin only (75 or 150 mg) for 14 days Correct answer

Explanation:

NICE recommends offering VTE prophylaxis to people undergoing elective knee


replacement surgery whose VTE risk outweighs their risk of bleeding. Choose any one
of:

2. Aspirin (75 or 150 mg) for 14 days.


3. LMWH for 14 days combined with anti-embolism stockings until discharge.
Reference:

4. National Institute for Health and Care Excellence, 2018. Venous Thromboembolism in
over 16s: reducing the risk of hospital-acquired deep vein thrombosis or pulmonary
embolism. NICE guideline [NG89].

A 69-year-old man was admitted for arthroscopic knee surgery. He has hypertension
that is well-controlled. The expected time for the surgery is about two hours.
What is the venous thromboembolism (VTE) prophylaxis regimen for arthroscopic
knee surgery for two hours that should be used?

1. He does not need VTE prophylaxis


2. Low molecular weight heparin (LMWH) 6 to 12 hours after surgery for 14 days
3. Low molecular weight heparin (LMWH) 6 to 12 hours after surgery for 28 days
4. Low molecular weight heparin (LMWH) immediately after surgery for 14 days
5. Low molecular weight heparin (LMWH) immediately after surgery for 28 days

1. Low molecular weight heparin (LMWH) 6 to 12 hours after surgery for 14 days
Correct answer selected
2.

Explanation:

NICE recommends that LMWH should be considered 6 to 12 hours after surgery for
14 days for people undergoing arthroscopic knee surgery if total anaesthesia time is
more than 90 minutes or the person's risk of VTE outweighs their risk of bleeding.

Reference:

3. National Institute for Health and Care Excellence, 2018. Venous Thromboembolism in
over 16s: reducing the risk of hospital-acquired deep vein thrombosis or pulmonary
embolism. NICE guideline [NG89].

You are called to review a patient on the Orthopaedic ward who has become unwell.
The patient is an 87-year-old woman with a past medical history of hypertension, type
two diabetes, osteoporosis and previous left-sided hip replacement for osteoarthritis.

Her medication includes, atenolol 25 mg od po, bendroflumethiazide 2.5mg od po,


gliclazide 80mg bd, vitamin D with calcium and alendronic acid.

She was initially admitted following a fall under the care of the medical team, her care
however was transferred to the trauma and orthopaedic team following a fall on the
ward, where she suffered a right-sided hip fracture.

She underwent a hemiarthroplasty two days previously. Postoperatively she


complained of tiredness, nausea and weakness, with occasional vomiting. She had not
eaten and was given supplementary fluids post-operatively. The staff had noticed an
element of confusion. She had not taken any analgesia for one day.

You were called as she had suffered what was described as an acute tonic-clonic
seizure, which was prolonged, around one hour ago, and which ultimately ceased
following treatment with diazepam. She is now drowsy and non-responsive

Her observations are stable (MEWS score of 1)

Bloods immediately post-seizure showed:

Investigation Result Normal range


Sodium 125 mmol/L 133-146 mmol/L
Potassium 4.1 mmol/L 3.5 -5.3 mmol/L
Urea 5.3 mmol/L 2.5 – 7.0 mmol/L
CRP 18 mg/L < 6 mg/L
Procalcitonin 0.01 <0.05
Creatinine 104 umol/L 46-92 umol/L
Hb 112 g/L 115-165 g/L
MCV 88 fL 80 - 100 fL
Neutrophil 4.1 x 109/L 0.2-0.8 x 109/L
White Cell 7.1 x 109/L 4.0-11 x 109/L
Count
Capillary Blood 11.2 mmol/L < 7.8 mmol/L
Glucose
Pre-operatively:

Investigation Result Normal range


Sodium 141 mmol/L 133-146 mmol/L
Potassium 4.1 mmol/L 3.5 -5.3 mmol/L
Urea 11.2 mmol/L 2.5 – 7.0 mmol/L
Creatinine 92 umol/L 46-92 umol/L
CRP 24 mg/L < 6 mg/L
Hb 123 g/L 115-165 g/L
MCV 88 f/L 10^9/L 80 - 100 f/L
Neutrophil 4.8 x 109/L 0.2-0.8 x 109/L
White Cell count 8.3 x 109/L 4.0-11 x 109/L

What would be the most appropriate immediate investigation?

1. CT head
2. CT-thorax, abdomen and pelvis
3. EEG
4. Repeat CRP, white cell count and mid-stream urine
5. Repeat urea and electrolytes

1. Repeat urea and electrolytes Correct answer

Explanation:

The patient has postoperative hyponatremia with a reduction in urea and creatinine on
a background of treatment with intravenous fluids.

She has symptoms suggestive of acute hyponatremia with altered conscious level,
lethargy, weakness and neurological signs with a seizure.

The sodium taken immediately after or during the seizure can be falsely elevated as a
result of epileptic activity.

She is taking bendroflumethiazide and atenolol, classes of drugs that are associated
with the increased risk of developing postoperative hyponatremia.
The most appropriate immediate action would be to repeat her urea and electrolytes,
which often is performed via arterial gasses as her hyponatremia will require urgent
treatment.

Reference:

2. Nardone R, Brigo F, Trinka E. Acute Symptomatic Seizures Caused by Electrolyte


Disturbances. J Clin Neurol. 2016 Jan;12(1):21-33. doi: 10.3988/jcn.2016.12.1.21.
PMID: 26754778; PMCID: PMC4712283.
3. Jeong, Y.M., Lee, E., Kim, K.I., Chung, J.E., Park, H.I., Lee, B.K. and Gwak, H.S.,
2016. Association of pre-operative medication use with post-operative delirium in
surgical oncology patients receiving comprehensive geriatric assessment. BMC
geriatrics, 16(1), p.134.
A 91-year-old woman presented following a fall with an intracapsular right-sided hip
fracture.

The patient stated that she had tripped over a dog.

She had a right hemiarthroplasty with no immediate intra or postoperative


complications. She was living alone prior to admission with no package of care.

You are called to review her three days postoperatively as she has developed an
element of confusion and mild paranoia and aggression which is out of keeping with
the initial presentation.

Today's bloods show renal function, liver function and bone profile are all in the
normal range. Her CRP was 12, with normal white cell count and neutrophil count.
Her capillary blood glucose is 6.5.

Her observations are stable with a National Early Warning Score (NEWS) score of 1.

She has not required analgesia other than paracetamol for 24 hours. Her medication
includes amitriptyline 10mg od po, ramipril 2.5 mg od, paracetamol 1g QDS po.

What is the most likely factor contributing to her delirium?

1. Amitriptyline
2. Electrolyte abnormality
3. Paracetamol
4. Ramipril
5. Urinary tract infection

Explanation:

Amitriptyline is the most likely agent causing postoperative delirium in this example.
Delirium inducing medications causing symptoms such as delirium, confusion or
hallucination, were reported over 1 % by the drug information database Micromedex.

There are no signs to suggest infection and bloods were normal, including renal
function.
Reference:

1. Jeong, Y.M., Lee, E., Kim, K.I., Chung, J.E., Park, H.I., Lee, B.K. and Gwak, H.S.,
2016. Association of pre-operative medication use with post-operative delirium in
surgical oncology patients receiving comprehensive geriatric assessment. BMC
geriatrics, 16(1), p.134.

2. Amitriptyline Correct answer selected


You are called by the surgical team regarding an 80-year-old woman who initially
presented to hospital with vomiting and was treated for pseudo-obstruction. Her
symptoms have now resolved and she is preparing for discharge that morning.

She has a past medical history of type two diabetes and hypertension.

Her heart rate is 74 bpm, blood pressure of 132/76, saturations on air of 99% with a
respiratory rate of 16 and temperature of 37.3 oC.

She was initially treated on admission for a urinary tract infection with trimethoprim.
A mid-stream urine test on admission showed E.coli which was sensitive to
trimethoprim, fosfomycin, and pivmecillinam with resistance to nitrofurantoin,
amoxicillin and co-amoxiclav.

The patient is uncomplaining of dysuria, although a dipstick test on the ward was
positive for nitrites and negative for red blood cells. Her blood results were described
as unremarkable. A repeat midstream urine culture (MSU) taken the day before grew
E. Coli. The patient has a penicillin allergy.

What would you advise regarding treatment?

1. Amoxicillin
2. Fosfomycin
3. Pivmecillinam
4. Treatment with antibiotics not indicated
5. Trimethoprim

Explanation:

The correct answer is that treatment with antibiotics is not indicated.

The patient has no symptoms or clinical signs to suggest urinary tract infection. The
MSU shows asymptomatic bacteriuria.

NICE advises against the usage of urinary dipsticks to diagnose UTIs in female
patients over the age of 65.
Reference:

1. NICE: Urinary tract infection (lower) - women Last revised in October 2020

2. Treatment with antibiotics not indicated Correct answer selected


An 86-year-old man presents with severe left iliac fossa pain, with guarding. He has a
pulse of 126bpm regular, a temperature of 40.2oC and a blood pressure of 102/56
mmHg.

A CT scan shows a perforated appendicitis with abscess and free fluid in the
peritoneal cavity.

The patient has a history of hypertension and ischaemic heart disease. He currently
takes ramipril, bisoprolol, simvastatin and omeprazole.

The National Emergency Laparotomy Audit (NELA) will estimate the risk of death of
a patient following emergency laparotomy.

What is the time scale that NELA will predict the risk of death post-emergency
laparotomy?

3. 5 days
4. 10 days
5. 20 Days
6. 30 days
7. 50 days

Explanation:

Emergency laparotomy is associated with a significant risk of morbidity and mortality,


with a 30-day mortality rate of 10-15% amongst all patients. However, this score will
significantly vary between patient groups, NELA is designed to give an individual 30-
day mortality risk.

Reference:

1. Eugene, N., Oliver, C.M., Bassett, M.G., Poulton, T.E., Kuryba, A., Johnston, C.,
Anderson, I.D., Moonesinghe, S.R., Grocott, M.P., Murray, D.M. and Cromwell,
D.A., 2018. Development and internal validation of a novel risk adjustment model for
adult patients undergoing emergency laparotomy surgery: the National Emergency
Laparotomy Audit risk model. British journal of anaesthesia, 121(4), pp.739-748.
2. National Emergency Laparotomy Audit

3. 30 days Correct answer selected

An 83-year-old man with a past medical history of poorly controlled type two diabetes
(HbA1c 82), hypertension, and vertigo presented to hospital with chest pain and ECG
changes consistent with an acute coronary syndrome.

This was further confirmed by a significant rise in the patient’s troponin levels.

The patient was living independently at home prior to admission with no package of
care. However, he had shortness of breath on exertion with an element of left
ventricular dysfunction with a left ventricular ejection fraction of 40% and mild
diastolic dysfunction.

Percutaneous coronary angiography showed the patient had significant arterosclerotic


disease which was not amenable to coronary stenting and a coronary artery bypass was
arranged.

Prior to surgery, the patient felt unwell and a staff nurse took his pulse which was fast
and felt irregular, although a subsequent ECG showed sinus rhythm with a heart rate
of 112 bpm.

You are contacted by the anaesthetist as he is concerned regarding the possibility of


postoperative atrial fibrillation given the events earlier, and the patient’s tachycardia.
Bloods including renal function, liver function, bone profile, thyroid profile and full
blood count were reported as unremarkable.

Which is the most appropriate management to prevent post-operative atrial fibrillation


for this patient?

1. Amiodarone
2. Beta-blocker
3. Digoxin
4. No treatment indicated
5. Sotalol
1. Beta-blocker Correct answer selected
2.

Explanation:

NICE suggests that in patients at risk of atrial fibrillation who are undertaking
cardiothoracic surgery amiodarone, a beta-blocker (other than sotalol) or calcium
channel blocker are the most appropriate treatments.

When considering prescribing amiodarone treatment in the elderly, NICE considers


amiodarone as a caution and advises that:

Prescription potentially inappropriate (STOPP criteria) as first-line antiarrhythmic


therapy in supraventricular tachyarrhythmias (higher risk of side-effects than beta-
blockers, digoxin, verapamil or diltiazem). Therefore, the most appropriate option
would be to consider a beta-blocker.

The patient has a number of risk factors in developing postoperative atrial fibrillation,
including type two diabetes, element of cardiac failure, hypertension, Ischaemic heart
disease and advanced age.

Reference:

3. Yamashita, K., Hu, N., Ranjan, R., Selzman, C.H. and Dosdall, D.J., 2019. Clinical
risk factors for post-operative atrial fibrillation among patients after cardiac surgery.
The Thoracic and cardiovascular surgeon, 67(2), p.107.
4. BNF: Prescribing in the elderly
5. NICE: Atrial fibrillation: management Clinical guideline [CG180]Published date: 18
June 2014 Last updated: 01 August 2014
An 85-year-old lady who lived in a nursing home was discharged from hospital two
days ago. The admission to the hospital was a result of a witnessed fall, where she
landed on her left hip and was admitted under the Trauma and Orthopaedic team. A
hip X-ray was taken but it did not reveal any fractures. She was able to mobilise with
supervision prior to the fall but failed to engage with mobility assessment by a
physiotherapist. She was discharged back to the care home with a plan for ongoing
therapy in the care home. Her past medical history included dementia, hypothyroidism
and ischaemic heart disease.

Later, she re-presented to the Emergency Department having become bed-bound at the
care home. She was getting more confused and the staff became worried for her
wellbeing. Her oral intake had declined. Due to the underlying dementia, she was
unable to communicate her needs. During this second admission, there was no limb
asymmetry or shortening. Further clinical examination revealed an ejection systolic
murmur with soft heart second heart sound. Routine investigations during this
admission including a blood count, liver and renal profile were insignificant apart
from a chronic kidney disease. A CT brain scan did not reveal any acute pathology.
An electrocardiogram showed the heartbeat to be in sinus rhythm with left ventricular
hypertrophy.

You wish to establish the cause of her physical and cognitive decline. What is the
ideal next choice of investigation that can establish the cause?

1. Blood tests to look for re-feeding syndrome


2. Echocardiogram
3. Magnetic resonance imaging of the brain
4. Magnetic resonance imaging of the hip
5. Serum troponin test
Explanation:

Hip fracture is a major public health burden due to its associated high morbidity and
health/ social care costs. The higher mortality is due to the complications associated
with the fracture. Hence, prompt diagnosis and treatment is essential to improve
outcomes and to reduce pain and suffering for the affected individual.

Failure on the part of the affected individual to engage with the therapist or weight
bearing is one sign of an occult fracture affecting the hip or other joints of the lower
limbs. Re-feeding syndrome is a possibility when an individual is fed after a period of
prolonged starvation. This does not apply in this instance. An MRI brain is unlikely to
add any value to a CT brain in the absence of localising neurological signs. A cardiac
cause for her decline again is unlikely, given the fact that there is no evidence or
mention of decompensated cardiac function in the clinical story. In the context of a
recent fall, a hip fracture is very likely.

Reference:

1. National Clinical Guideline Centre, (2011) [The Management of Hip Fracture in


Adults]. London: National Clinical Guideline Centre. Available from: ww.ncgc.ac.uk.
2. The care of patients with fragility fracture. Published by the British Orthopaedic
Association. September2007.

3. Magnetic resonance imaging of the hip Correct answer


A 75-year-old man presented to his general practitioner with a sensation of abdominal
bloating, weight loss and an altered bowel habit of 4 weeks duration. His blood tests
including INR are below.

Investigation Result Normal range


Prothrombin 11.4 seconds 9.1 to 12.5
time, blood
INR, blood 1.0 0.9 to 1.2
APTT, blood 31 seconds 26 to 40
APTT ratio, 1.1 0.8 to 1.2
blood
A CT abdomen under the surgical team at his local hospital raised the possibility of a
splenic flexure colonic lesion. An urgent colonoscopy was organised. The patient's
medication included a direct oral anticoagulant (rivaroxaban) for non-valvular atrial
fibrillation and a history of lower limb deep vein thrombosis. He had been compliant
with his medication. The calculated creatinine clearance was 70 ml/ min. As the
medical registrar, you are called upon to advise when to discontinue the rivaroxaban to
facilitate colonoscopy and biopsy.

What would be appropriate to advise on the timing of the endoscopy?

1. Advise the last dose of rivaroxaban should be taken 24 hours pre-procedure


2. Advise the last dose of rivaroxaban should be taken 48 hours pre-procedure
3. Calculate creatinine clearance
4. Measure anti-Xa concentration
5. Organise coagulation tests
1.

Explanation:
This is an elective procedure and can be timed based on when the last dose of the drug
was taken. Blood coagulation tests do not guide the timing of the procedure. This
procedure carries a low risk of bleeding and in the presence of a CrCl > 30ml/min, it is
recommended to take the last dose of rivaroxaban 24 hours before the elective
procedure.

Reference:

2. The 2018 European Heart Rhythm Association Practical Guide on the use of non-
vitamin K antagonist oral anticoagulants in patients with atrial fibrillation. European
Heart Journal (2018) 39, 1330–1393
3. Sikorska J, Uprichard J. Direct Oral Anticoagulants: A Quick Guide. European
Cardiology Review 2017;12(1):40–5

4. Advise the last dose of rivaroxaban should be taken 24 hours pre-procedure Correct
answer

A 69-year-old man was admitted under the surgical team for a laparoscopic anterior
resection for rectal adenocarcinoma. He has a history of Parkinson’s disease (PD) that
was diagnosed seven years ago. His Parkinson's disease was well controlled on
dispersible co-careldopa 25mg/100mg four times per day and pramipexole salt
0.875mg three times per day. During surgery, the operation was converted to an open
laparotomy secondary to adhesions from a previous laparotomy. In the postoperative
period, he was allowed oral sips of fluids with a gradual increase in his feed. On day
two, he started to vomit with pyrexia, increasing tremor and muscular rigidity.
Abdominal sepsis was suspected due to the high temperature. The medical registrar
was called in to assist his management.

What would be the next important step in his management?

1. Advise he needs a repeat laparotomy to look for an anastomotic leak


2. Ensure dopaminergic medication is administered as a transdermal patch
3. Make a routine referral to a Parkinson’s disease specialist
4. Obtain blood cultures and commence an intravenous antibiotic
5. Organise a CT scan of the abdomen to assess if there is an intra abdominal abscess
1. Ensure dopaminergic medication is administered as a transdermal patch Correct
answer selected
2.

Explanation:

Vomiting and reduced oral intake in a patient with PD is likely to result in individuals
not taking their scheduled PD medications. In this setting, one should be alert to the
fact that dopaminergic stimulation in the brain can rapidly drop resulting in either
worsening PD symptoms or neuroleptic malignant-like syndrome. The symptoms of
pyrexia and muscular rigidity is a life-threatening complication and needs to be
addressed rapidly. These patients will need to be managed in a HDU setting and the
key feature would be to replace the dopamine either via enteral route or transdermally,
if enteral route is not possible. Subcutaneous apomorphine, a potent dopamine agonist,
is another alternative but it needs to be administered under specialist supervision. The
other options mentioned above are also valid, but dopamine replacement is an
important step that one cannot afford to miss.

Reference:

3. Alty, J., Robson, J., Duggan-Carter, P. and Jamieson, S., 2016. What to do when
people with Parkinson's disease cannot take their usual oral medications. Practical
neurology, 16(2), pp.122-128.
Acute illness (Diagnosis and Management)
You are reviewing a patient in clinic with a 12-month history of cholangiocarcinoma.
He has a biliary stent in situ for 3 months with good symptomatic relief of jaundice
and pruritus. He has felt unwell for 24 hours with nausea, sweats and malaise. On
examination, has a temperature of 38.6oC, a pulse of 82 bpm regular and a blood
pressure of 136/80 mmHg. Examination of the abdomen reveals tenderness over his
right upper quadrant.

Which of the following would be the most appropriate approach to the management of
this patient?

1. Admit from clinic and start on IV antibiotics


2. Arrange urgent ERCP
3. Arrange ultrasound scan of his abdomen
4. Obtain baseline bloods and inform GP that these need follow up
5. Start on oral metronidazole and cefalexin
1. Admit from clinic and start on IV antibiotics Correct answer selected

Explanation:

Ascending cholangitis may occur in patients with a partially obstructed or stented


common bile duct. It often causes severe systemic upset and should be promptly
treated with antibiotics.

The patient should be treated with intravenous antibiotics - common regimes include
Gentamicin and Metronidazole. If the patient is in septic shock, a single dose of
gentamicin should be given (5 mg/kg; maximum dose of 500 mg IV over 30 mins).

It is likely that this patient has a blocked stent and will require ERCP when stable.

Reference:

2. Cholangitis treatment
An 84-year-old female is admitted acutely as the GP is concerned due to the purple
discolouration of the urine in the catheter bag.

She has a long term catheter as she had a stroke 5 years ago and is now a resident in a
care home. The staff were concerned that the urine in the bag was purple. She has also
been a little more constipated of late.

There is nothing else of note and she feels reasonably well. She takes losartan,
bisoprolol, atorvastatin, apixaban and omeprazole.

On examination, she has a pulse of 76 bpm AF, a blood pressure of 108/78 mmHg and
a temperature of 37oC. Aside from a right-sided hemiparesis, there are no other
abnormalities other than purple discolouration of the urine in the catheter bag.

Investigations reveal:

Investigatio Result Normal range


n
FBC Normal
Urea 12.9 mmol/L 3 - 8 mmol/L
Creatinine 110 micromol/L 50 - 100 micromol/L
LFTs Normal
CRP 10 mg/L <5
Which of the following is the most appropriate treatment for this patient?
1. Change catheter; treat with nitrofurantoin
2. Reassurance; no specific treatment is needed
3. Stop losartan
4. Treat with lactulose
5. Treat with methylene blue

1. Change catheter; treat with nitrofurantoin Correct answer


2.

Explanation:

This is a typical description of purple urine bag syndrome where there is a urine tract
infection that results in the increased production of a blue pigment (indigo) and a red
pigment (indirubin) which results in the purple discolouration of the urine. Treatment
is focused on replacing the catheter, identification and treatment of the causative
organism together with treatment of constipation. The 1st line treatment for lower UTI
in a catheterised patient according to the BNF is nitrofurantoin, amoxicillin or
trimethoprim.

Beetroot produces a red discolouration of the urine.

Reference:

3. Al Montasir, A., & Al Mustaque, A. (2013). Purple urine bag syndrome. Journal of
family medicine and primary care, 2(1), 104–105. https://doi.org/10.4103/2249-
4863.109970
4. NICE: Urinary-tract infections
You are asked to see a patient on the ward who has recently undergone potentially
curative surgery for colorectal carcinoma. The patient is 2 days post-op and has been
doing quite well. However, you've been asked to see the patient as he has become
acutely unwell.

On examination, his temperature is 37.3oC, his saturations are 86% (previously 98%
on air) with a heart rate of 115/min in atrial fibrillation and blood pressure 80/40
mmHg. His chest sounds quite clear and there is no other abnormality to find.

Which of the following is the likely diagnosis?

1. Acute pulmonary oedema due to fast atrial fibrillation


2. Acute pulmonary oedema due to ischaemic heart disease
3. Septic shock
4. Chest sepsis
5. Pulmonary embolism
1. Pulmonary embolism Correct answer selected

Explanation:

The features of sudden collapse associated with shock and reduced saturations in the
post-operative period would suggest a diagnosis of large pulmonary embolism. The
fact that the chest is clear would argue against a diagnosis of acute pulmonary oedema.
Similarly, fast AF is a sequelae of PE. Treatment for this needs to be considered in the
context of his previous surgery but thrombolysis may be required if the PE considered
life-threatening.

Reference:

2. Please refer to the BTS Guideline for the outpatient management of pulmonary
embolism
A 75-year-old man is admitted being generally unwell. He has a known diagnosis of
lung cancer with liver, adrenal and contralateral lung metastases.

He was commenced on antibiotics for a lower respiratory tract infection yesterday and
today is very lethargic, dizzy and clammy.

You ask the staff nurse to check his observations and she’s very worried when his
heart rate is recorded at 123/min and blood pressure as 77/52 mmHg.

What would be your first course of action?

1. Give a dose of atenolol 25 mg od


2. Give 20 mg hydrocortisone orally
3. Give IV hydrocortisone 100 mg stat
4. Measure urea and electrolytes urgently
5. Perform a short synacthen test
1. Give IV hydrocortisone 100 mg stat Correct answer selected
2.

Explanation:

The history and clinical features are suggestive of an Addisonian crisis. You would
expect a high potassium and lowered sodium and raised urea (secondary to
hypovolaemia). A short synacthen test will confirm adrenal insufficiency but this takes
time and the results would not be back in time to influence management in this patient
over the weekend. The initial treatment should consist of rapid IV rehydration and IV
steroids. Oral hydrocortisone may be given to manage adrenal insufficiency but not in
crisis. Atenolol would only lower BP further.

Reference:
3. NICE Addison's disease

You are asked to review a 76-year-old gentleman who has cancer of the prostate and
widespread bony metastases. His back pain has become much worse this morning and
his legs feel weak and “wobbly”. He is unable to pass urine.

On examination, he has weakness of both legs with reduced tone and brisk reflexes of
the lower limbs with upgoing plantar responses. Sensation is difficult to assess but
seems impaired in the lower limbs up to the waist.

What is the most appropriate immediate course of action for this patient?

1. Commence IV hydrocortisone
2. Commence stat 16 mg oral dexamethasone
3. Commence IV morphine
4. Commence IV pamidronate
5. Commence IV zolendronate
1. Commence stat 16 mg oral dexamethasone Correct answer

Explanation:

The history suggests spinal cord compression and with neurological symptoms and
signs, spinal instability cannot be excluded. The patient should therefore be nursed flat
in bed and log rolled as necessary until spinal stability is known. The urgent request
for an MRI and commencement of steroids is recommended by NICE - recommending
16 mg Dexamethasone stat.

Reference:

2. NICE guidelines on malignant cord compression


A 76-year-old male who is being treated for renal cell carcinoma and has just received
trimethoprim for a urine infection. You are called to see him as within minutes of
receiving the drug he developed facial swelling, light-headedness and difficulty
breathing.

On immediate assessment, you note that he has swelling of the lips and eyes with a
widespread red blotchy rash. He is particularly wheezy, with a respiratory rate of
28/min, a pulse of 110 bpm and supine blood pressure of 100/70 mmHg.

Which of the following would be the most appropriate immediate treatment for this
patient?

1. IM adrenaline 1:1000 0.5ml


2. IM adrenaline 1:10000 0.5ml
3. IM chlorpheniramine 10mg
4. IV Hydrocortisone 100mg
5. IV adrenaline 1:1000 0.5ml

1. IM adrenaline 1:1000 0.5ml Correct answer selected


2.

Explanation:

This patient is likely to have developed a severe allergic reaction as evidenced by the
swelling and rash, but more worryingly there are also features of anaphylaxis, as
evidenced by the difficulty breathing and wheeze. He requires urgent treatment and
the acknowledged first-line treatment for anaphylaxis is IM adrenaline 1:1000 0.5ml.
which is typically administered via an adrenaline autoinjector.
Adjuvant therapies that can be given include high flow oxygen, IV fluids, nebulised
bronchodilators, corticosteroids and antihistamines. IV adrenaline is not recommended
as a first-line agent and should only be given by experienced specialists.

Reference:

3. Emergency treatment of anaphylactic reactions: Guidelines for healthcare providers

A 72-year-old man has been admitted under the surgeons for a prostate biopsy as an
investigation into possible prostate carcinoma. He has a history of atrial fibrillation
(AF) for which he is being treated with apixaban, digoxin, and bisoprolol. The
surgeons have asked for an opinion regarding the management of his anticoagulation
as they have scheduled his procedure for 24 hrs hence. He is nil by mouth and he is
receiving IV digoxin.

On examination, he has a temperature of 37.2oC, his pulse is 88 bpm in AF, and he


has a blood pressure of 132/80 mmHg.
Which of the following is the most appropriate approach to the management of his
anticoagulation up to the procedure and immediately post-operatively?

1. Stop apixaban and switch to Low Molecular Weight Heparin (LMWH) now and omit
on the day of the procedure then restart LMWH on post-operative day 1
2.
3. Stop apixaban and switch to Unfractionated heparin now and up to the procedure,
stopping an hour or so before the procedure then restarting Low Molecular Weight
Heparin (LMWH) on postoperative day 1
4.
5. Stop apixaban, give reversal agent, andexanet and anticoagulate with low molecular
weight heparin now, omitting the day of the procedure. Restart LMWH on post-
operative day 1
6.
7. Stop apixaban, give reversal agent, andexanet and do not anticoagulate for the next 24
hrs restarting apixaban on postoperative day 1 if creatinine clearance > 80 min/ml
8.
9. Stop apixaban, no need for bridging anticoagulation and restart apixaban the day after
the procedure if creatinine clearance > 80 min/ml

1. Stop apixaban, no need for bridging anticoagulation and restart apixaban the day after
the procedure if creatinine clearance > 80 min/ml Correct answer selected

Explanation:

This patient’s apixaban will need to be stopped for the procedure as he is currently nil
by mouth (NBM).
The above scenario does not provide a rationale for commencing anticoagulation.
However, the patient’s CHADVASC score is 1 (1 for age; 0 for male). Therefore, his
risk is regarded as low moderate with an annual stroke risk of 0.9% if untreated.
However, the risk of stroke/TIA through omitting the anticoagulation for a day or so
would be very low and would, therefore, need to be weighed against the risk of
bleeding in the context of imminent surgery. Thus, anticoagulation would need to be
stopped at this stage. Bridging treatment would be considered in the following
scenarios:

2. Patients with a previous stroke/TIA in the last three months.


3. Patients with a previous stroke/TIA and three or more of the following risk factors:
So, in this patient’s case, we could regard this as a moderately high risk of bleeding,
the Direct Oral Anticoagulant (DOAC) should be stopped, no anticoagulation given up
to the time of the procedure and during the procedure, and then the DOAC should be
restarted 24-48 hrs after the procedure.

Reference:

4. Algorithm for the Peri-operative Management of Anticoagulants and Antiplatelet


agents in Adult patients
5. Keeling, D., Tait, R.C., Watson, H. and British Committee of Standards for
Haematology, 2016. Peri-operative management of anticoagulation and antiplatelet
therapy. Br J Haematol, 175(4), pp.602-613.

Explanation:

Although the option “Explain to the patient (and family) why it might be appropriate
to deactivate the ICD and with the patient's agreement, proceed to deactivate with
magnet” is a reasonable approach, the best approach would be for the management
plan to be multidisciplinary and to involve the cardiologists. Given the concerns that
this patient may be entering the terminal phase, this should not be a routine referral,
although it does seem appropriate that a DNACPR order be discussed with the patient
and completed. Also, many cardiologists have remote monitoring systems in place for
their ICD patients and if you were to inactive a device with a magnet without
informing them the device downloads could be very confusing!

The gold standard deactivation method in such cases should be a visit from a cardiac
physiologist with a programmer to electronically deactivate the device (the
bradycardia pacing function of the ICD remains functional). The use of magnets
should be reserved for emergencies and out-of-hours when a cardiac physiologist may
not be available to deactivate the device. If a magnet is to be used it should be secured
in place e.g. with sleek and not removed, as soon as it’s removed the device becomes
active and at risk of delivering ‘shocks’. It is not good practice to leave the magnet at
the bedside for use should the patient receive a shock from the device.

There is no mention of loss of capacity in the question stem and therefore no need to
make a best interest decision in this case.

Reference:

1. A good summary article that discusses the practical, legal and ethical considerations
when managing patients with ICDs at the end of life. Russo, JE. Deactivation of ICDs
at the End of Life: A systematic review of clinical practices and Provider and Patient
Attitudes. AJN. 2011; 111(10):26-35.
Free text available at: http://journals.lww.com/ajnonline/toc/2011/10000

2. Contact the patient’s cardiology team (or on-call team if OOH) to discuss
appropriateness of ICD deactivation and then discuss with the patient (and family).
Correct answer selected
3.

You are reviewing a 70-year-old patient in the ward with severe chronic obstructive
pulmonary disease (COPD) who is recovering from community-acquired pneumonia
(CAP).

He is experiencing diarrhoea.

On further questioning, he tells you it is green/brown in colour and he is opening his


bowels up to 7 times a day.
The FY1 has sent a stool sample to the lab. His observations are stable. Investigations
reveal normal U+Es, white cell count 14x109/L (3-10) with a neutrophil count of
10x109/l.

Abdominal examination is unremarkable. What would you advise?

1. Await stool results


2. Obtain abdominal X-ray
3. Start loperamide
4. Start oral metronidazole
5. Start oral vancomycin

1. Start oral vancomycin Correct answer selected

Explanation:
The patient has recently completed antibiotics for a CAP, is elderly and is at risk of
C.difficile.

Oral Vancomycin 125 mg QDS is the first line treatment advised by NICE for mild to
moderate clostridium infection.

Vancomycin must be administered orally because intravenous administration does not


achieve gut lumen minimum therapeutic concentration.

Clostridia are anaerobic, spore-forming rods (bacilli). C. difficile is the most serious
cause of antibiotic-associated diarrhoea and can lead to pseudomembranous colitis.

C. difficile is transmitted from person to person by the faecal-oral route. However, the
organism forms large numbers of heat-resistant spores, and these are not killed by
alcohol-based hand cleansers or routine cleaning of surfaces. Thus, these spores
remain viable in the hospital or nursing home environment for long periods of time
and, because of this, the bacteria can be cultured from almost any surface in the
hospital. Once spores are ingested by a patient, they pass through the stomach
unscathed because of their acid resistance. They germinate into vegetative cells in the
colon upon exposure to bile acids, and multiply.

Reference:

2. Scenario: Diarrhoea - antibiotic associated. Last revised in July 2021

You are reviewing a 78-year-old man on the ward who has had potentially curative
surgery for colorectal carcinoma. The patient is 2 days post-op and feeling very
nauseous. Whilst the patient is on the commode, he becomes pale and breathless.
On examination, he is distressed and cyanosed with a heart rate of 110/min, blood
pressure 80/40 mmHg, a respiratory rate of 30/min and his neck veins are engorged.
His saturations are 86% (previously 98% on air). He already has an IVI in situ.

Which would be the most appropriate initial course of action for this patient?

1. Consider giving alteplase


2. Get an urgent bedside ECHO
3. Get further IV access
4. Give a stat dose of unfractionated heparin followed by LMWH
5. Give high flow oxygen

1. Give high flow oxygen Correct answer


Explanation:

The important aspect to his management is initial resuscitation of this sick patient with
a likely large PE. The initial approach would always be ABC. He has a patent airway
but needs oxygen, quickly followed by IV access. The clinical scenario is depicting a
massive PE and step-up care to HDU required with consideration of the most
appropriate treatment in this patient who has had very recent surgery.

Reference:

2. Resus Adult advanced life support guidelines


3. NICE: Suspected pumonary embolism

You are the medical registrar and are asked by your surgical colleagues about a patient
who requires an emergency laparotomy for an acute abdomen. The 80-year-old man is
treated with warfarin for atrial fibrillation and his INR is 3.5.

Which of the following do you advise?


1. Give IV fresh frozen plasma and vitamin k
2. Give IV fresh frozen plasma only
3. Give IV prothrombin complex concentrate and vitamin K
4. Give IV prothrombin complex concentrate only
5. Give IV vitamin K only

1.

Explanation:
In this emergency situation requiring urgent correction of the anticoagulation to permit
surgery, the patient requires an infusion of IV vitamin K (takes about 4 hrs to work)
and PCC (takes about 15 mins). These newer agents provide a far better reversal of
coagulation than FFP.

Reference:

2. Keeling D et al (2011) BJH Guidelines on oral anticoagulation with warfarin

3. Give IV prothrombin complex concentrate and vitamin K Correct answer selected

An 81-year-old man is admitted following a fall at home. He is unsure why he fell or


if he blacked out, but has a laceration to his head so-called ambulance. In his past
medical history, he takes apixaban for lone atrial fibrillation.

On admission his GCS was 15, he had a laceration to his left temporal region which
had now stopped bleeding. Neurological examination was normal but a CT head scan
was performed and it revealed a left subdural haematoma. On coming back from CT
scan, his GCS deteriorated to 12. The neurosurgeons were contacted and intend to
transfer for urgent evacuation.

Which of the following should be undertaken prior to surgery?

1. Give fresh frozen plasma


2. Give IV prothrombin complex concentrate
3. Give IV tranexamic acid
4. Give IV vitamin k
5. No prior treatment required; patient can undergo neurosurgery

Explanation:
This patient is taking one of the direct oral anticoagulants (NOAC), apixaban which is
a direct Xa inhibitor and has developed a subdural haematoma following a fall.
Surgery is deemed necessary due to clinical deterioration and therefore the
anticoagulative effect of apixaban should be reversed with a PCC such as octaplex or
factor VIII inhibitor bypass activity.

Reference:

1. Thrombosis Canada (2013) Management of bleeding in a patient receiving a new oral


anticoagulant.

2. Give IV prothrombin complex concentrate Correct answer selected

Explanation:
Hypercalcaemia and elevated parathyroid hormone-related peptide levels are most
commonly associated with squamous cell carcinoma. Small cell carcinoma and
cancers with bone metastases may be also be associated with hypercalcaemia but not
with increased PTHrH levels.

Reference:

1. Khanna, A, Periwal, P, Talwar, D. Small cell carcinoma lung presenting as life-


threatening hypercalcemia - a rare association, in Lung India, v.32(6); Nov-Dec 2015,
PMC44663891

2. Squamous cell carcinoma Correct answer selected

A 72-year-old man is sent to emergency admissions after his GP found that his INR
was >8 yesterday. He has been taking warfarin for atrial fibrillation for the last 3 years
and has had good INR control until now. He is recovering from a chest infection, for
which he had a recent course of antibiotics, but is otherwise well.
Which of the following is the most appropriate approach to the management of this
patient?

1. Stop warfarin and give IV octaplex


2. Stop warfarin and give IV vitamin K 0.5 mg
3. Stop warfarin and give IV vitamin K 2 mg
4. Stop warfarin and give oral vitamin K 2 mg
5. Stop warfarin only

1. Stop warfarin and give oral vitamin K 2 mg Correct answer

Explanation:
This patient has an elevated INR but no sign of haemorrhage. Therefore guidelines
suggest that the patient can be managed with a dose of oral vitamin K 1-5 mg and by
stopping the warfarin.

Reference:

Guidelines on oral anticoagulation with warfarin

You are called to see a 78-year-old man transferred from the hospice. He has a
diagnosis of metastatic pancreatic carcinoma. He was admitted from home for control
of liver capsule pain the previous day. He is currently ‘for resuscitation’ because when
the team discussed resuscitation with him, he was very clear that he would want to be
resuscitated and have ‘everything done’. He has become increasingly breathless
overnight.

Observations:

Investigation Result
Blood pressure 110/72 mmHg
Heart rate 106/min regular
Respiratory rate 25/min
Oxygen sats 93% on air
Temperature 37.0°C
Clinical examination of the respiratory system finds bilateral air entry with clear lung
fields. CXR is normal.

Which of the following would be the most appropriate treatment for this patient?

1. Commence lorazepam 0.5 mg sublingually or orally for anxiety


2. Commence prophylactic dose of low-molecular-weight heparin
3. Commence treatment dose of low-molecular-weight heparin
4. Keep the patient comfortable and employ supportive treatment only
5. Suggest the patient breathes into a paper bag

Explanation:
Acute onset shortness of breath in the middle of the night in a patient with cancer (in
particular brain, pancreas, ovary, lung, bladder, stomach, uterus) has to be a
pulmonary embolism until proven otherwise, especially with raised RR, tachycardia
and sats on the cusp of the desaturation curve. The biggest mistake is not to consider it
as a possibility and to diagnose panic attack instead whereas panic attacks need to be a
diagnosis of exclusion. Clearly, LMWH in this scenario can also be palliative in its
effect. Similarly, the patient should be counselled regarding future care and the
decision on resuscitation further discussed.

Reference:

1. Ansari, D. Ansari, D. Andersson, R. Andren-Sandber, A. 'Pancreatic cancer and


thromboembolic disease, 150 years after Trousseau,' in Hepatobilary Surg Nutr, 2015
Oct; 4(5): 325-335. doi: 10.3978/j.issn.2304-3881.2015.06.08
2. NICE: Suspected pulmonary embolism

3. Commence treatment dose of low-molecular-weight heparin Correct answer selected


A 80-year-old woman presented to the unit with a 2-day history of left leg pain and
swelling. She had a background history of metastatic non-small cell lung cancer with
cerebral metastases for which she had received cranial irradiation, chemotherapy and
dexamethasone.

The cerebral metastases were stable and she was otherwise living an independent life.

On examination, observations were within normal ranges and she appeared well
although was slightly Cushingoid in appearance and had a swollen and tender left leg.

Doppler studies revealed that she had a proximal deep vein thrombosis of her left leg.

Which of the following would be the most appropriate treatment for this patient’s
DVT?

1. Apixaban
2. Inferior vena caval filter
3. Initial low molecular weight heparin and warfarin
4. Initial unfractionated heparin and warfarin
5. Long term low molecular weight heparin
1. Long term low molecular weight heparin Correct answer

Explanation:

The patient has a DVT and this should be treated. The complication of cerebral
metastases may prompt some to consider the risk of intracerebral haemorrhage too
high with anticoagulation, and therefore to prefer an IVC filter. However, the evidence
indicates that whilst there is a risk of cerebral haemorrhage with cerebral metastases,
this risk is similar whether the patient is receiving anticoagulation or not. Thus the
most appropriate approach to the management of this patient’s thromboembolism is
LMWH. Filters are themselves problematic without concomitant anticoagulation with
obstruction and thrombotic problems.

Reference:

2. Nelson, R., 2015. Anticoagulant use safe in patients with brain metastases. Medscape.
3. Gerber, D, E. Grossman, S, A. Streiff, M, B. Management of venous thromboemolsim
in patients with primary and metastatic brain tumors. Journal of Clinical Oncology 24,
no. 8 (March 10, 2006) 1310-1318. doi: 10.1200/JCO.2005.04.6656
4. Lin, R, J. Green, D, L. Shah, G L. Therapeutic anticoagulation in patients with
primary brain tumors or secondary brain metastasis, in The Oncologist, 2018 Apr;
23(4): 468-473.
1. Ovarian cancer Correct answer selected

Explanation:

In order to get the answer to this question, it is important to work out the
dermatological diagnosis which is dermatomyositis as reflected by the heliotrope rash
on the eyelids and Gottron’s patches on the hands and feet. Whilst dermatomyositis
can occur in isolation, there is a 3 fold increased risk of underlying malignancy and
the weight loss in this patient should point toward that diagnosis. Cancers associated
with dermatomyositis include ovarian, lung, pancreatic and colorectal. Treatment is
usually with immunosuppressive therapy, typically steroids, methotrexate or
cyclophosphamide.

Reference:

2. Dermatomyositis
You are called to the cardiac arrest of a 76-year-old male who was admitted with
sepsis due to a chest infection. On your arrival, the team are performing chest
compressions at a rate of 30:2 and the anaesthetist is preparing to intubate the patient.
He has a peripheral patent IV line in place. You attach the defibrillator and pause CPR

to perform the first rhythm check. The rhythm is as follows:

Which of the following is the most appropriate initial approach to the management of
this patient?

1. Give 1 mg Adrenaline IV
2. Give 300 mg Amiodarone IV
3. Give a 150J shock from a defibrillator
4. Give a 300J shock from a defibrillator
5. Perform a precordial thump

Explanation:

The rhythm strip reveals ventricular fibrillation and the initial treatment according to
guidance is to give at least a 150J shock from a biphasic defibrillator. For the
speciality exam, one needs to be very familiar with the latest guidance on resuscitation
procedure.

Reference:
1. Resuscitation Council (UK) (2015) Guidelines - Adult Advanced Life Support

2. Give a 150J shock from a defibrillator Correct answer

You are called to the cardiac arrest of a 77-year-old woman who was admitted with
chest pain two hours ago. She was being treated for an acute coronary syndrome but
collapsed suddenly and staff could not palpate a central pulse. The arrest call went out
and on your arrival, the team are resuscitating at a rate of 30:2. She has a peripheral
patent IV line in place. The defibrillator is attached and CPR is briefly stopped to
perform a rhythm check. The rhythm tracing is as shown.

Which of the following is the most appropriate approach to the management of this
patient?

1. Give 1 mg Adrenaline IV
2. Give 300 mg Amiodarone IV
3. Give a 150J shock from a defibrillator
4. Give a 300J shock from a defibrillator
5. Perform a precordial thump
1. Give 1 mg Adrenaline IV Correct answer selected

Explanation:

The rhythm strip reveals asystole – non-shockable rhythm. The initial treatment is
adrenaline 1 mg IV as soon as possible. Resuscitation in pulseless electrical activity
and asystolic arrest is unlikely to be successful unless a reversible cause can be found
– eg electrolyte disturbance (hyperkalaemia), tamponade, hypoxia, PE.

Reference:

2. Resuscitation Council (UK) (2015) Guidelines - Adult Advanced Life Support


A 72-year-old male with known coronary artery disease is receiving cardiopulmonary
resuscitation for a VF arrest. He has received 3 escalating shocks over the last 5 mins.
He remains pulseless in VF therefore CPR is continued.

Which of the following is the next step in the management of the patient?

1. Give 1 mg atropine IV
2. Give 5 mg adrenaline IV
3. Give 100 mg lidocaine IV
4. Give 300 mg amiodarone IV with 1 mg adrenaline IV
5. Give 2 g magnesium IV
Explanation:

The Resus council UK, now advise to give adrenaline 1 mg IV (IO) after the 3rd shock
for adult patients in cardiac arrest with a shockable rhythm, and to give amiodarone
300 mg IV (IO) for adult patients in cardiac arrest who are in VF/pVT after three
shocks have been administered.

Reference:

1. Resuscitation Council (UK) (2021) Guidelines - Adult Advanced Life Support

2. Give 300 mg amiodarone IV with 1 mg adrenaline IV Correct answer selected


You attend a cardiac arrest on MAU. A 75-year-old man has collapsed and upon
assessing his airway, breathing and circulation you determine he has no cardiac or
respiratory output. You secure his airway and commence cardio-pulmonary
resuscitation. You perform an ECG rhythm trace (below):

What is the correct initial management?

1. 1 mg adrenaline IM
2. 1 mg adrenaline IV
3. 0.5 mg atropine
4. DC cardioversion
5. External pacing
Explanation:

The rhythm above shows pulseless electrical activity.

The correct management according to guidelines is to give 1 mg adrenaline IV and


continue with CPR.

PEA is not a shockable rhythm so is not appropriate, and external pacing should only
be used if the patient has a cardiac output.

Reference:

1. Resus 2021 resuscitation guidelines/adult advanced life support guidelines.

2. 1 mg adrenaline IV Correct answer


You are part of the cardiac arrest team at a local district general hospital. You are
called to a 78-year-old patient with ventricular tachycardia, but who still has a cardiac
output.

Which of the following scenarios would you deliver urgent electrocardioversion?

1. Pulsed VT with BP 101/80 mmHg


2. Pulsed VT with preceding symptoms of chest pain, but who is now pain free
3. Pulsed VT with signs of cardiac failure
4. Pulsed VT with symptoms of shortness of breath
5. All of the above
Explanation:

ALS guidelines recommend immediate DC cardioversion for all decompensated


tachycardias. Criteria for decompensation include:

1. Chest pain
2. Signs of cardiac failure
3. Hypotension
4. Impaired GCS.
Reference:

5. Resus: Adult advanced life support guidelines

6. Pulsed VT with signs of cardiac failure Correct answer selected


An 83-year-old woman presents with a 1-day history of drowsiness, confusion and
urinary incontinence. She has a past history of type 2 diabetes for which she takes
metformin and gliclazide together with ramipril and amlodipine for associated
hypertension.

On examination, her heart rate is 120 bpm, blood pressure 80/60 mmHg, temperature
35.4oC, oxygen saturation is 92% on room air and she has a respiratory rate of 28/min.
She is drowsy but rousable. Her results reveal a white cell count of 13.1 (4-10), a CRP
of 110 mg/dl (<5) and a lactate of 5 mmol/l (<1.5).

Which of the following is the most appropriate immediate management?

1. Broad-spectrum antibiotics and arterial blood gas sample


2. Central venous line insertion and rewarming
3. Intravenous fluid bolus and broad-spectrum antibiotics
4. Intravenous fluid bolus and rewarming
5. Noradrenaline and broad-spectrum antibiotics
Explanation:

The patient has severe sepsis. Fluid resuscitation and broad-spectrum antibiotics (after
blood cultures) are part of the "Sepsis Six" management, which should be completed
within the first hour of treatment. Central venous access and use of inotropes should
be considered later, if the patient does not respond to fluid resuscitation. Infection in
the elderly may not result in raised temperature due to blunted inflammatory response.
Gram-negative sepsis can cause low temperature.

Reference:

1. Surviving Sepsis Campaign: International Guidelines for Management of Severe


Sepsis and Septic Shock: 2012. Critical Care Medicine. 41(2):580-637
2. Surviving Sepsis Campaign Guidelines for Adult Patients
3. Howell, M. Davis, A. M. Manamgent of Sepsis and Septic Shock, JAMA,
2017;317(8):848. DOI:10.1001/JAMA.2017.0131
4. NICE Guidelines: Sepsis

5. Intravenous fluid bolus and broad-spectrum antibiotics Correct answer selected


This is the ECG obtained on a 77-year-old man with type 2 diabetes who was admitted
with lethargy and tiredness.

Which electrolyte abnormality would be consistent with this ECG?

1. Hypercalcaemia
2. Hyperkalaemia
3. Hypocalcaemia
4. Hypokalaemia
5. Hypothermia
1.

Explanation:

This ECG shows tented T waves and p waves with a tachycardia suggestive of
hyperkalaemia.

Reference:

2. Kenny, B. J. Brown, K. N. ECG T Wave, StatPearls.

3. Hyperkalaemia Correct answer selected


Explanation:

Metformin is known to cause vitamin B12 deficiency, so should always be suspected


as a cause of raised MCV or megaloblastic anaemia. It is thought to inhibit vitamin
B12 absorption from the ileum by affecting the calcium-dependent membrane action.

When metformin is stopped, the vitamin B12 deficiency resolves; if it does not, an
alternative cause must be sought.

Long term metformin therapy has been known to cause a much more severe condition,
with deterioration in mental state and peripheral neuropathy.

Reference:

1. NICE: Metformin Hydrochloride


2. Medicine Leaflet: Metformin
3. Pernicious Anaemia and B12 Deficiency
4. Metformin-related Vitamin B12 Deficiency

5. Metformin Correct answer selected


A 76-year-old man has a history of mild exercise-induced angina, atrial fibrillation,
hypertension and type II diabetes mellitus.

He is taking pioglitazone, metformin, isosorbide mononitrate, digoxin, atorvastatin,


bendroflumethiazide and warfarin.

He is admitted after presenting with increasing breathlessness, a blood pressure of


148/100 mmHg, an irregular pulse of 84 bpm and swollen ankles.

Which of the medications is most likely to have contributed to his presentation?

1. Atorvastatin
2. Bendroflumethiazide
3. Digoxin
4. Metformin
5. Pioglitazone
Explanation:

Studies suggest that approximately 5% of patients taking glitazones develop oedema


which may precipitate heart failure.

Oedema is often worsened with the addition of other therapies such as calcium
antagonists.

NICE recommends that pioglitazone should be avoided in people with heart failure,
and those at risk of fracture.

If used in patients taking insulin, the danger is further increased. Treatment is to stop
pioglitazone and treat heart failure as usual.

References:

1. Medline Plus: Pioglitazone


2. Medicine Leaflet: Pioglitazone
3. NICE Clinical Guidelines 66: Type 2 Diabetes

4. Pioglitazone Correct answer selected


A 80-year-old man was diagnosed with herpes zoster (shingles) and treated with
aciclovir. The rash subsided but significant pain remained which was managed with
co-codamol 30/500 and amitriptyline.

His regular medication consists of metformin, gliclazide, lisinopril and simvastatin. He


is admitted with acute lower abdominal pain and an inability to pass urine.

Which drug has caused this man’s problems?

1. Acyclovir
2. Amitriptyline
3. Gliclazide
4. Lisinopril
5. Simvastatin
1.

Explanation:

This man has acute urinary retention. Tricyclic antidepressants, including


amitriptyline, have anticholinergic adverse effects including dry mouth, blurred vision,
constipation, urinary retention, and drowsiness. Some tolerance develops to these
effects and patients should be encouraged to persevere as long as these effects are
mild. Severe adverse effects, such as acute urinary retention require prompt treatment.
It can be difficult and demoralising when the adverse effects of a drug manifest
themselves before the beneficial ones, so it is important to know when this will
happen.

References:

2. NICE: Amitriptyline
3. Medicine Leaflet: Amitriptyline

4. Amitriptyline Correct answer selected


A 78-year-old woman with a history of depression is admitted after taking an overdose
of atenolol. She had taken approximately 40x50mg tablets 2 hours earlier. On
examination, she had a blood pressure of 86 mmHg systolic and a pulse of 48 bpm
regular. Her ECG revealed a sinus bradycardia without clear p waves. Her GCS was
13. She is administered IV fluids but which of the following should initially be given
to this patient?

1. IM adrenaline
2. IV adrenaline
3. IV dobutamine
4. IV glucagon
5. IV hydrocortisone
Explanation:

This woman has taken a life-threatening overdose of beta-blockers and is critically ill.
She requires immediate transfer to ICU; the most appropriate initial treatment for a
beta-blocker overdose would be IV glucagon.

Other approaches to treatment include dialysis in atenolol overdose as well as High


Dose Insulin Infusion.

Similarly, once the airway is secure, one could also use activated charcoal.

References:

1. NICE: poisoning emergency treatment

2. IV glucagon Correct answer selected


A 79-year-old female patient has a history of osteoarthritis and hypercholesterolaemia
for which she takes co-codamol and simvastatin.

She was recently prescribed Lisinopril as she had a third successive hypertensive
reading at her GP surgery. Additionally, she has been prescribed cinnarizine for
motion sickness.

A week later she presents in the pharmacy asking for something to treat a raised, red
rash on her legs, arms, and trunk.

What medication is most likely to be responsible for these symptoms?

1. Cinnarizine
2. Codeine
3. Lisinopril
4. Paracetamol
5. Simvastatin
1.

Explanation:

ACE inhibitors are known to cause urticarial rash, especially lisinopril. The lisinopril
should be stopped since there is some risk of anaphylaxis and alternative
antihypertensive medication should be considered. A suitable antihistamine should be
prescribed, but in this case, the patient should be warned not to take it with
cinnarizine, which is also an antihistamine.

Reference:

2. NICE: Lisinopril
3. Medicine Leaflet: Lisinopril
4. Urticaria
5. Angioedema and Urticaria with Angiotensin-Converting Enzyme Inhibitors P. I.
Pillans, D. M. Coulter and P. Black European Journal of Clinical Pharmacology
Volume 51, Number 2 / October, 1996
6. Lisinopril Correct answer selected

An 80-year-old female presents as her GP is concerned about her INR which is now 6.
She has taken warfarin for approximately 2 years as anticoagulation for atrial
fibrillation with the INR kept around 2.5 on a dose of 3-4 mg of warfarin daily.

Her only other treatment includes losartan and amlodipine for hypertension.

On this occasion, her INR is 6.2. On further discussion, she informs you that she has
most recently embarked on lifestyle and dietary changes.

Which of the following products is likely to be responsible for this change in INR?

1. Cranberry juice
2. Fish oil tablets
3. Garlic tablets
4. Cherry juice
5. Vitamin C tablets
1.

Explanation:

There is some evidence to suggest that cranberry juice enhances the anticoagulant
effect of warfarin. It is important to recall that many patients take cranberry juice as
prophylaxis against urinary tract infection.

It is also taken by some for the flavinoids (‘antioxidants’) that it contains, which are
thought to reduce the risk of developing various malignancies. Unfortunately, they are
also known to inhibit cytochrome P450 enzymes.

Warfarin is predominantly metabolised by P450 CYP2C9 and if this is inhibited, the


INR will obviously rise.

Patients taking warfarin should avoid cranberry juice, at least until more is known
about this potential interaction.

It should be noted, however, that the potential interaction was observed in a patient
who had consumed extremely large quantities of cranberry juice and very little else for
several days. Consumed in moderation, the juice is probably unlikely to interfere with
warfarin.

Reference:

2. Possible interaction between warfarin and cranberry juice. Rafe Suvarna, Munir
Pirmohamed, Leigh Henderson. BMJ;2003;327:1454,
doi:10.1136/bmj.327.7429.1454.
3. http://www.patient.co.uk/medicine/Warfarin.htm

Cranberry juice Correct answer selected

A 77-year-old woman developed a subdural haematoma following a road traffic


accident. She underwent neurosurgery and has made a good recovery.

Her medication at discharge consisted of lisinopril, bendroflumethiazide, simvastatin


and phenytoin.

Three months later she re-attends clinic and informs you that her dentist said that she
had something called 'gingival hyperplasia'.

Which drug is likely to have caused this?


1. Bendroflumethiazide
2. Lisinopril
3. Phenytoin
4. Simvastatin
5. None of the above

1.

Explanation:

Phenytoin, along with calcium channel blockers and cyclosporine, causes gingival
hyperplasia. It is often asymptomatic, though patients with poor dental hygiene may
report pain and bleeding on tooth brushing and in extreme cases displacement of the
teeth and periodontal disease has been reported. It is obviously the role of the dentist
to educate on the matter of dental hygiene, though it does not hurt to make sure that
this occurs. The patient will need to have her medication changed at the neurology
clinic.

Reference:

2. NICE: Phenytoin
3. Drug-induced Gingival Hyperplasia

Phenytoin Correct answer selected

A 78-year-old man presents a prescription for co-amoxiclav and paracetamol after


being bitten on the hand by his pet dog. He has a history of epilepsy for which he takes
sodium valproate.

Six days later, he returns feeling unwell and would like to know if his medication may
be the cause.

He says he has been passing very pale stools but dark urine, has abdominal pain, fever
and an intense itch.
Which drug is most likely to be the cause of this man’s symptoms?

1. Amoxicillin
2. Sodium Valproate
3. Co-amoxiclav
4. Paracetamol
5. None of the above

1.

Explanation:

Cholestatic jaundice can occur either during or shortly after the use of co-amoxiclav.
The risk of acute liver toxicity is about 6 times greater with co-amoxiclav than with
amoxicillin. Cholestatic jaundice is more common in males and in patients over the
age of 65 years. Jaundice is usually self-limiting and very rarely fatal.

The duration of treatment should be appropriate to the indication and should not
usually exceed 14 days.

Although paracetamol overdosage is associated with liver damage, there is nothing to


indicate that this patient has received an overdose.

Liver dysfunction (including fatal hepatic failure) in association with valproate,


usually occurs in the first 6 months of therapy. Valproate is unlikely to be responsible
for cholestatic jaundice in this patient, as he has been taking it for the last 5 years.

However, valproate is contraindicated in patients with active liver disease and


treatment should be reviewed if the patient’s liver dysfunction continues to deteriorate
after stopping co-amoxiclav.

Reference:

2. NICE: Co-amoxiclav
3. Visentin, M. Lenggenhager, D. Gai, Z. Kullak-Ublick, G, A. Drug-induced bile duct
injury, in Viochimica et Biophsyica Acta (BBA) - Molecular basis of disease, vol
1864.4.B (2018), pp. 1498-1506

4. Co-amoxiclav Correct answer selected

An 83-year-old lady is admitted from home as she has been more breathless of late.
She lives alone and usually manages well. She was seen by her daughter who felt she
was more breathless than usual as she has a background of COPD associated with
smoking.

The GP decided to admit her. She used inhalers and took zopiclone at night to help her
sleep.
Other than some mild memory problems she had little else of note in her history. She
does not drink any alcohol but smokes about 5 cigarettes daily.

On examination, she appears well, her temperature was 37oC, pulse 80 bpm reg and
blood pressure 110/80 mmHg. Her oxygen saturations were 94% on air. Other than
occasional wheeze there was nothing else of note on examination.

Full blood count, U+Es were normal and her CRP was 15 mg/l (<5). She was given
oral antibiotics by the night team and was to be reviewed by the consultant the
following morning.

The following morning she was agitated, aggressive and would not let anyone perform
any observations.

She would lock herself in the toilet and felt that people were intent on harming her.

She was walking around the unit quite unperturbed and did not otherwise seem
distressed but would not permit any medical staff to go near her nor would she listen
to what they were saying.

Which of the following is the most appropriate approach to the management of this
patient?

1. Give diazepam IV/IM


2. Give haloperidol IV/IM
3. Give lorazepam IV/IM
4. Move to a side room on a ward and observe
5. Call next of kin and ask them to attend to provide reassurance and encourage oral
intake

1. Call next of kin and ask them to attend to provide reassurance and encourage oral
intake Correct answer
Explanation:

The patient is acutely agitated with evidence of delirium and requires de-escalation
management to allow for further investigation.

NICE guidelines suggest "If a person with delirium is distressed or considered a risk to
themselves or others, first use verbal and non-verbal techniques to de-escalate the
situation".

If a person with delirium is distressed or considered a risk to themselves or others and


verbal and non-verbal de-escalation techniques are ineffective or inappropriate,
consider giving short-term (usually for 1 week or less) haloperidol. Start at the lowest
clinically appropriate dose and titrate cautiously according to symptoms.

Benzodiazepines are not recommended in the elderly other than for delirium tremens
as they cause respiratory suppression.

Reference:

2. NICE: Guidelines
3. Sign Guidelines

A 73-year-old woman is admitted to MAU through her GP as she has become


increasingly unwell over the last few days. She is lethargic, has had occasional
episodes of abdominal pain, and is clinically dehydrated. She has a recent history of
metastatic breast cancer with bony involvement, for which she is receiving palliative
treatment. Her results reveal:
Investigation Result Normal range
Sodium 144 mmol/L 134-144 mmol/L
Potassium 3.1 mmol/L 3.5-5.5 mmol/L
Calcium 4.1 mmol/L 2.2-2.6 mmol/L
Which of the following would you expect to find on the ECG?

1. Prolonged PR Interval
2. QT shortening
3. ST-elevation
4. Tall tented T waves
5. Widened QRS complexes

Explanation:
The predominant abnormality here will relate to the markedly elevated calcium, not
the mild hypokalaemia. The QT interval is typically shortened in hypercalcaemia.
Other features in severe hypercalcaemia include J waves and bizarre QRS appearance.

Reference:

1. Some examples of ECG abnormalities in hypercalcaemia

2. QT shortening Correct answer selected

A 73-year-old woman attends A&E feeling unwell. She is lethargic, has had
occasional episodes of abdominal pain and is clinically dehydrated. She has a recent
history of metastatic breast cancer with bony involvement, for which she is receiving
palliative treatment. Her results reveal:
Investigation Result Normal range
Sodium 144 mmol/L (134-144 mmol/L)
Potassium 3.3 mmol/L 3.5-5.5 mmol/L
Urea 10.8 mmol/L 3-8 mmol/L
Creatinine 255 micromol/L 50-100 micromol/L
Calcium 4.1 mmol/L 2.2-2.6 mmol/L
Which of the following is the most appropriate initial treatment for this patient?

1. IM Calcitonin
2. IM Denosumab
3. IV Hydrocortisone
4. IV N Saline – 1l over 2hrs initially
5. IV Pamidronate

Explanation:
The most appropriate initial treatment for this patient is IV rehydration with N Saline
(4-6l over 24hrs). Once rehydration is considered adequate, the patient can then be
treated with IV bisphosphonates such as pamidronate, but caution needs to be
exercised with the acute kidney injury. In this case, the likely cause is metastatic bone
disease, so regular bisphosphonate infusions may be required.

>Reference:

1. Endocrine Society Guidelines on Acute Hypercalcaemia

2. IV N Saline – 1l over 2hrs initially Correct answer selected

A 73-year-old woman is referred to the medical intake with tingling and paraesthesia.
The patient has been aware of deteriorating symptoms for one week. The GP asks
whether this is a stroke. She has a background history of type 2 diabetes, hypertension
and GORD for which she takes omeprazole, metformin, sitagliptin, simvastatin,
Ramipril and amlodipine. On examination, there is little of note.

Investigations reveal:

Investigation Result Normal range


Calcium 2.06 mmol/L
Magnesium 0.3 mmol/L 0.7-1 mmol/L
LFTs Normal
You see on the computer that a 25OHD vitamin D level checked 1 month before by
the GP reveals a level of 18 nmol/l (>50)
Which of the following is the likely cause of this patient’s presentation?

1. Diabetic autonomic neuropathy


2. Metformin
3. Omeprazole
4. Sitagliptin
5. Vitamin D deficiency

1.

Explanation:

This patient has hypocalcaemia associated with a severe hypomagnesaemia. This is


likely due to the omeprazole, which results in increased GI magnesium losses. The
hypomagnesaemia impairs the calcium-sensing on the parathyroid cells; hence, this
results in an induced hypoparathyroidism with hypocalcaemia. It is a good diagnosis
to make but is often not appreciated.

This is not due to vitamin D deficiency as with osteomalacia the 25OHD would be
lower, ALP raised and the patient would likely have a proximal myopathy.
Reference:

2. PPI induced hypoparathyroidism

3. Omeprazole Correct answer selected

1.

Explanation:

This patient has hypocalcaemia associated with a severe hypomagnesaemia. This is


likely due to the omeprazole, which results in increased GI magnesium losses. The
hypomagnesaemia impairs the calcium-sensing on the parathyroid cells; hence, this
results in an induced hypoparathyroidism with hypocalcaemia. It is a good diagnosis
to make but is often not appreciated.
This is not due to vitamin D deficiency as with osteomalacia the 25OHD would be
lower, ALP raised and the patient would likely have a proximal myopathy.

Reference:

2. PPI induced hypoparathyroidism

3. Omeprazole Correct answer selected

You are called to see a 78-year-old male patient on the neurosurgical ward. He was
admitted with a head injury 3 days ago which required only observation.

However, he has become more confused in the last 12 hours and his most recent U+Es
performed this morning reveal:

Investigation Result Normal range


Sodium 125 mmol/L 134-144 mmol/L
Potassium 4.5 mmol/L 3.5-5.5 mmol/L
Urea 6.8 mmol/L 3-8 mmol/L
Urine Osmolality 620 mosmol/L
Urine Sodium 125 mmol/L
Checking through his records, the previous days his U+Es were normal. He had been
allowed to eat and drink normally and had stopped IV fluids 2 days ago.

He was on dexamethasone the dose of which had been reduced to 4mg daily from 8
mg daily over the last day but he was on no other treatment.

On examination, he had Glasgow Coma Scale of 15, though seemed confused with
him being disorientated.

Oxygen saturations were 97% on air and his blood pressure was 110/80 with a pulse of
88 bpm.

Which of the following is the most likely diagnosis?

1. Acute hypoadrenalism
2. Cerebral Salt Wasting
3. Pituitary apoplexy
4. Syndrome of inappropriate ADH secretion
5. Type IV renal Tubular acidosis

Explanation:

The most likely diagnosis in this patient with head injury who then develops acute
symptomatic hyponatraemia and grossly elevated urine sodium is cerebral salt
wasting.

The grossly elevated urine sodium (>100) is highly suggestive as the sodium is usually
less with SIADH. Similarly, with SIADH, there is a dilutional element so urea and
potassium typically reflect this dilution being themselves low normal in contrast to
CSW where there is hypovolaemia and hence higher concentrations.
Hypoadrenalism is an incorrect answer as the patient is already on a high dose of
steroids – dexamethasone 4mg/d.

Reference:

1. Cerebral Salt Wasting Medscape


2. Leonard, J. Garrett, R. E. Salottolo, K. et al. Cerebral salt wasting after traumatic brain
inuury: a review of the litetature, in Scand J Trauma Resusc Emerg Med. 2015; 23:90
doi: 10.1186/s13049-015-0180-5

3. Cerebral Salt Wasting Correct answer selected

A 78-year-old woman with rheumatoid arthritis and type-2 diabetes mellitus presents
with a chesty productive cough. Her medications include metformin 1 g bd,
methotrexate 15 mg weekly, folic acid 5 mg daily and prednisolone 5 mg od. She is
treated at home for a chest infection with a one-week course of amoxicillin and makes
a good recovery.

6 weeks later, she presents with severe lower lumbar back pain and is noted to have a
temperature of 37 oC, a pulse of 100 bpm and blood pressure of 138/78 mmHg. She is
noted to have a left foot drop and a rash over the left shin which is waxy brown-yellow
in colour.

Her investigations reveal:


Investigation Result Normal range
Haemoglobin 12.9 g/dL 13 - 18 g/dL
WBC 15.3 x 109/L 4 - 11 x 109/L
Neutrophils 12.1 x 109/L 2 - 8 x 109/L
Platelets 514 x 109/L 150 - 400 x109/L
CRP 53 mg/L < 10 mg/L
ESR 74 mm/hr < 40 mm/hr
Ig M 2.4 g/L 0.4 - 2.3 g/L
Ig A 7.8 g/L 0.6 - 5.0 g/L
Ig G 12.1 g/L 6.0 - 15.6 g/L
Serum electrophoresis - increased alpha and beta globulins, no paraprotein detected.
Urine electrophoresis - Bence Jones protein not detected.

Lumbar spine X-ray - mild degenerative lumbar spondylotic changes.


Chest X-ray - clear lung fields.

Which of the following is the most likely diagnosis?

1. Infective discitis
2. Lumbar spondylosis
3. Multiple myeloma
4. Osteoporotic fracture
5. Rheumatoid vasculitis

1.

Explanation:

This lady has risk factors for infection - type 2 diabetes mellitus and
immunosuppression with methotrexate for rheumatoid arthritis. Haematogenous
spread of infection after the recent chest infection seems the most probable cause of
the presentation, resulting in seeding of the infection at the lumbar intervertebral disc.
She has nerve root signs at L4/5 with weak tibialis anterior affecting ankle
dorsiflexion. Incidentally, the rash is the description of necrobiosis lipoidica
diabeticorum, and not erythema nodosum or a vasculitis rash. Less than 50% of
infective discitis patients have a fever. Urgent MRI imaging is the first choice here.
CT-guided biopsy can help to identify an organism if blood cultures remain negative,
but only in 50% of cases. Staph. aureus and coagulase-negative staphylococci are the
most common organisms. Tuberculosis should always be considered on the list of
differential diagnoses in the immunosuppressed patient. The waxy brown rash on the
shin is a red herring and, in this patient with diabetes, suggests necrobiosis lipoidica
diabeticorum.

Reference:

2. Lener, S. Hartmann, S. Et al. Management of spinal infection: a review of the


literature, in Acta Neurochir (Wien) 2018; 160(3):487-496. 10.1007/s00701-018-
3467-2

3. Infective discitis Correct answer selected

An Asian man is admitted with a history of weight loss, lethargy, nausea and
vomiting. On examination, he looks dehydrated and pigmented.

His biochemical profile is as shown below:

Investigation Result Normal range


Sodium 124 mmol/L 135-145 mmol/L
Potassium 5.6 mmol/L 3.5-5.5 mmol/L
Urea 30 mmol/L 7-20 mmol/L
Creatinine 104 µmol/L 60-115 µmol/L
9 am cortisol 55 nmol/L
30 min cortisol 220 nmol/L (Post ACTH injection)

The CT image shows the presence of calcification within the adrenal gland. What is
the likely aetiology of his adrenal insufficiency?

1. Adrenal leukodystrophy
2. Adrenal tuberculosis
3. Histoplasmosis
4. Metastatic disease
5. Sarcoidosis

1.

Explanation:
Adrenal tuberculosis remains one of the commonest causes of adrenal insufficiency in
the developing world with about 5% of the patients with disseminated tuberculosis
(Miliary Koch's) developing significant adrenal insufficiency.

A low threshold of suspicion to seek adrenal insufficiency is warranted in patients


with tuberculosis as the symptoms and signs are non-specific and at times are
attributed to underlying condition (Tuberculosis).

Adrenal tuberculosis is reversible with anti-tuberculosis medications if these are given


at an early stage of the disease.

Short Synacthen Test: 30-minute cortisol should be at least 420 nmol/L after being
given synthetic ACTH.

Another way of interpreting the test is that the normal basal cortisol level (between 8
and 10 am) should be at least 180 nmol/L, with a normal response being an increment
of at least 179 nmol/L from the basal value.

Reference:

2. https://www.nbt.nhs.uk/sites/default/files/Short%20Synacthen%20Test.pdf
3. Upadhyay, J, et al. Tuberculosis of the adrenal gland: a case report and review of the
literature of infections of the adrenal gland, International Journal of Endocrinology
(2014). https://doi.org/10.1155/2014/876037

4. Adrenal tuberculosis Correct answer selected

A 76-year-old woman is admitted with a history of being acutely unwell, diarrhoea


and more recent confusion.

She gives no other history of note and is particularly agitated on admission.


Her husband informs you that she has little of note in her history but has become
increasingly agitated and has lost weight over the last month.

Examination reveals that she has confused speech with a GCS of 14 (E4 V4 M6). She
has a BMI of 18 kg/m2, a pulse of 120 bpm irregular, blood pressure of 110/65 mmHg
and a temperature of 38.9oC.

She has an obvious goitre with an audible bruit.

Results reveal:

Investigation Result Normal range

FBC Normal
U+Es Normal
CRP 15 <5
ECG Atrial fibrillation
Which of the following is the most appropriate initial treatment for this patient?

1. IV amoxicillin and metronidazole


2. IV potassium Iodide
3. IV propranolol
4. Oral carbimazole
5. Oral propylthiouracil
1.
Explanation:

This patient appears to be developing thyroid storm as revealed by a temperature,


agitation, confusion, atrial fibrillation and goitre.

We are not provided with the TFTs as these will often take some time to process even
if asked for acutely.

In this case, the IV beta-blockers would be the most appropriate approach as treatment
for the atrial fibrillation and these also prevent the peripheral conversion of T4 to T3.

Either oral propylthiouracil or carbimazole should be given but both will take 2 hours
to have an effect on the production of thyroid hormone and of course neither will
prevent the release of stored hormone.

Thioamides must be given at least one hour prior to the use of iodides which do block
the release of stored hormone.

Also, the patient should receive IV steroids.

Reference:

2. Hyperthyroid crisis thyrotoxic storm.

3. IV propranolol Correct answer selected

A 77-year-old woman with acute symptomatic hyponatraemia likely due to SIADH


and associated with drowsiness is considered to be appropriate for immediate
treatment with hypertonic saline.
Her serum sodium is 115 mmol/l.

What is the target sodium to which you would treat this patient over the next 1hr?

1. 116 mmol/l
2. 120 mmol/l
3. 125 mmol/l
4. 130 mmol/l
5. 135 mmol/l

1.

Explanation:
A decision has been taken to treat this patient with hypertonic saline as she has acute
symptomatic hyponatraemia which reflects underlying cerebral oedema.

In this context, treatment with 150mls 3% hypertonic saline given over three hours can
be lifesaving.

The target sodium by which one should elevate the sodium is 5 mmol/L over the first
hour.

The increase in sodium should be then limited to 10mmol/L over the next 24hrs based
on a clinical improvement.

Reference:

2. European Journal of Endocrinology (2014) Clinical practice guideline on diagnosis


and treatment of hyponatraemia

3. 120 mmol/l Correct answer selected

A 71-year-old woman was admitted with a two day history of increasing fatigue and
decreasing level of consciousness.
She had otherwise been well. She had type 2 diabetes for which she was being treated
with metformin 500 mg bd, empagliflozin 10 mg daily, ramipril 10mg daily and
atorvastatin 10 mg daily.

On examination she had a temperature of 38C, a blood pressure of 100/70 mmHg and
a pulse of 100 bpm with a respiratory rate of 28/min. Her Glasgow Coma Scale score
was 12.

There was nil of note on abdominal, respiratory, neurological or cardiovascular


examination. Results revealed:

Investigation Result Normal range


Haemoglobin 12 g/dl 12-16 g/dl
White cell count 14.5 x 109/L 3.5-10 x 109/L
Platelet 210 x 109/L 150-400 x 109/L
Sodium 140 mmol/L 134-144 mmol/L
Potassium 5 mmol/L 3.5-5 mmol/L
Creatinine 210 micromol/L 50-100 micromol/L
Urea 8.3 mmol/L 3-8 mmol/L
Glucose 14.5 mmol/l 3.5-5.5 mmol/L
pH 7 7.35-7.45
Bicarbonate 7 mmol/L 22-28 mmol/L
pCO2 1.9 kPa 4.5-6
pO2 14.8 kPa 10.5-13
Lactate 2.1 mmol/L 0.5-1 mmol/L
Ketones (on meter) Hi
Which of the following is the likely cause of her presentation?

1. Diabetic ketoacidosis
2. Drug-induced ketoacidosis
3. Drug-induced lactic acidosis
4. Sepsis with fasting ketosis
5. Sepsis with mixed fasting ketosis/lactic acidosis
Explanation:

This patient has developed severe metabolic acidosis as suggested by the low pH and
low bicarbonate with some respiratory compensation. The ketones are high which is
highly relevant here, particularly as the lactate is only modestly high and does not
suggest lactic acidosis. A good going lactic acidosis would be above 5. Also, the
glucose is only modestly elevated. This fits the picture of drug-induced ketoacidosis
due to the SGLT-2 inhibitor, empagliflozin. Whilst a very effective diabetic
medication, all SGLT-2 inhibitors have been rarely implicated in euglycaemic or
modestly hyperglycaemic ketoacidosis, an effect of their mechanism of action relaying
on urinary glucose excretion. Often illness, fasting and vomiting exacerbate the
ketosis.

Reference:

1. Tucker, M. E., 2017. SGLT2 Inhibitors Double the Risk for Diabetic Ketoacidosis.
Medscape.
2. Rajeev, S. P. Wilding, J. P. SGLT2 inhibition and ketoacidosis - should we be
concerned. The British Joirnal of Diabetes, vo.15.4 (2015).

3. Drug-induced ketoacidosis Correct answer selected


An 81-year-old man is admitted acutely with a week history of feeling increasingly
unwell and weak. He has a history of type 2 diabetes for which he takes metformin 1g
bd and gliclazide 80mg bd.

His last HbA1c was 10.1%. He also takes Ramipril and simvastatin.

On examination, he appears dehydrated and unwell. His temperature is 37oC, blood


pressure 116/70 mmHg and a pulse of 102 bpm.

Oxygen saturation is 98% on air and he has a respiratory rate of 22/min.

Blood results reveal:

Investigation Result Normal range


Sodium 155 mmol/L 134-144 mmol/L
Potassium 4.2 mmol/L 3.5-5.5 mmol/L
Urea 20.5 mmol/L 3-8 mmol/L
Glucose 37.2 mmol/L 3.5-5.5 mmol/L
pH 7.35 7.35-7.45
Bicarbonate 18 mmol/L 22-28 mmol/L
Ketones 1.8 mmol/L <1.5 mmol/L
Which of the following is the likely diagnosis?

1. Acute kidney injury associated with ACE inhibitor


2. Diabetic Ketoacidosis
3. Hyperosmolar Hyperglycaemic State
4. Severe dehydration with acute kidney injury
5. Type B lactic acidosis associated with metformin
1.

Explanation:

The most likely diagnosis is HHS as plasma osmolality 376 mosmol/L with glucose
above 33, bicarbonate above 15 and a pH >7.3. A trace of ketones may be noted (<3).

Plasma osmolality is calculated by: 2(Na + K) + glucose + urea. Normal value is 275-
295 mmol/kg.

Reference:

2. Hyperosmolar hyperglycaemic state

3. Hyperosmolar Hyperglycaemic State Correct answer selected


A 72-year-old woman is admitted by her GP with a history of fatigue and weakness
which had deteriorated over the last month.

The GP had checked her bloods and discovered haemoglobin of 6.7 g/dl (12-16) with
an MCV of 59 fl.

She had a past history of type 2 diabetes for which she takes metformin 500 mg bd and
her most recent HbA1c checked 2 weeks ago was 68 mmol/mol (<48).

She also has a history of previous DVT and atrial fibrillation with arterial embolism
for which she had required a left below-knee amputation. Consequently, she remains
on lifelong warfarin.

She takes bisoprolol 2.5 mg/d, ramipril 2.5 mg daily, omeprazole 40mg daily, ferrous
fumarate 210mg bd and simvastatin 40mg daily.

She describes feeling weak and lethargic over the last month, which has culminated in
her seeing her GP.

She denies any abdominal pains and has been unaware of any melaena or PR blood
loss. She denies any weight loss.

There was some story about iron deficiency a year or so ago for which she had been
prescribed iron tablets. There was nothing on the clinical portal about this.

On examination, she is comfortable but appears pale. Her BMI is 32.2 kg/m2. Her
pulse is 70 bpm irregular and blood pressure is 110/70 mmHg. There is nothing to find
on examination other than she had a below-knee amputation and PR examination is
normal.
Other than the above all other results are normal including U+Es and LFTs. Ferritin
was 10 microg/l (30-100) and INR was 2.5. Chest x-ray is normal.

Which of the following is the most appropriate management strategy for this patient?

1. Give IV iron and arrange outpatient upper and lower GI studies


2. Increase dose of iron, stop warfarin and discharge with OPD upper and lower GI
studies
3. Transfuse, continue warfarin and arrange OPD upper and lower GI studies
4. Transfuse, stop her warfarin and arrange inpatient upper and lower GI studies
5. Transfuse, stop her warfarin and arrange outpatient endoscopy

Explanation:

This lady has an iron deficiency anaemia which appears to be long-standing as she is
haemodynamically stable and has no overt source of bleeding. The likely source is
therefore diverticular disease or malignancy, which seems less likely in view of the
normal weight and lack of GI symptoms. The issue is further complicated by the fact
that she needs to be anticoagulated which is required lifelong due to the embolism and
previous DVT. Therefore, on balance warfarin should continue, the patient should be
transfused for symptomatic improvement and the dose of iron can be increased. The
patient can then be discharged with an assessment of Hb to ensure stability and GI
studies organised as an outpatient – CT colonogram initially and endoscopy.

Reference:

1. Guidelines for the management of iron deficiency anaemia

2. Transfuse, continue warfarin and arrange OPD upper and lower GI studies Correct
answer
A 79-year-old man with a previous ischaemic stroke attends A&E. He is percutaneous endoscopic
gastrostomy (PEG) fed and his PEG tube has “fallen out” 45 minutes ago after it got caught in his
clothing, whilst using the toilet. Unfortunately, the PEG tube fell into the toilet pan but Mr Jones has
brought it with him “in case it can be sterilised”. He is otherwise well.

What is the best management plan?

1. Arrange for CCSD to sterilize the old PEG and re-insert it


2. Arrange for the gastroenterologists to insert a new PEG through a new tract
3. Insert a new PEG tube using the kit from endoscopy
4. Reassure the patient that the gastroenterology team will be able to use the same tract in the morning
and not to put anything down the PEG tract
5. Insert a foley catheter into the tract to ensure the tract stays open
Back
A 79-year-old man with a previous ischaemic stroke attends A&E. He is percutaneous endoscopic
gastrostomy (PEG) fed and his PEG tube has “fallen out” 45 minutes ago after it got caught in his
clothing, whilst using the toilet.

Unfortunately, the PEG tube fell into the toilet pan but Mr Jones has brought it with him “in case it
can be sterilised”. He is otherwise well.

What is the best management plan?

1. Arrange for CCSD to sterilize the old PEG and re-insert it


2. Arrange for the gastroenterologists to insert a new PEG through a new tract
3. Insert a new PEG tube using the kit from endoscopy
4. Reassure the patient that the gastroenterology team will be able to use the same tract in the morning
and not to put anything down the PEG tract
5. Insert a foley catheter into the tract to ensure the tract stays open
Explanation:

PEG tubes are fitted endoscopically and cannot be re-inserted (especially after toilet exposure!). A
balloon gastrostomy is a viable alternative not listed here. A foley catheter prevents the peg tract
from closing (which can happen within 4 hours) and allows the safe re-insertion of a balloon
gastrostomy the next day. Using the same tract is desirable as it avoids an invasive procedure and the
risks of forming a new tract.

Reference:

1. Prosser, B. Common issues in PEG tubes—what every fellow should know, 64.6, P970-972,
doi:10.1016/j.gie.2006.07.042

2. Insert a foley catheter into the tract to ensure the tract stays open Correct answer selected

A 78-year-old man was admitted with haematemesis. He had gone out the evening
before and had drunk too much alcohol at a wedding party.
He vomited in the early hours and then on the last couple of occasions vomited frank
blood.

He vomited blood on two occasions, the last of which was 2 hours ago.

His wife brought the pillow cloth on which he vomited, which shows dark blood
staining.

He had nothing of note in his past medical history other than type 2 diabetes and
hypertension, for which he takes metformin, ramipril, simvastatin and clopidogrel.

On examination, he was subdued with a pulse of 68 bpm and a blood pressure of


116/60 mmHg.

His haemoglobin concentration was 116 mg/L (120-160). Coagulation and LFTs were
normal.

As part of the management plan, clopidogrel was stopped.

Which of the following is the most appropriate approach to the management of this
patient?

1. Admit and arrange endoscopy (within 4 hrs)


2. Admit and arrange endoscopy (within 24 hrs)
3. Admit and monitor without necessarily arranging endoscopy unless clinical condition
deteriorates
4. Commence oral PPI, discharge if clinical observations stable over view to outpatient
endoscopy (within 1 week)
5. Discharge in about 4 hours if no further haematemesis has occurred and observations
stable with no requirement for future endoscopy
Explanation:

In this man’s case, he has had a witnessed haematemesis and he is over 60 giving a
Rockall score of 1. The most likely diagnosis is a Mallory-Weiss tear from the
description but even so, the guidelines suggest that this man should be admitted and,
where possible have an endoscopy performed within 24 hrs. There is no evidence that
commencing a PPI before endoscopy would have any prognostic benefit.

Reference:

NICE: Guidelines
Stanley, A. Management of acute upper gastrointestinal bleeding BMJ 2019; 364 doi:
https://doi.org/10.1136/bmj.l536.

Admit and arrange endoscopy (within 24 hrs) Correct answer


A 76-year-old man undergoes an upper GI endoscopy for dyspepsia. The findings are unremarkable
and he is discharged home. However, he attends A&E 3hrs later that evening with a 1-hour history of
acute, severe central chest pain associated with feeling hot and sweaty.

On examination, he appears in pain with a pulse of 98 bpm, a blood pressure of 148/88 mmHg and a
temperature of 38.3oC.

Which is the most appropriate investigation from the following list to confirm this gentleman’s
diagnosis?

1. Barium swallow
2. CXR
3. ECG
4. Gastrograffin swallow
5. Troponin
Explanation:

The likely diagnosis here is iatrogenic oesophageal perforation. Although very rare in diagnostic
OGD, it can occur and the fever here suggests that this is unlikely to be cardiac in origin, and as
such, the most likely test to confirm the diagnosis is a contrast swallow. Gastrografin should be used
as it is water-soluble and will not cause mediastinitis, unlike barium. Ideally, a plain film swallow
should be performed prior to CT as the radiation dose is less.

Reference:

1. Søreide, J. A. Viste, A. Esophageal perforation: diagnostic work-up and clinical decision-making in


the first 24 hours. Scand J Trauma Resusc Emerg Med. 2011; 19: 66. Published online 2011 Oct 30.
doi: 10.1186/1757-7241-19-66

2. Gastrograffin swallow Correct answer selected


A 81-year-old lady presents with a septic right knee joint. Joint aspiration is sent for culture and
Streptococcus bovis is confirmed. She is treated with intravenous antibiotics for 2 weeks and a
further 4 weeks of oral antimicrobial therapy.

Which investigation should be carried out next?

1. Arthroscopy
2. Chest X Ray
3. Colonoscopy
4. Lumbar spine X rays
5. Synovial biopsy
Explanation:

Steptococcus bovis is an organism that is associated with colonic malignancy. Genomic


reassociations have shown that S. bovis biotype 1 is associated with malignancy in 100% of cases
and biotype 2 only in 25%. In this case, S. bovis has seeded into the joint to produce a septic arthritis.
As it is uncommon to isolate S. bovis in a joint, further follow up with colonoscopy is advised.

References:

1. Garcia-Porrua et al. Septic arthritis due to Streptococcus bovis as presenting sign of ‘silent’ colon
carcinoma Rheumatology (2000) 39 (3): 338-339. Isolated septic arthritis due to Streptococcus bovis.
Grant et al. Clinical Infectious Diseases 1997;24:102.

2. Colonoscopy Correct answer selected


A 76-year-old man is admitted with a 3-day history of left leg pain and fever. He has an 8-year
history of type 2 diabetes for which he takes metformin and gliclazide together with ramipril and
simvastatin.

On examination his BMI is 32.3 kg/m2, his temperature is 38.5oC, pulse 90 bpm and his blood
pressure 120/80 mmHg. His left leg is swollen to mid-calf, the skin is red and inflamed with a clear
demarcation just below the knee. He has some onycholysis of the left toenails with some maceration
between the toes. His peripheral pulses are all palpable.

Results show:

Investigation Result Normal range


Sodium 136 mmol/L 134-144 mmol/L
Potassium 4.2 mmol/L 3.5-5.5 mmol/L
Urea 12.1 mmol/L 3-8 mmol/L
Creatinine 128 micromol/L 50-100 micromol/L
Haemoglobin 13.3 g/L 120-160 g/L
White cell Count 12.8 x109/L 3-10 x109/L
Platelets 158x109/L 140-450 x109/L
C-Reactive Protein 135 mg/dl <5
Glucose 12.3 mmol/L
HbA1c 7.8% <6
Which of the following would be the most appropriate treatment for this patient?

1. IV flucloxacillin
2. Oral amoxicillin
3. Oral ciprofloxacin
4. Oral flucloxacillin
5. IV Vancomycin

Explanation:
This patient has a clinical class II to class III cellulitis with systemic upset associated with a
comorbidity. The vast majority of cases of cellulitis are due to beta haemolytic strep or S Aureus.
Therefore, Flucloxacillin is the treatment of choice in this setting and recommended in guidelines as
IV treatment in Class II-III patients. In subjects allergic to beta-lactams, clarithromycin/clindamycin
can be used.

Reference:

1. Guidance on management of cellulitis in adults

2. IV flucloxacillin Correct answer

A 73-year-old man is admitted to the MAU by his GP with a 5-day history of cough and feeling
unwell. He has otherwise been well with no other history of note. His cough is productive of
greenish sputum and he has become increasingly breathless. He is a non-smoker. He informs you
that he has a penicillin allergy. Otherwise, he does not take any medication.

On examination, he is orientated, a pulse of 82 bpm regular, blood pressure 122/88 mmHg,


respiratory rate 24 breaths/min and oxygen saturations on air of 95%. His temperature is 38.5oC.
Auscultation of the heart is normal and examination of the chest reveals some crackles at the right
base.

His results reveal:

Investigation Result Normal range


Sodium 136 mmol/L 134-144 mmol/L
Potassium 4.2 mmol/L 3.5-5.5 mmol/L
Urea 6.8 mmol/L 3-8 mmol/L
Creatinine 98 micromol/L 50-100 micromol/L
Haemoglobin 13.3 g/L 120-160 g/L
White cell Count 12.8 x 109/L 3-10 x 109/L
Platelets 158 x 109/L 140-450 x 109/L
C-Reactive Protein 55 mg/dl <5
The chest x-ray reveals some haziness at the right base.

Which of the following is the most appropriate treatment for this patient?

1. IV cefotaxime
2. IV clarithromycin
3. Oral amoxicillin
4. Oral doxycycline
5. Oral levofloxacin

Explanation:

This patient has community-acquired pneumonia but is allergic to penicillin and therefore, according
to guidance, could be treated with oral non-penicillin alternatives such as doxycycline or
clarithromycin.

Reference:
1. BTS: Pneumonia guidelines

2. Oral doxycycline Correct answer selected

A 62-year-old man is admitted to MAU by his GP with a 5-day history of productive cough and
feeling unwell. He has no other medical history of note. His cough is productive of greenish sputum
and he has become increasingly breathless. He is an ex-smoker having stopped 10 years ago. He
takes no medication.

On examination, he appears unwell, is confused and combative. Observations: pulse rate 90 bpm
regular, blood pressure 124/68 mmHg and oxygen saturations on air of 94%. His temperature is
38.8oC and his respiratory rate is 26/minute. Auscultation of the heart is normal and examination of
the chest reveals some crackles at the right base.

His results reveal:

Investigation Result Normal range


Sodium 136 mmol/L 134-144 mmol/L
Potassium 4.2 mmol/L 3.5-5.5 mmol/L
Urea 8.6 mmol/L 3-8 mmol/L
Creatinine 110 micromol/L 50-100 micromol/L
Haemoglobin 13.5 g/L 120-160 g/L
White cell Count 14.8 x109/L 3-10 x109/L
Platelets 158 x109/L 140-450 x 109/L
C-Reactive Protein 128 mg/dl <5 mg/dl
The chest x-ray reveals haziness at the right base.

What is the CURB-65 score in this patient?

1. 0
2. 1
3. 2
4. 3
5. 4

1. 2 Correct answer selected


2.

Explanation:

The CURB-65 score is a score used in community-acquired pneumonia with prognostic value. He
scores 2 based on confusion and elevated urea. His CURB65 score can be calculated as follows:

Confusion 1
Urea >7 1
Respiratory rate >30 0
BP Systolic <90, Diastolic <60 0
65 years of age or above 0
Reference:

3. BTS Guidelines: pneumonia in adults


A 66-year-old lady with a 20-year history of seropositive erosive rheumatoid arthritis presents with a
two-week history of deteriorating weakness and neck pain. She has noticed neck pain and a sensation
that her head is slumping forwards. Her arms and legs feel generally weak and she has complained of
urinary hesitancy and constipation in the past week.

Clinical examination reveals painful restricted neck movements. Tone is increased in the upper and
lower limbs. There is proximal weakness graded 4/5 on shoulder abduction and hip flexion
bilaterally. Biceps and knee jerk reflexes are bilaterally brisk.

What is the most appropriate investigation?

1. Cerebrospinal Fluid analysis


2. Cervical spine X ray with flexion / extension views, and MRI cervical spine
3. MRI Brain
4. MRI lumbar spine
5. Nerve conduction studies / electromyography
Explanation:

Atlanto axial instability affects between 12 and 33% of patients with rheumatoid arthritis. Assessing
the atlanto-axial joint requires cervical spine radiographs in flexion and extension in the first
instance, followed by magnetic resonance imaging of the cervical portion of the spinal cord. An
urgent spinal and neurosurgical opinion should be sought.

Although a primary brain disorder such as sagittal sinus thrombosis, space-occupying lesions or
demyelination could be considered in the differential diagnosis, in a rheumatoid arthritis patient with
neck pain, tetraplegia and upper motor neurone signs, atlantoaxial instability must be considered
first. Imaging of the brain and lumbar puncture is, therefore, less relevant. Nerve conduction
studies/electromyography would not be helpful, and imaging the lumbar spine would miss a higher
central spinal cord lesion.

Reference:

1. Zikou AK, Alamanos Y, Argyropoulou MI, et al. Radiological cervical spine involvement in patients
with rheumatoid arthritis: a cross sectional study. (2005) J Rheumatol 32: 801-06
2. Naranjo A, Carmona L, Gavrila D, et al. Prevalence and associated factors of anterior atlantoaxial
luxation in a nation-wide sample of rheumatoid arthritis patients. (2004) Clin Exp Rheumatol 22:
427-32
3. Kauppi M, Hakala M. Prevalence of cervical spine subluxations and dislocations in a community-
based rheumatoid arthritis population. (1994) Scand J Rheumatol 23:133-6
4. Karhu JO, Parkkola RK, Koskinen SK. (2005) Evaluation of flexion/extension of the upper cervical
spine in patients with rheumatoid arthritis: an MRI study with a dedicated positioning device
compared to conventional radiographs. Acta Radiol 46(1): 55-66
5. Hung S-C, Wu H-M, Guo W-Y. Revisiting Anterior Atlantoaxial Subluxation with overlooked
information on MR images. (2010) Am J Neurorad 31: 838-43
6. Krauss WE, Bledsoe JM, Clarke MJ, Nottmeier EW, Pichelmann MA. Rheumatoid arthritis of the
craniovertebral junction Neurosurgery (2010) 66 (3 Suppl): 83-95

7. Cervical spine X ray with flexion / extension views, and MRI cervical spine Correct answer selected
8.

A 74-year-old lady presents with a history of gradual weakness in her legs to an extent that she can
no longer walk.

She complains of back pains and numbness down both legs leading up to this presentation.

There is no history of urinary or faecal incontinence.

She has no significant past medical history except for previous carpal tunnel syndrome release and
cervical spondylosis.

On examination, you find that there is full power in her upper limbs with intact reflexes and
sensation.

The legs are weak with absent knee and ankle jerks. There is a loss of sensation to pinprick at the big
toes of both feet, soles, and the back of both legs.

There is perianal numbness and reduced sphincter tone.

What is the next appropriate investigation?

1. Isotope bone scan


2. MRI Cervical spine
3. MRI Lumbar spine
4. Nerve conduction studies
5. Xray lumbar spine
Explanation:

This lady has the classic features of a cauda equina syndrome:

1. Back pain
2. Radicular pain/numbness down the legs
3. Areflexic paralysis of the legs
4. Sensory loss in a root distribution
5. Reduced sphincter power

She has bilateral lower leg flaccid weakness with absent reflexes and sensory disturbance with loss
of perianal sensation. Usually these lesions present at an earlier stage with conus medullaris lesions:
urinary retention, constipation, back pain, sacral sensory loss, with or without leg weakness, &
saddle anaesthesia. Previous cervical spondylosis and carpal tunnel syndrome release are just
distractors and bear no clinical relevance to the current symptoms.

Reference:

6. Oxford Handbook of Clinical Medicine, 5th Edition [book].


7. BMJ Clinical review

8. MRI Lumbar spine Correct answer selected


Explanation:

The most likely diagnosis, in this case, is pseudogout presenting with a pseudorheumatoid pattern of
chronic, recurrent swelling affecting primarily large joints. It is often associated with inflammatory
symptoms such as stiffness, pyrexia, and malaise. A monoarticular presentation is typical.

However, any patient presenting with a warm swollen joint must have the joint aspirated and sent for
urgent gram stain, culture and microscopy to exclude a septic arthritis. It is likely the microscopy
will provide the diagnosis by showing brick-shaped, weakly positively birefringent crystals but it
should be remembered that crystals may still be seen in a septic joint.

Reference:

li>Oxford Desk Reference Rheumatology. Oxford University Press 2009. p388-9.


1. V. Weston and G. Coakley. Guideline for the management of the hot swollen joint in adults with a particular focus
on septic arthritis. Journal of Antimicrobial Chemotherapy (2006) 58, 492-493.
2. NICE: Gout assessment

3. Aspirate and send fluid for crystals and MC&S Correct answer selected
A 74-year-old woman presents to clinic with a number of leg ulcers.

She has longstanding rheumatoid arthritis and had stopped her Methotrexate therapy over one year
ago.

Her weight has been stable. She is not keen on further therapy but is frustrated that she has had 6
months of troublesome leg ulcers.

Abdominal examination reveals an 8cm mass in the left upper quadrant that is dull to percuss and
descends on inspiration.

Her investigations are below:

Investigation Result Normal range


Haemoglobin 10.9 g/dL 13 - 18 g/dL
WBC 2.4 x109/L 4 -11 x109/L
Neutrophils 0.7 x109/L >2 x109/L
Lymphocytes 1.5 x109/L >1 x109/L
Platelets 85 x109/L 150 - 400 x 109/L

What is the most likely explanation for her symptoms?

1. Chronic lymphocytic leukaemia


2. Felty’s syndrome
3. Methotrexate induced pancytopenia
4. Myelofibrosis
5. Occult Malignancy
Explanation:

This longstanding patient with rheumatoid arthritis has the triad of Neutropenia, Splenomegaly and
leg ulcers. This is Felty’s syndrome. Patients may also have other haematological findings including
thrombocytopenia, anaemia and pancytopenia. Methotrexate induced pancytopenia would not occur
a year after stopping therapy.

Felty’s syndrome ties in all the features of this presentation, and the other diagnoses would not
account for all these features.

Reference:

1. Oxford desk reference in Rheumatology. Watts et al. 2009 [book].


2. McGraw H. Current diagnosis and treatment. Rheumatology. Imboden et al. 2007 [book].
3. NCBI: Felty Syndrome

4. Felty’s syndrome Correct answer selected


A 76-year-old man is admitted on the acute medical take via the GP, following repeated episodes of
falling. He was living at home with a four times a day package of care, and family assistance, but had
become increasingly dependent on his family as his mobility had significantly deteriorated over the
past 6 months.

On admission the patient is complaining of hallucinations and is concerned that the nursing staff are
“out to get him”. He is not aggressive but his family states that these hallucinations have been
worsening and at times he can be very disorientated and confused.

Physical examination is unremarkable apart from some minor cogwheeling rigidity in his hands, and
a shuffling gait with reduced arm swing.

He showed evidence of mild cognitive deficit on testing and was unable to draw a clock.

Bloods were in the normal range, ECG showed normal sinus rhythm and CT head showed mild small
vessel disease.

He is finding the hallucinations to be distressing.

What is the most appropriate treatment for this?

1. Haloperidol
2. Lorazepam
3. Pramiprexole
4. Rivastigmine
5. Trazadone

1. Rivastigmine Correct answer


Explanation:

The patient has features of Parkinsonism with worsening mobility, frequent falls, fluctuating levels
of lucidity and hallucinations suggesting a diagnosis of Lewy Body Dementia.

Anticholinergic medication and typical antipsychotics medication are contraindicated in Lewy Body
Dementia and are associated with increased mortality.

Reference:

2. Boot, B. P. McDade, E. M. McGinnis, S. M. Boeve, B. F. Treatment of Dementia with Lewy Bodies.


Curr Treat Options Neurol. 2013 Dec; 15(6): 738–764. doi: 10.1007/s11940-013-0261-6

A 79-year-old woman is admitted via the GP with left-sided weakness that had developed over the
last two to three days.

Three weeks ago she had fallen on compacted sand on the beach whilst playing with her
grandchildren and had hit her head.
She did not lose consciousness but was 'a bit dazed' and had a headache afterwards. She was
otherwise fit and well and took no medication.

On examination, GCS is 15, she had mild left-sided weakness of arm and leg 4/5 with an equivocal
plantar response.

CT head scan revealed a right-sided chronic subdural haematoma with some compression of the right
ventricle but no midline shift.

Which of the following is the most appropriate management plan for this patient?

1. Admit and refer to neurosurgery for evacuation


2. Admit and treat with dexamethasone
3. Admit and treat with mannitol
4. Discharge and treat with outpatient dexamethasone
5. Discharge with reassurance that it will resolve and arrange further CT in 3 months

Explanation:

This patient has a chronic subdural haematoma and associated hemiparesis, which has developed
over the course of 3 weeks. The patient should be referred to neurosurgery and treated with burr hole
evacuation. In chronic subdural with no midline shift and symptoms such as headache then
observation with re-scanning can be undertaken. However, this patient has developed a significant
neurological deficit despite the lack of midline shift and therefore surgery is required.

Reference:

1. Management of chronic subdural haematoma


2. Yadav, Y. R. Parihar, V. Namdev, H. Bajaj, J. Chronic subdural hematoma, in Asian J Neurosurg.
2016, 11(4): 330-342. doi: 10.4103/1793-5482.145102

3. Admit and refer to neurosurgery for evacuation Correct answer selected

A 71-year-old male presents to his GP with a 3-month history of weight loss, cough and malaise. He
has a 50-pack year smoking history. His GP organises a number of investigations including a chest
X-ray that shows a left upper lobe mass. Some key blood test results are as follows:

Investigation Result Normal range


Haemoglobin 14.9 g/dl 12-16 g/dl
White Blood Count 6.3 x 109/L 4-10 x 109/L
Corrected Calcium 2.58 mmol/L 2.2-2.6 mmol/L
Potassium 3.1 mmol/L 3.5-5 mmol/L
Sodium 125 mmol/L 134-144 mmol/L
Based on the investigations, what is the most likely diagnosis?
1. Adenocarcinoma of the lung
2. Large cell lung carcinoma
3. Small cell lung cancer
4. Squamous cell lung cancer
5. Tuberculosis

Explanation:

This patient has symptoms of underlying lung cancer, supported by the mass on X-ray.

His blood picture demonstrates a neuroendocrine syndrome, namely syndrome of inappropriate ADH
secretion (SIADH). SIADH is most commonly associated with small cell lung cancer.

Squamous cell lung cancer is more commonly associated with hypercalcaemia, whilst
adenocarcinoma and large cell carcinoma are not as commonly associated with the neuroendocrine
syndromes.
Although TB can present with the symptoms described, given the history and key findings, it is
important to exclude lung cancer first in this patient.

Reference:

1. Iyer, P. Ibrahim, M. Siddqui, W. Dirweesh, A. Syndrome of inappropriate secretion of anti-diuretic


hormone (SIADH) as an initial presenting sign of non small cell lung cancer-case report and
literature review, in Respir Med Case Rep. 2017; 22: 164–167. Published online 2017 Aug 11. doi:
10.1016/j.rmcr.2017.08.004

2. Small cell lung cancer Correct answer selected

This 76-year-old man presents to the unit with a one-week history of a widespread deteriorating skin
rash. His skin has become increasingly inflamed and red.

He has had little sleep over the last couple of days as he feels unwell and has been itching intensely.
He has no particular history of note other than some mild eczema of the flexural areas. He has been
taking Lisinopril, Amlodipine, Aspirin and Simvastatin for many years for hypertension.

1. Chronic angioedema
2. Erythroderma
3. Lichen planus
4. Norwegian Scabies
5. Red man syndrome

Explanation:
This elderly man with a history of eczema has a widespread erythematous rash involving the
majority of his skin. The diagnosis is therefore erythroderma and this may be due to a number of
disorders including eczema, psoriasis and mycosis fungoides. In this patient’s case, the likely
diagnosis is erythrodermic eczema and the patient should be treated with cooling baths with
emollients and topical steroids. Where infection is also considered, antibiotics such as Flucloxacillin
should be employed.

Reference:

1. PCDS: Erythroderma

2. Erythroderma Correct answer selected

You are called to see a 70-year-old woman in the dermatology day unit. She had a non-healing ulcer
above the right medial malleolus of 4 months’ duration. On further inspection, the ulcer is shallow
but has a significant exudate which is soaking through the current dressings applied by the GP
practice nurse.

You also note evidence of venous flaring and haemosiderin staining in the gaiter region of both legs.

The ABPI is measured at 1.0.


Which of the following would be the most appropriate treatment?

1. Alginate dressing with compression bandaging


2. Hydrocolloid dressing with compression bandaging
3. Larval therapy (sterile maggots)
4. Paste bandages
5. Topical metronidazole with compression bandaging

1.

Explanation:

This is a venous ulcer. Clues include the site (gaiter area), venous flaring, haemosiderin staining and
the ABPI of 1.0. An alginate dressing would be an appropriate choice in this case as there is a large
amount of exudate which the alginate, a seaweed derivative, would absorb. Compression is essential
to expedite healing in venous ulcers.
Hydrocolloid dressings seal the moisture and are useful for dry wounds. Hence in this wound with
excessive exudate, a hydrocolloid dressing would not be helpful. Topical metronidazole would be
helpful to control the foul odour caused by anaerobes.

Reference:

2. Wounds UK: How to choose the appropriate dressing for each wound type
3. Fonder MA, Lazarus GS, Cowan DA, et al. Treating the chronic wound: A practical approach to the
care of nonhealing wounds and wound care dressings. J Am Acad Dermatol. 2008 Feb;58(2):185-
206.

4. Alginate dressing with compression bandaging Correct answer

A 72-year-old woman attends the leg ulcer clinic for a review. She was known to have chronic
venous hypertension and leg ulcers.

Her BMI is 33. She had been visiting her GP surgery for dressings of her leg ulcers for a few months
and was making slow progress.

She had previously been recommended 4-layer compression bandaging but has not been very
compliant wearing it due to discomfort, only being able to tolerate a single layer tubigrip.

Her peripheral pulses are well palpable and her ABPI is 1.0 in both legs.

Which of the following facts is true about ulcer healing?

1. Chronic ulcer fluids have mediators that promote ulcer healing


2. Occluded ulcers have better scarring
3. Occluding the ulcer will slow down ulcer healing by 40 percent
4. There is clear evidence of benefit in using silver dressings in chronic venous ulcers
5. There is clear evidence of faster healing of venous ulcers with honey dressings

1.

Explanation:

Moist wounds heal faster than the ones that dry out. Occluding a wound makes it heal 40 per cent
faster than non-occluded wounds. Chronic ulcer fluids have increased inflammatory mediators that
hinder the healing process by inhibiting the proliferation of fibroblasts. Occluded ulcers have lesser
inflammatory mediators and hence better scarring. There is no evidence for either silver or honey
dressings in faster healing of venous ulcers.

Reference:

2. Vogt PM, Andree C, Breuing K, et al. Dry, moist, and wet skin wound repair. Ann Plast Surg 1995;
34:493-9
3. McCarty SM & Percival SL (2013) Advances in Wound Care: Proteases and Delayed Wound
Healing
4. Carter MJ, Tingley-Kelley K, Warriner RA 3rd. Silver treatments and silver-impregnated dressings
for the healing of leg wounds and ulcers: a systematic review and meta-analysis. J Am Acad
Dermatol 2010; 63:668
5. Jull A, Walker N, Parag V, et al. Randomized clinical trial of honey-impregnated dressings for
venous leg ulcers. Br J Surg 2008; 95:175

6. Occluded ulcers have better scarring Correct answer selected

An 86-year-old lady presented with a chronic superficial ulcer on the right medial malleolus. She had
surrounding red-brown discolouration of the leg and it had recently become painful, with malodorous
green exudate.

Which of the following is the most appropriate treatment?

1. Collagen dressings
2. Swab wound, foam dressings
3. Swab wound, systemic antibiotics
4. Swab wound, tegaderm dressings
5. Swab wound, topical antibiotics
1. Swab wound, systemic antibiotics Correct answer selected

Explanation:

This is an infected chronic venous ulcer. The recent onset of pain indicates infection and will need
treatment with systemic antibiotics. Topical antibiotics are best avoided for fear of inducing contact
allergy and antibiotic resistance.

Reference:

2. Sign: (2010) Management of chronic venous leg ulcers, a national clinical guideline
Explanation:

This lady has crusted scabies, a highly contagious form of scabies in which the patient can have a
substantially higher mite load. The risk factors for crusted scabies are: Immunosuppression,
malnutrition, and modified host response as in dementia, cognitive impairment. Crusted scabies is
often observed in older, debilitated or immunocompromised patients who have reduced ability to
control the proliferation of the mites and in patients with cognitive impairment who have lack of
awareness of symptoms and as a result, do not seek advice. Itching is usually not a significant
symptom in crusted scabies.

History of oral prednisone use in scabies increases the risk of transformation to crusted scabies.
Another challenge with the use of corticosteroids for a patient with a scabies infection is the potential
to mask the typical pruritus and inflammation, producing a rare, and difficult to recognise, entity
known as scabies incognito. This patient also had eosinophilia, which was observed in over half of
patients with crusted scabies in one study.

The presence of palms and soles involvement with accentuation in the palmar creases, finger/toe web
spaces along with flexural involvement should raise suspicion of scabies, especially in a patient with
known risk factors. Scabies can usually be treated with topical permethrin, but in crusted scabies
penetration of topical medication may be impaired reducing their efficacy. In such cases, oral
ivermectin may be an alternative.

Reference:

1. Karthikeyan K. Crusted scabies. Indian J Dermatol Venereol Leprol 2009;75:340-7


2. NICE: Management of scabies

3. Ivermectin Correct answer selected

A 75-year-old woman attends the dermatology outpatient clinic with a longstanding ulcer on her left
lateral lower shin.

She has a history of type 2 diabetes mellitus and mild congestive cardiac failure. She is overweight
and mobility is limited to her house with the use of a zimmer frame.

On examination, she has a 20mm ulcer on her left lateral lower shin with surrounding
hyperpigmentation, varicosities and pitting oedema of both lower legs to her knees. Her right leg
shows dull reddish discolouration with induration.

Foot pulses were palpable.

What of the following would be the most appropriate treatment?

1. Advise elevating left lower leg


2. Multilayer compression bandaging with ABPI
3. Single layer compression bandaging
4. Urgent incisional biopsy
5. Use compression stockings
Explanation:

A large Cochrane systematic review has demonstrated that chronic venous leg ulcers heal faster with
compression, specifically with multi-component systems containing elastic which are more effective
than single-component systems.

Reference:

1. O’Meara S et al. Compression bandaging and stockings to help the healing of venous ulcers.
Cochrane Systematic Review Database 2012

2. Multilayer compression bandaging with ABPI Correct answer selected


A 76-year-old obese diabetic woman is seen on a home visit complaining of low-grade fever and
pain in a chronic foot ulcer which has deteriorated over the last 24 hours. She is on metformin and
insulin therapy, following the instructions of the diabetologist.

Her blood pressure is 140/100 mmHg, and capillary glycemia 10.5 mmol/L. On examination, she has
a temperature of 38.2oC, a pulse of 106 bpm and a blood pressure of 128/80 mmHg.

The skin examination reveals marked erythema and oedema on the right ankle and foot.

On the plantar surface, a malodorous irregular ulcer is observed, draining a yellowish exudate. The
peripheral pulses are palpable.

There is no cyanosis, necrosis or gangrene.

Which of the following would be the most appropriate management for this patient?

1. Arrange admission to hospital


2. Arrange for her to be seen in the diabetes foot clinic at the next available clinic
3. Commence oral antibiotics and arrange routine referral to the wound healing team
4. Commence oral flucloxacillin and see again in 1 week
5. Commence oral flucloxacillin and clarithromycin and arrange diabetic foot review
Explanation:

This elderly patient has symptoms and signs of cellulitis in a chronic foot ulcer (diabetic foot), which
is a serious complication in view of her multiple comorbidities (obesity, diabetes, high blood
pressure). Infected diabetic foot represents a real threat to the involved limb, thus needing active and
prompt antibiotic treatment. In diabetic patients, classic signs of infection, like pain, fever and
leukocytosis may be absent. In this patient, guidance would suggest that she would be categorised as
Class 2/3 in terms of severity score, suggested by systemic illness (pyrexia, tachycardia) together
with unstable co-morbidities, and so she requires hospitalisation. IV antibiotics would be appropriate
and IV flucloxacillin is considered first-line.

Reference:

1. Health Service Executive IE (2009) National best practice and evidence based guidelines for wound management
2. Clinical Resource Efficiency Support Team (CREST) (2005). Guidelines for the management of cellulitis in adults

3. Arrange admission to hospital Correct answer selected


An 82-year-old lady with moderately-severe Alzheimer’s dementia is referred from a primary health
care unit to your emergency clinic. The referral form is incomplete and just says: "elderly woman
with infected leg wound referred for adequate antibiotic therapy"

. Which of the following is the earliest sign indicating secondary bacterial infection in a chronic
ulcer?

1. Acutely increasing pain


2. Frequent bleeding
3. Moderate serous exudate
4. Severe itchiness
5. Worsening in the lymphoedema
Explanation:

Chronic ulcers may complicate with secondary infection, and contact irritant or allergic dermatitis,
the latter mainly due to topical medications. The earliest symptom of secondary bacterial infection in
a chronic ulcer is pain exacerbation. Later, other clinical signs of cellulitis may develop (swelling,
redness, malodorous exudate). If untreated, such infections can progress to periostitis and
osteomyelitis.

Reference:

1. Oakley, A. (2009). Leg ulcers CME.DermNet New Zealand

2. Acutely increasing pain Correct answer


A 71-year-old man presented with a raised pigmented lesion on his abdomen which had been present
for many years, his wife felt that this had grown over the last year but the patient believed the lesion
hadn’t changed.

He had no previous skin problems and no relevant family history.

Which of the following is the most likely diagnosis?

1. Seborrhoeic keratosis
2. Malignant melanoma
3. Basal cell carcinoma
4. Actinic keratosis
5. Lentigo
1.

Explanation:

Pigmented lesions should always raise the possibility of melanoma. This lesion demonstrated 3
distinct colours this would be unusual in benign moles which tend to have one or two colours. It is
not unusual for the patient to give a history suggesting the lesion hadn’t changed and this shouldn’t
be relied upon. Seborrhoeic keratoses are usually regularly pigmented and have a stuck-on
appearance. Basal cell carcinomas can be pigmented but this usually presents as blue/grey globules
within the lesion. Lentigos are evenly pigmented and brown whilst actinic keratoses are red and
rough to the touch.

Reference:

2. Dermnetnz: melanoma

3. Malignant melanoma Correct answer selected


A 70-year-old man attended his GP surgery to review his blood pressure. He had a history of
intermittent claudication and hypercholesterolaemia, smoked 20 cigarettes a day and drinks 18 units
of alcohol weekly.

He takes aspirin and simvastatin.

On examination, he was apyrexial, had a heart rate of 82bpm, a blood pressure of 156/110mmHg and
a respiratory rate of 14 breaths per minute.

Since this was the third hypertensive reading, the GP prescribed lisinopril.

Two weeks later, the man returned, feeling generally unwell. He was admitted to the hospital, where
the following urea and electrolyte results were obtained:

Investigation Result Normal range


Sodium 140 mmol/L 134-1440 mmol/L
Potassium 4.0 mmol/L 3.5-5.0 mmol/L
Urea 13.2 mmol/L 2.5-6.7 mmol/L
Creatinine 499 µmol/L 70-150 µmol/L
Calcium 2.32 mmol/L 2.2-2.6 mmol/L
Bicarbonate 28 mmol/L 22-30 mmol/L
Magnesium 0.90 mmol/L 0.75-1.05 mmol/L

What is the most likely underlying condition?

1. Focal segmental glomerulonephritis


2. Lisinopril hypersensitivity.
3. Minimal change glomerulonephritis.
4. Polycystic kidney disease.
5. Renal artery stenosis
Explanation:

This man is in acute renal failure because he suffers from bilateral renal artery stenosis and took
ACE inhibitors. Glomerular filtration rate (GFR) usually depends on the rate of blood flow in the
afferent glomerular arteriole. That is an increase in flow results in an increased GFR. In patients with
renal artery stenosis, flow through the afferent arteriole is restricted. This activates the renin-
angiotensin system, resulting in hypertension. It also means that GFR is dependent on angiotensin II-
mediated efferent arteriolar vasoconstriction. Consequently, preventing angiotensin II production
with an ACE inhibitor will cause a fall in GFR and renal failure. This effect will only occur in
patients with bilateral renal artery stenosis. It is important to be vigilant in prescribing ACE
inhibitors to patients with any known arterial disease.

Reference:

1. NICE: lisinopril
2. Medicine leaflet: Lisinopril
3. Acute kidney Injury
4. NICE: Acute kidney injury
5. RCPE: Acute kidney injury recognition

6. Renal artery stenosis Correct answer selected

A 72-year-old man presented two weeks ago with a burning discomfort on the left side of his back,
followed by an intensely painful, pruritic rash of red vesicles.

He was diagnosed with herpes zoster (shingles) and treated with aciclovir.

The rash subsided but significant pain remained. This pain was managed with a TENS machine and
amitriptyline.
He has type 2 diabetes mellitus, hypertension and hypercholesterolaemia, for which he takes
metformin and gliclazide, lisinopril and simvastatin.

He presents today with acute low abdominal pain and an inability to pass urine.

On examination, he is apyrexial, has a heart rate of 74bpm, blood pressure of 142/100mmHg and a
respiratory rate of 16. He has a very tender suprapubic region.

Which drug has caused this man’s problems?

1. Acyclovir
2. Amitriptyline
3. Gliclazide
4. Lisinopril
5. Simvastatin

Explanation:

This man has acute urinary retention. Tricyclic antidepressants, including amitriptyline, have
anticholinergic adverse effects including dry mouth, blurred vision, constipation, urinary retention,
and drowsiness. Some tolerance develops to these effects and patients should be encouraged to
persevere as long as these effects are mild. Severe adverse effects, such as acute urinary retention
require prompt treatment. It can be difficult and demoralising when the adverse effects of a drug
manifest themselves before the beneficial ones, so it is important to know when this will happen.

It is not recommended that patients with depression take amitriptyline, since it is particularly
dangerous in overdose.

Reference:

1. NICE: amitriptyline-hydrochloride
2. Medicine leaflet: Amitriptyline

3. Amitriptyline Correct answer selected

A 70-year-old woman presented to her GP with tremor and agitation. She has a past history of
depression for which she has been on long term lithium therapy which has provided effective control.

More recently, her GP noted that she was hypertensive and commenced new therapies.

Initiation of which of the following is most likely to have precipitated her lithium toxicity?

1. Amlodipine
2. Atenolol
3. Bendroflumethiazide
4. Doxazosin
5. Simvastatin
1.

Explanation:

Lithium is a drug used for bipolar affective disorder. It is a drug that needs to be kept within a
narrow therapeutic range. If the drug level goes above this range, lithium toxicity occurs. Lithium
toxicity ranges in severity from mild tremors, agitation, and confusion to ataxia, convulsions and
arrhythmias. Interactions can occur between lithium and other agents that precipitate toxicity; in
particular, diuretics (both loop and thiazides) through sodium depletion and reduction of excretion
which increases the potential for lithium toxicity. ACE inhibitors may also cause lithium toxicity as
can NSAIDs.

Reference:
2. http://www.patient.co.uk/doctor/Lithium.htm
3. Lithium prescribing guidelines

4. Bendroflumethiazide Correct answer selected

A 77-year-old woman visited her GP to discuss her polydipsia and sight problems. The GP took a
blood sample for fasting glucose, which was 12.2 mmol/L (normal is 3.5-5.5 mmol/L). He also sent a
sample for urea and electrolytes. The results are as follows:

Investigation Result Normal range


Sodium 139 mmol/L 134-144 mmol/L
Potassium 4.4 mmol/L 3.5-5 mmol/L
Creatinine 80 µmol/L 50-100 µmol/L
Urea 4.7 mmol/L 2.5-6.7 mmol/L
Calcium 2.25 mmol/L 2.2-2.6 mmol/L
She returned for subsequent visits where further fasting glucose samples confirmed that she had type
2 diabetes. She was also found to have hypertension and hypercholesterolaemia. The GP prescribed
metformin, verapamil, atenolol and simvastatin.

Six months later, HbA1c results and glucose monitoring showed that glycaemic control was still
inadequate, so the GP added glipizide.

In addition, the patient reported low mood and early morning waking. After careful exploration of
these issues, the GP diagnosed depression and prescribed fluoxetine.
One month later, the patient presented with loss of appetite, nausea and vomiting, headache,
restlessness and irritability.

On examination, she was apyrexial, had a heart rate of 74bpm, blood pressure of 116/72mmHg and a
respiratory rate of 16 breaths per minute.

Physical examination was otherwise unremarkable. Blood for random glucose and full blood count
was taken.

Investigation Result Normal range


Glucose 7.6 mmol/L 3.5-5.5 mmol/L
Sodium 119 mmol/L 134-144 mmol/L
Potassium 3.4 mmol/L (3.5-5 mmol/L)
Creatinine 92 µmol/L 50-100 µmol/L
Urea 4.9 mmol/L 2.5-6.7 mmol/L
Calcium 2.27 mmol/L 2.2-2.6 mmol/L
Which is the appropriate treatment to remove for this patient?

1. Atenolol
2. Fluoxetine
3. Metformin
4. Nifedipine
5. Simvastatin

Explanation:

Selective serotonin reuptake inhibitors (SSRIs) are known to cause hyponatraemia. While it is more
common in older patients, this effect is not exclusively seen in that age group. SSRIs are thought to
cause hyponatraemia by inducing a syndrome of inappropriate antidiuretic hormone (SIADH).
Antidiuretic hormone causes water to be absorbed from the collecting ducts of the kidneys, resulting
in concentrated urine. Therefore, urine sodium concentration should be measured. A concentration of
>20mmol/L and a urine osmolality of >500mosmol/kg, in the presence of hyponatraemia, is
diagnostic. Treat by stopping fluoxetine and consider alternative antidepressant therapy for this
patient.

Reference:

6. NICE: fluoxetine
7. Spasovski, G. Vanholder, R. Allolio, B. et al. Clinical practice guideline on diagnosis and treatment
of hyponatraemia in European Journal of Endocrinology. (2014) 170;3, G1-G47. Doi:
https://doi.org/10.1530/EJE-13-1020
8. Weismann, D. Schneider, Hoybye, C. Clinical aspects of symptomatic hyponatremia in Endocrine
Connections, 5;5. R35-R43. Doi: https://doi.org/10.1530/EC-16-0046
9. Fluoxetine Correct answer selected

A 64-year-old woman attended her GP surgery to enquire about motion sickness treatment. In
addition, the GP decided to review her blood pressure. She has a history of osteoarthritis and
hypercholesterolaemia and takes co-codamol and simvastatin. She has never smoked and consumes 8
units of alcohol weekly.

On examination, she was apyrexial, had a heart rate of 82bpm, a blood pressure of 150/108mmHg
and a respiratory rate of 16 breaths per minute.

Since this was the third hypertensive reading, the GP prescribed lisinopril and cinnarizine for motion
sickness.

A week later, the patient returned, complaining of raised, red lesions on her legs, arms, and trunk. On
inspection, there is a rash consisting of widespread red wheals of approximately 1cm diameter.

What substance is most likely to be responsible?

1. Cinnarizine
2. Codeine
3. Lisinopril
4. Paracetamol
5. Simvastatin
1. Lisinopril Correct answer selected

Explanation:

ACE inhibitors are known to cause urticarial rash, especially lisinopril.

The lisinopril should be stopped since there is some risk of anaphylaxis and alternative
antihypertensive medication should be considered.

A suitable antihistamine should be prescribed, but in this case, the patient should be warned not to
take it with cinnarizine, which is also an antihistamine.

Reference:

2. NICE: lisinopril
3. Medicine leaflet: Lisinopril
4. Urticaria
5. Angiooedema and urticaria with angiotensin converting enzyme inhibitors P. I. Pillans, D. M.
Coulter and P. Black European Journal of Clinical Pharmacology Volume 51, Number 2 / October,
1996.
A 76-year-old man was admitted to hospital with progressive shortness of breath at rest. He has a
history of mild exercise-induced angina, atrial fibrillation, hypertension and type II diabetes mellitus.

He was taking pioglitazone, metformin, isosorbide mononitrate, digoxin, atorvastatin, bendrofluazide


and warfarin.

On examination, he was apyrexial, had a heart rate of 84bpm irregular, blood pressure of 148/100
mmHg and a respiratory rate of 34 breaths per minute. Auscultation of the chest revealed bilateral
crepitations.

There was oedema of both ankles.

Which of the medications is most likely to have been responsible for his presentation?

1. Atorvastatin
2. Bendrofluazide
3. Digoxin
4. Metformin
5. Pioglitazone
Explanation:

Studies suggest that approximately 5% of patients taking glitazones develop oedema which may
precipitate heart failure. Oedema is often worsened with the addition of other therapies such as
calcium antagonists. NICE recommends that pioglitazone should be avoided in people with heart
failure, and those at risk of fracture. If used in patients taking insulin, the danger is further increased.
Treatment is to stop pioglitazone and treat heart failure as usual.

Reference:

1. NICE: pioglitazone
2. Medicine leaflet: Pioglitazone
3. NICE clinical guidelines 66: type 2 diabetes

4. Pioglitazone Correct answer selected


You are asked to review a 78-year-old male being treated for bronchogenic carcinoma. You've been
asked to see the patient as he has become more unwell and confused.

He has a past history of hypertension and diet-controlled diabetes.

Currently, he takes amlodipine, ramipril, aspirin and a statin.

Which of the following symptoms help you make the diagnosis of hypoactive delirium?

1. Visible Agitation
2. Auditory hallucinations
3. Visual hallucinations
4. Lack of co-operation with reasonable requests
5. Changes in appetite
Explanation:

Indicators of delirium: at presentation

At presentation, assess people at risk for recent (within hours or days) changes or fluctuations in
behaviour.

These may be reported by the person at risk, or a carer or relative. Be particularly vigilant for
behaviour indicating hypoactive delirium (marked *). These behaviour changes may affect:

1. Cognitive function: for example, worsened concentration*, slow responses*, confusion.


2. Perception: for example, visual or auditory hallucinations.
3.
4. Physical function: for example, reduced mobility*, reduced movement*, restlessness, agitation,
changes in appetite*, sleep disturbance.
5.
6. Social behaviour: for example, lack of cooperation with reasonable requests, withdrawal*, or
alterations in communication, mood and/or attitude.

Reference:

7. (NICE delirium 2010)

8. Changes in appetite Correct answer


A 78-year-old man is admitted with a four-day history of nausea, vomiting and visual disturbances.

He had a past history of congestive heart failure and atrial fibrillation for which he was taking stable
doses of losartan, warfarin, bisoprolol and digoxin.

Recently, he was diagnosed with H Pylori-associated gastritis and had been treated with
esomeprazole, clarithromycin and amoxicillin.

In the last month, he had also had spironolactone 25 mg added to his anti-failure therapy.

A diagnosis of digoxin toxicity was suspected and confirmed with a digoxin level of 3.3 ng/ml
(therapeutic range 0.5-1).

Which of the following is most likely to have precipitated the digoxin toxicity?

1. Amoxicillin
2. Clarithromycin
3. Esomeprazole
4. Spironolactone
5. Warfarin and spironolactone

1. Clarithromycin Correct answer selected


2. 6% Esomeprazole
3. 29% Spironolactone
4. 5% Warfarin and spironolactone

Explanation:

This patient has developed digoxin toxicity, which is most likely due to interaction with
clarithromycin. Clarithromycin is considered to raise plasma concentrations of digoxin through a
reduction in the gut flora responsible for some metabolism of digoxin or inhibition of cyto-p450
metabolism of digoxin.

Reference:

5. Consultant 360: Digoxin-Clarithromycin Interaction: Well-Reported Yet Oft-Forgotten

A 79-year-old male from a nursing home is admitted with a 5-day history of abdominal distention
and pain.

He has a background history of cognitive impairment, previous stroke, type 2 diabetes and
hypertension.
On examination, his abdomen is distended, with mild diffuse tenderness with sluggish bowel sounds.
His observations are as follows:

Temperature 37oC
Pulse 78 bpm
Blood pressure 110/70 mmHg

His abdominal X-ray shows diffuse colonic distention with caecal diameter of 7cm.

Per rectal examination shows empty rectum. His laboratory results are as follows:

Investigation Result Normal range


Haemoglobin 132 g/L 130 – 150 g/L
White cell count 8.2 x 109/L 4-11 x 109/L
Platelets 324 x 109/L 150-400 x 109/L
Neutrophils 5.3 x 109/L 1.7-7.5 x 109/L
Sodium 141 mmol/L 133 – 146 mmol/L
Potassium 3.4 mmol/L 3.5 -5.3 mmol/L
Urea 10.3 mmol/L 2.5-7.8 mmol/L
Creatinine 88 mmol/L 46-92 mmol/L
CRP 10 mg/L <5 mg/L
He is managed conservatively with intravenous fluids, kept nil by mouth and potassium replacement.

His repeat X-ray after 48 hours shows a caecal diameter of 9cm.

CT abdomen has excluded mechanical obstruction and perforation. What is the next step in the
management of this patient?

1. Emergency surgery for colonic resection


2. Nasogastric tube for decompression
3. Neostigmine
4. Phosphate enema
5. Urgent endoscopic decompression

Explanation:

This elderly patient has likely got acute pseudo-obstruction (also referred to as Ogilvie’s syndrome)
which is characterised by diffuse colonic dilatation in the absence of mechanical obstruction.

This typically is more severe on the right side mainly the caecum although the colonic dilatation may
extend to the rectum.
Risk factors for acute pseudo-obstruction includes hospitalised patients, post-operative state,
institutionalised patients, presence of multiple co-morbidities and certain medications.

There is a 3% risk of spontaneous colonic perforation and associated with high mortality.

Neostigmine is a cholinesterase inhibitor that enhances the availability of acetylcholine at neuro-


neuronal synapses in the enteric nervous system and at the neuromuscular junction thus facilitating
smooth muscle contractions.

Neostigmine has been shown in randomised controlled trials to have an initial response rate of 60%
to 90% in patients with acute pseudo-obstruction and should be tried before endoscopic
decompression.

Reference:

1. Chudzinski A.P., (2015) Thompson EV, Ayscue JM., Acute colonic pseudoobstruction. Clin Colon
Rectal Surg. 2015;28(2):112-7.
2. Vanek, V.W. and Al-Salti, M., (1986). Acute pseudo-obstruction of the colon (Ogilvie's syndrome).
Diseases of the colon & rectum, 29(3), pp.203-210.
3. Lopez Valle, R, G. Lopez Godoy, F. Neostigmine for acute colonic pseudo-obstruction: A meta-
analysis, in Annals of Medicine and Surgery Volume 3, Issue 3, September 2014, Pages 60-64

4. Neostigmine Correct answer selected

A 75-year-old woman with alcohol dependence was admitted into hospital with a 3-day history of
increasing abdominal pain and nausea.

She was taking 50 mg orally of thiamine daily and 20 mg omeprazole daily.

She admitted to drinking around one bottle of wine a day but had not drunk alcohol over the
preceding 3 days.

On examination, she was confused, appeared irritable with tremors of the hands and had yellow
sclera. Her temperature was 37oC, pulse 100 bpm regular and her blood pressure was 128/88 mmHg.

On examination of the abdomen, she had 4 cm tender hepatomegaly with no evidence of ascites.

Her investigations revealed:

Investigation Result Normal range


Sodium 131 mmol/L 134–144 mmol/L
Potassium 3.6 mmol/L 3.5–5 mmol/L
Urea 3 mmol/L 3–8 mmol/L
Creatinine 66 µmol/L 50–100 µmol/L
Glucose 3.6 mmol/L 3.5-5.5 mmol/L
Albumin 26 g/L 35-45 g/L
Bilirubin 80 µmol/L 1–20 µmol/L
ALT 280 iu/L 5-40 iu/L
Alkaline Phosphatase 164 U/L 50–110 U/L

Which of the following is the most appropriate immediate treatment for this patient?

1. 10% dextrose infusion


2. IV vitamin B complex
3. Oral diazepam as per a CIWA protocol
4. Oral lactulose
5. Prophylactic low molecular weight heparin

1. IV vitamin B complex Correct answer selected


2.

Explanation:

The most appropriate approach to the management of this patient who appears to have acute chronic
alcoholic hepatitis, possibly encephalopathy and alcohol withdrawal, would be the addition of IV
Thiamine/vitamin B complex such as Pabrinex to avert Wernicke-Korsakoff syndrome.

Whilst the patient is prescribed thiamine, poor compliance is often an issue and, where there is any
doubt about thiamine intake in alcoholism, the patient must be treated with IV vitamin B complex.
Vit B complex should certainly be administered before any glucose.

Reference:

3. Herrington, J.D., (2015) Why Alcoholics Should Receive Vitamin B1 (Thiamine) by IV Before any
Glucose Infusions. [ONLINE]
4. NICE: Alcohol withdrawal management for acute admissions to hospital
A 78-year-old man is admitted with a four-day history of nausea, vomiting, and visual disturbances.

He had a past history of congestive heart failure and atrial fibrillation for which he was taking stable
doses of losartan, warfarin, bisoprolol and digoxin.

Recently, he was diagnosed with H Pylori associated gastritis and had been treated with
esomeprazole, clarithromycin and amoxicillin.

In the last month, he had also had spironolactone 25 mg added to his anti-failure therapy.

Investigations reveal:

Investigation Result Normal range


FBC Normal
CRP 12 mg/dl <5 mg/dl
Urea 8.8 mmol/L 3-8 mmol/L
Creatinine 128 micromol/L 50-100 micromol/L
Sodium 138 mmol/L 134-144 mmol/L
Potassium 3.8 mmol/L 3.5-5.5 mmol/L
LFTs Normal
INR 2.2
ECG Rate 60/min controlled atrial fibrillation; nil acute
A diagnosis of digoxin toxicity was suspected and confirmed with a digoxin level of 3.3 ng/ml
(therapeutic range 0.5-1).

Which of the following is the most appropriate approach to the management of this patient?

1. Stop digoxin and give IV calcium gluconate


2. Stop digoxin and give digoxin-specific antibody fragments
3. Stop digoxin and give IV lidocaine
4. Stop digoxin and give IV potassium chloride
5. Stop digoxin only required at this stage

Explanation:

In this particular case, digoxin toxicity is likely to have been precipitated by the co-administration of
clarithromycin and digoxin. As for the options list, the digoxin should be stopped and, in this case,
only supportive treatment offered.

There is no clear guidance on the use of digoxin specific antibody fragments.

However, digoxin-specific antibody fragments are often reserved for digoxin toxicity associated with
life-threatening arrhythmias, in situations where the potassium concentration is 5 mmol/l or above
and where the concentration is above 12 ng/ml.

Useful treatments for digoxin related arrhythmias include atropine and lidocaine. In this case, with
the ECG revealing controlled AF, no antiarrhythmic is required.

Reference:

1. Pincus, M., 2016. Management of digoxin toxicity. Australian prescriber, 39(1), p.18.
2. NICE: digoxin

3. Stop digoxin only required at this stage Correct answer


A 65-year-old lady is being seen at outpatients following a hospital admission with an exacerbation
of asthma after contacting a common cold.

During her admission, she was given a five-day course of prednisolone and antibiotics.

She is asking about the need for bone protection. She was previously healthy with her asthma well
controlled with inhaled budesonide.

She does not smoke. Her 10-year fragility risk is low.

What advice would you give to this woman?

1. Offer lifestyle advice


2. To have a DEXA scan
3. To start alendronate
4. To start hormone replacement therapy
5. To start risedronate
Explanation:

"Fragility fracture risk should be calculated using the QFracture® (preferred) or FRAX® online
assessment calculators.

People at high risk should be offered a DXA scan to confirm osteoporosis.

People at intermediate risk whose fracture risk is close to the recommended threshold and who have
risk factors that may be underestimated by FRAX®, such as people taking high doses of oral
corticosteroids, should be offered a DXA scan.

People at low risk should not be offered treatment or a DXA scan, but given lifestyle advice."
–NICE CKS

Reference:

1. NICE (updated 2016). CKS: Osteoporosis – prevention of fragility fractures.


2. SIGN: management of osteoporosis and the prevention of fragility fractures
3. NOGG Guidelines

4. Offer lifestyle advice Correct answer


A 68-year-old man with a history of Huntington’s disease and atrial fibrillation presents with a one-
week history of headache and fluctuating altered consciousness.

His wife says he has been having frequent falls as of late.

He is currently on rivaroxaban 20 mg daily and tetrabenazine 25 mg TDS. On examination, the


patient is disoriented to time and place.

He has choreoathetotic movements and an unstable gait.

There is no fever, all parameters are within normal range.

The rest of the neurological examination is normal with downgoing plantars.

What is the most likely diagnosis in this man?

1. Encephalitis
2. Haemorrhagic stroke
3. Meningitis
4. Subarachnoid haemorrhage
5. Subdural haematoma
Explanation:

Patients on oral novel anticoagulants are at risk of bleeding which can be serious or fatal.

In a patient on anticoagulants with a history of head injuries and altered consciousness, a subdural
haematoma needs to be excluded.

In elderly people on anticoagulants the mechanism of head injury need not be severe for a subdural
haematoma to occur. A high level of suspicion is needed in these cases.

Subdural haematoma needs to be considered in the differential diagnosis of elderly patients


presenting with headache, fluctuating confusion and a normal neurological examination.

Reference:

1. Meagher, R.J. (updated 2018). Subdural hematoma. Medscape.

2. Subdural haematoma Correct answer selected


A 72-year-old woman was admitted with acute shortness of breath and palpitations. She is currently
being treated for post-influenza pneumonia.

She gives a history of palpitations over the past few months but has never really bothered to seek
medical help.

She is known to suffer from hypertension but has no other relevant medical history of note.

Examination and an ECG confirmed the presence of atrial fibrillation and the rate was controlled
with treatment. Heparin was started as an anticoagulant.

Following stabilisation of the heart rate and discharge from hospital, what anticoagulation would you
use?

1. Aspirin
2. Aspirin + clopidogrel
3. Enoxaparin
4. No anticoagulation needed
5. Rivaroxaban

1. Rivaroxaban Correct answer selected


Explanation:

This lady has a CHA2DS2 VASc score of 3 (female, 72 years old and hypertensive). Her annual
stroke risk is thus 3.2%.

Stroke risk was 3.2% per year in >90,000 patients (the Swedish Atrial Fibrillation Cohort Study) and
4.6% risk of stroke/TIA/systemic embolism.

One recommendation suggests a 0 score is "low" risk and may not require anticoagulation; a 1 score
is “low-moderate” risk and should consider antiplatelet or anticoagulation, and score 2 or greater is
“moderate-high” risk and should otherwise be an anticoagulation candidate.

The above is taken from the MDCalc CHA2DS2-VASc Score for Atrial Fibrillation Stroke Risk.

FORMULA

Addition of the selected points:

Criteria Points
Age <65 years old 0
65-74 years old +1
=75 years old +2
Sex Male 0
Female +1
Congestive heart failure history +1
Hypertension history +1
Stroke/TIA/thromboembolism history +2
Vascular disease history (prior MI, peripheral artery disease, or aortic +1
plaque)
Diabetes mellitus history +1
FACTS & FIGURES

Interpretation:

CHA2DS2-VASc Score Risk of ischaemic stroke Risk of stroke/TIA/systemic embolism


0 0.2% 0.3%
1 0.6% 0.9%
2 2.2% 2.9%
3 3.2% 4.6%
4 4.8% 6.7%
5 7.2% 10.0%
6 9.7% 13.6%
7 11.2% 15.7%
8 10.8% 15.2%
9 12.2% 17.4%
From Friberg (2012). Note the paradoxical decrease in risk between 7 and 8 points; this reflects the
findings published in the study, but in general, assume increasing risk with higher scores.

NICE clinical guideline [CG180]:


“Anticoagulation
Anticoagulation may be with apixaban, dabigatran etexilate, rivaroxaban or a
vitamin K antagonist.
NICE has also produced a patient decision aid about anticoagulation in atrial
fibrillation.
1.5.2 Consider anticoagulation for men with a CHA2DS2-VASc score of 1.
Take the bleeding risk into account. [new 2014]
1.5.3 Offer anticoagulation to people with a CHA2DS2-VASc score of 2 or
above, taking bleeding risk into account. [new 2014]
1.5.4 Discuss the options for anticoagulation with the person and base the choice
on their clinical features and preferences. [new 2014]”
Reference:

2. Friberg, L., Rosenqvist, M. and Lip, G.Y. (2012). Evaluation of risk stratification schemes for ischaemic stroke and bleeding in
182 678 patients with atrial fibrillation: the Swedish Atrial Fibrillation cohort study. European Heart Journal, 33(12), pp.
1500–1510.
3. NICE (2014). Clinical Guideline [CG180]: Atrial fibrillation: management.

A 72-year-old male presents with acute onset left facial droop, left arm weakness and dysarthria of
30 minutes duration.

He also describes some associated chest discomfort.

On admission, he had the above features, a pulse of 100 bpm reg, blood pressure of 160/98 mmHg.

There was mild left facial droop, weakness of the left arm and some mild dysarthria.

Troponin was 21 ng/l (<14). ECG shows:


Chest x-ray is shown:

Trop
onin
was
21
iu/L
(<14
)

Urge
nt
CT
head
scan was reported as normal.

Which of the following would be the most appropriate approach to the management of this patient?

1. Arrange CT chest
2. Arrange CT PA
3. Arrange coronary angiography
4. Arrange transthoracic echo
5. No further investigation required treat with urgent thrombolysis with rTPA

1. Arrange CT chest Correct answer selected


2.

Explanation

This patient has a presentation of an acute stroke with chest pain and slight elevation of the troponin
that should alert one to the possibility of aortic dissection particularly in the context of the widened
mediastinum as revealed on the CXR. In this case, the patient requires an urgent CT of chest with
focus on the aorta to confirm the diagnosis.

Aortic dissection is classified as per the Stanford system.

Type A involves the ascending aorta and type B occurs distal to the left subclavian artery.
Recognised risk factors include hypertension, cystic medionecrosis, bicuspid aortic valve and
Marfan’s/Ehlers-Danlos syndrome.

The most frequent neurological sequelae associated with a type A aortic dissection is stroke and
particularly right hemisphere stroke.

Only two-thirds report associated chest pain.

Reference:
3. Gaul, C., Dietrich, W., Friedrich, I., Sirch, J. and Erbguth, F.J. (2007). Neurological Symptoms in
Type A Aortic Dissections. Stroke, 38(2), pp.292–297.

An 87-year-old male attends the Geriatric clinic to discuss headaches that he has been suffering. He
is a widower and he freely admits to "being a bit of a worrier" about his health, despite never really
bothering the doctor and taking no regular medication.

He appears panicked, clutching a private healthcare report that includes some blood work.

He points out his cholesterol of 6.7.

This, he says, is despite walking two miles to town (there and back) on a daily basis and being
religious about a low fat, low-carbohydrate diet. He keeps records of his meals and he shows them to
you.

This confirms he has a diet that is nutritionally well balanced, following all the public health advice
about portion size, carbohydrate and fat consumption.

His blood pressure is 128/80.

What would be your advice to him?

1. Offer him a prescription for Bezafibrate


2. Reassure him that at 87 treatment is unnecessary
3. Suggest he continues with his strict diet and re-test his cholesterol
4. Suggest he takes atorvastatin 20mg once daily
5. Suggest he adds an Omega-3 supplement to his diet
Explanation:

NICE actually suggests that in the over 85s, primary prevention with atorvastatin 20mg may reduce
the risk of non-fatal myocardial infarctions.

They specifically comment that frailty, polypharmacy, comorbidity and life expectancy should be
considered. It seems that this patient is remarkably fit and should therefore be considered for Statin
therapy.

Reference:

1. NICE: Cardiovascular disease: risk assessment and reduction, including lipid modification Clinical
guideline [CG181]Published date: 18 July 2014 Last updated: 27 September 2016

2. Suggest he takes atorvastatin 20mg once daily Correct answer selected


An 82-year-old lady presents for a "well-woman" appointment. Her only comorbidity is psoriasis
that she self manages with a number of over-the-counter creams.

She takes no regular medication. At her appointment, her blood pressure is 149/89.

She dutifully goes home and utilises her husband's blood pressure machine to take her BP over a
number of days.

She comes back having prepared a spreadsheet showing that her average BP is 146/88.

What advice should she be given?

1. Amlodipine 5mg should be prescribed and up titrate to response


2. Bendroflumethiazide 2.5mg should be started and a second agent added if needed
3. Doxazosin 2mg should be offered and uptitrated to response
4. Lifestyle advice should be given
5. She should start ramipril 1.25mg and up titrate to response
Explanation:

NICE Guidance suggests that in this age group, patients with a blood pressure of 150/90 should be
considered for antihypertensive treatment, other stage 1 hypertensive patients should receive lifestyle
advice.

Reference:

1. NICE: Starting antihypertensive drug treatment

2. Lifestyle advice should be given Correct answer selected


A 72-year-old female is admitted with a two-day history of delirium associated with visual
hallucinations.

On review of her history, she had been prescribed Drug X 4 days before this presentation. She had no
previous history of any psychiatric disorder.

Examination revealed the patient to be agitated, slightly confused, apyrexial, and she complains of
seeing insects on her bedsheets. Neurological examination is otherwise normal.

CT head scan was reported as normal and all blood tests were otherwise normal.

Researching the literature, there were a few prior case reports of Drug X being associated with
similar presentations.

On stopping Drug X, her symptoms resolved entirely.

Which of the following defines the likelihood of Drug X is responsible for this reaction as
determined by the Adverse Drug Reaction Probability Scale (Naranjo’s Scale)?

1. Improbable
2. Definite
3. Doubtful
4. Possible
5. Probable
Explanation:

The ADR probability scale or Naranjo Scale was developed as a way of assessing ADRs particularly
in clinical trials and has been widened in the assessment of potential ADRs in clinical practice. The
scale consists of 10 questions with specific weightings as shown below:

Adverse Drug Reaction Probability Scale

Question Yes No Do Not Know Score


1. Are there previous conclusive reports on this +1 0 0
reaction?
2. Did the adverse event appear after the suspected +2 -1 0
drug was administered?
3. Did the adverse event improve when the drug +1 0 0
was discontinued or a specific antagonist was
administered?
4. Did the adverse event reappear when the drug +2 -1 0
was readministered?
5. Are there alternative causes that could on their -1 +2 0
own have caused the reaction?
6. Did the reaction reappear when a placebo was -1 +1 0
given?
7. Was the drug detected in blood or other fluids in +1 0 0
concentrations known to be toxic?
8. Was the reaction more severe when the dose +1 0 0
was increased or less severe when the dose was
decreased?
9. Did the patient have a similar reaction to the +1 0 0
same or similar drugs in any previous exposure?
10. Was the adverse event confirmed by any +1 0 0
objective evidence?
In our case, the reaction has been reported previously (+2), it appeared after the drug was
administered (+2); it improved after the drug was stopped (+1), we do not know if the ADR
reappeared (0); there were no other alternative causes evident (+2); no placebo was given (0); No
blood concentrations were measured (0); we don’t know re the increase or decrease in dose (0); there
was no previous exposure (0). The final question relating to whether the ADR was confirmed by any
objective measure is a little more tricky as it is difficult to determine if the patient had hallucinations
or not but we are told that the patient had features of delirium - so the patient would score +1. This
gives a total of 8 making it PROBABLE on the scoring system.

Greater than or equal to 9 – Definite


5-8 – Probable
1-4 possible
Less than 0 – doubtful

Reference:

1. Murayama, H., Sakuma, M., Takahashi, Y. and Morimoto, T., 2018. Improving the assessment of adverse drug reactions
using the Naranjo Algorithm in daily practice: The Japan Adverse Drug Events Study. Pharmacology Research &
Perspectives, 6(1), p.e00373.

2. Probable Correct answer selected


An 86-year-old female was admitted with a two-day history of anorexia, nausea, weakness and
occasional vomiting. She did not want to eat anything and felt continually nauseous. If she tried to
eat anything then she would vomit. She was not sure if there was food stuck in her gullet or not. She
gave no history of weight loss. She had a slight productive cough one week before this and had seen
her GP. He had diagnosed a chest infection and had treated her with amoxicillin and clarithromycin.
She had stopped taking the antibiotics after 4-5 days as she had developed nausea, but this had
deteriorated despite stopping her antibiotics.

Her past medical history included congestive heart failure and atrial fibrillation for which she was
taking furosemide 40 mg/d, digoxin 0.25mg/d, ramipril 5mg/d, warfarin and spironolactone 25mg/d.

On examination, she had a pulse of 55 bpm irregular, a blood pressure of 122/80 mmHg and a
temperature of 37.3oC.

Investigations revealed:

Investigation Result Normal range


FBC Normal
CRP 12 mg/dl <5 mg/dl
Urea 8.8 mmol/L 3-8 mmol/L
Creatinine 128 micromol/L 50-100 micromol/L
Sodium 138 mmol/L 134-144 mmol/L
Potassium 5 mmol/L 3.5-5.5 mmol/L
LFTs Normal
INR 2.2
Chest x-ray Some upper lobe blood diversion
Abdominal Nil acute; some constipation
XR
ECG Rate 60/min atrial fibrillation; nil
acute
Which of the following investigations would you request for this patient?

1. Calcium
2. CT headscan
3. Digoxin level
4. Troponin
5. Urine legionella antigen

1.

Explanation:
This elderly female has been admitted with anorexia, nausea and vomiting following treatment for a
suspected chest infection.

This could well be an adverse reaction between digoxin and clarithromycin causing digoxin toxicity.

This is reported in the literature and the mechanism is thought to be either related to a reduction in
gut bacterial metabolism of digoxin resulting in increased absorption or the inhibition of cytochrome
p450 resulting in reduced metabolism of digoxin.

It is possible that this patient has hypercalcaemia as the cause for her symptoms, although this seems
less likely and digoxin toxicity needs to be excluded.

Reference:

2. Consultant 360: Digoxin-Clarithromycin Interaction: Well-Reported Yet Oft-Forgotten.

3. Digoxin level Correct answer selected

An 89-year-old woman with dementia and a clinical frailty score of 9 was admitted from her care
home with sepsis and a Glasgow coma score (GCS) of 12.

Which action is most likely to determine her outcome at 30 days?

1. Assessment for delirium


2. Early discussion about her future preferences, involving advocates
3. Early do not attempt resuscitation (DNAR) decision and adoption of palliative care
4. Early intravenous fluids and broad spectrum antibiotics
5. Early medication review to seek out potential inappropriate prescribing
Explanation:

In this elderly patient, it is important to undertake an early discussion about her future preferences
which can also include patient advocates. Given this patient is likely to have delirium due to sepsis
together with the frailty, the likely prognosis is poor and we are looking at end of life care. Therefore
one would consider withdrawal of treatment but without discussion and determining the patient
preferences – involving key informants as necessary, there is a serious risk that there is therapeutic
nihilism.
Reference:

1. Atterton, B. Paulino, M. C. Povoa, P. Martin-Loeches, I. Sepsis Associated Delirium. Medicina


(Kaunas). 2020 May; 56(5): 240. Published online 2020 May 18. doi: 10.3390/medicina56050240.

2. Early discussion about her future preferences, involving advocates Correct answer selected

A 78-year-old woman presents with sudden onset of bilateral shoulder pain. This started suddenly
two weeks ago and was not preceded by any unusual exertion. The pain is worse in the first hour of
the morning.

There has been no pain, visual disturbances, joint swellings or rashes.

Blood tests show a raised ESR and CRP with a normal creatine kinase.

What is the most likely diagnosis?

1. Dermatomyositis
2. Fibromyalgia
3. Osteoarthritis
4. Polymyalgia rheumatica
5. Polymyositis
1.

Explanation:

Patients with PMR tend to present with an acute onset of pain that usually involves the shoulder or
pelvis.

Pain is worse in the morning.

Initially, both ESR and CRP are raised.

It is important to recognise PMR as up to a third of patients can get giant cell arteritis which can lead
to blindness.

Reference:
2. Royal College of Physicians (2010). Diagnosis and management of polymyalgia rheumatica.

3. Polymyalgia rheumatica Correct answer selected

The DAPA-HF study (NEJM 2019) randomised ~4700 patients with heart failure and reduced
ejection fraction to either dapagliflozin (10mg daily) or placebo in addition to conventional heart
failure treatments.

The primary endpoint of the trial was a composite of worsening heart failure or cardiovascular death.

Which of the following was demonstrated in this study?

1. A significant reduction of the primary end-point was demonstrated only in sub-group analysis within
the group with type 2 diabetes
2. There was a significant reduction in cardiovascular death but no difference in worsening of heart
failure in those subjects treated with dapagliflozin versus placebo
3. There was a significant reduction in the primary end-point but a significant increase in serious
adverse events associated with dapagliflozin versus placebo
4. There was a significant reduction in the primary end-point in those patients treated with
dapagliflozin versus placebo
5. There was a small but non-significant reduction in the primary end-point in those treated with
dapagliflozin versus placebo
Explanation:

SGLT-2 inhibitors have been extensively adopted in the treatment of patients with type 2 diabetes
following studies such as EMPA-REG, which revealed significant improvements in glycaemic
control but also reductions in cardiovascular risk in addition to conventional treatments. The role of
SGLT-2 inhibitors has been further extended to patients with heart failure associated with a reduced
ejection fraction following landmark studies such as the one above – DAPA-HF published in the
NEJM in late 2019. The study revealed the following as described in the abstract:

‘Over a median of 18.2 months, the primary outcome occurred in 386 of 2373 patients (16.3%) in the
dapagliflozin group and in 502 of 2371 patients (21.2%) in the placebo group (hazard ratio, 0.74;
95% confidence interval [CI], 0.65 to 0.85; P<0.001).

A first worsening heart failure event occurred in 237 patients (10.0%) in the dapagliflozin group and
in 326 patients (13.7%) in the placebo group (hazard ratio, 0.70; 95% CI, 0.59 to 0.83).

Death from cardiovascular causes occurred in 227 patients (9.6%) in the dapagliflozin group and in
273 patients (11.5%) in the placebo group (hazard ratio, 0.82; 95% CI, 0.69 to 0.98); 276 patients
(11.6%) and 329 patients (13.9%), respectively, died from any cause (hazard ratio, 0.83; 95% CI,
0.71 to 0.97).

Findings in patients with diabetes were similar to those in patients without diabetes.

The frequency of adverse events related to volume depletion, renal dysfunction, and hypoglycaemia
did not differ between treatment groups.’
In consequence, SGLT-2 inhibitors are increasingly being adopted for the treatment of such patients.

Such conclusions regarding class effect have been supported with the use of empagliflozin in the
EMPEROR-REDUCED study showing similar reductions in mortality and morbidity in subjects
with heart failure.

Reference:

1. McMurray, J.J., Solomon, S.D., Inzucchi, S.E., Køber, L., Kosiborod, M.N., Martinez, F.A.,
Ponikowski, P., Sabatine, M.S., Anand, I.S., Belohlávek, J. and Böhm, M., 2019. Dapagliflozin in
patients with heart failure and reduced ejection fraction. New England Journal of Medicine, 381(21),
pp.1995-2008.
2. Packer, M., Anker, S.D., Butler, J., Filippatos, G., Pocock, S.J., Carson, P., Januzzi, J., Verma, S.,
Tsutsui, H., Brueckmann, M. and Jamal, W., 2020. Cardiovascular and renal outcomes with
empagliflozin in heart failure. New England Journal of Medicine, 383(15), pp.1413-1424.

3. There was a significant reduction in the primary end-point in those patients treated with dapagliflozin
versus placebo Correct answer selected
A 68-year-old male presents to ER with acute onset paraparesis and distal paresthesias.

An examination reveals hypotonia and reduced reflexes in both the lower limbs.

There is no sensory level and Babinski’s sign is absent.

He is afebrile with no bowel or bladder involvement.

He has no similar medical history.

A lumbar puncture is performed and the cerebrospinal fluid (CSF) findings are as follows:

Investigation Result Normal range


Colour Colourless
Appearance Clear
Glucose 65 mg/dL 50-80
Protein 100 mg/dL 15-45
White Cell Count 1-2 1-5/NPF
RBC Nil

What is the most likely diagnosis?

1. Acute viral meningitis


2. Guillain-Barre syndrome
3. Parasagittal meningioma
4. Subacute combined degeneration of the cord
5. Transverse myelitis
1.

Explanation:

This is a case of Guillain-Barré syndrome: AIDP variant.

This patient presented with acute onset, bilaterally symmetrical flaccid paralysis with no bowel and
bladder involvement and no fever.

All these features suggest a diagnosis of GBS (as per Asbury criteria).

In addition to this, there is no Babinski’s sign, sensory involvement, nor any sensory level - that
excludes myelopathy.

There is no significant past history or any other risk factors, which rules out multiple sclerosis and
acute vascular events.

CSF analysis reveals characteristic cyto-albumino dissociation and nerve conduction studies show
conduction slowing.

The treatment of choice for GBS is IV immunoglobulins and plasmapheresis.

Reference:

2. Ramachandran, T, et al. (2018). ‘Acute Inflammatory Demyelinating Polyradiculoneuropathy’,


Medscape.

3. Guillain-Barre syndrome Correct answer selected

A 65-year-old man presents in the emergency department with acute onset weakness of both legs.

Examination reveals grade-2 power, hypotonia and areflexia of both lower limbs.

He has no sensory loss or sensory level, and there is no bowel and bladder involvement. Babinski’s
sign is absent.
The rest of the systemic examination is normal. His past history is unremarkable.

How should you manage this patient?

1. Alteplase
2. IV Immunoglobulins
3. Methylcobalamin
4. Methylprednisolone
5. Ocrelizumab

Explanation:

This patient presents with a case of Guillain-Barré syndrome (AIDP variant), and the treatment of
choice is IV immunoglobulins and plasmapheresis.

Intravenous immunoglobulins (IVIg) have been shown in a number of trials to be an effective and
standard treatment for the Guillain-Barré syndrome (GBS) because of their ease of administration
and better side effect profile.
IVIg is thought to affect many immune system components, including B-cells, T-cells, macrophages,
complement, cytokines, and cellular adhesion molecules, to exert its immunomodulatory impact.

Alteplase is a thrombolytic used in acute Ischaemic stroke while methylcobalamin is used in vitamin
B12 deficiency. Methylprednisolone is used in acute attacks of multiple sclerosis.

Ocrelizumab is a humanised anti-CD20 monoclonal antibody used in multiple sclerosis.

Reference:

1. Ramachandran, T, et al. (2018). ‘Acute Inflammatory Demyelinating Polyradiculoneuropathy’,


Medscape.

2. IV Immunoglobulins Correct answer selected

A 72-year-old male has been admitted under the surgeons due to small bowel obstruction related to
an incisional hernia. He has a history of atrial fibrillation (AF) for which he is being treated with
apixaban and bisoprolol.

The surgeons have asked for an opinion regarding the management of his AF, as they have scheduled
his procedure for 24 hrs hence. He is nil by mouth and is receiving IV fluids with NG suction.

On examination, he has a temperature of 37.2oC, his pulse is 125-135 bpm in AF and he has a blood
pressure of 132/80 mmHg.

What is the most appropriate approach to the further control of this patient’s AF?

1. Continue bisoprolol via the NG tube


2. Switch to IV diltiazem od
3. Switch to IV metoprolol tds
4. Switch to IV verapamil od
5. Use IV digoxin
1.

Explanation:

This patient has uncontrolled AF which requires control prior to surgical intervention.

While his AF previously appeared well controlled with bisoprolol, this is not available as an IV
preparation and, therefore, needs to be changed to IV metoprolol, at a dose of about 2.5mg tds.
Hopefully, this will provide good rate control and can be increased to 5mg tds, if required.

Verapamil or diltiazem are options in those where beta-blockers are contraindicated, however, as we
are not told about the last dose of bisoprolol that this patient received, it is best to avoid these
calcium antagonists. Digoxin would be the third line option.

Reference:

2. Atrial Fibrillation Management

3. Switch to IV metoprolol tds Correct answer selected


An 84-year-old female is admitted acutely as the GP is concerned due to the purple discolouration of
the urine in the catheter bag.

She has a long-term urine catheter in place, as she had a stroke 5 years ago.

She had incontinence and is a resident in a care home.

The staff were concerned that the urine in the bag was purple.

She has also been a little more constipated of late.

There is nothing else of note and she feels reasonably well. She takes losartan, bisoprolol,
atorvastatin, apixaban and omeprazole.

On examination, she has a pulse of 76 bpm AF, a blood pressure of 108/78 mmHg and a temperature
of 37oC.

Aside from a right-sided hemiparesis, there are no other abnormalities other than purple
discolouration of the urine in the catheter bag.

Investigations reveal:

Investigation Result Normal range


FBC Normal
Urea 12.9 mmol/L 3 - 8 mmol/L
Creatinine 110 micromol/L 50 - 100 micromol/L
LFTs Normal
CRP 10 mg/L <5

Which of the following is the likely cause for her presentation?

1. Apixaban with losartan


2. Aspirin
3. Beetroot intake
4. Omeprazole
5. Urine tract infection

Explanation:

This is a typical description of purple urine bag syndrome where there is a urine tract infection that
results in the increased production of a blue pigment (indigo) and a red pigment (indirubin) which
results in the purple discolouration of the urine.

Treatment is focused on replacing the catheter, identification and treatment of the causative organism
together with treatment of constipation.

Whilst this patient remains well, it is important to recognise that a UTI is the cause for her
presentation and this should be treated to prevent its progression.

Beetroot produces a red discolouration of the urine.

Reference:

1. Al Montasir, A., & Al Mustaque, A. (2013). Purple urine bag syndrome. Journal of family medicine
and primary care, 2(1), 104–105. https://doi.org/10.4103/2249-4863.109970

2. Urine tract infection Correct answer selected


A 76-year-old female presents to MAU with a 4-day history of deteriorating weakness in the legs and
paraesthesiae.

Two weeks previously, she had received the AZ COVID-19 vaccine. In her past medical history, she
had a 5-year history of hypertension for which she was treated with losartan and bendrofluazide.

She also had a diagnosis of asthma and had been prescribed a steroid inhaler which she used once
daily.

On examination, she had a temperature of 36.7oC, a pulse of 88 bpm, a blood pressure of 128/82
mmHg and a respiratory rate of 18/min. She had weakness of the lower limbs 3-4/5, reduced tone,
absent plantar, ankle and knee reflexes but normal sensation to pinprick and light touch.

There was also weakness of 4/5 of the upper limbs and loss of biceps and triceps reflexes. Co-
ordination appeared intact and the cranial nerves were normal.

Investigations revealed the following:

Investigation Result Normal range


FBC Normal
Sodium 131 mmol/L 134 - 144 mmol/L
Potassium 3.2 mmol/L 3.5 - 5 mmol/L
Urea 8.8 mmol/L 3 - 8 mmol/L
CRP 6 mg/L <5
Glucose 6.1 mmol/L 3.5 - 5.5 mmol/L
CXR Normal
CT Head Normal

LP results reveal:

Investigation Result Normal range


Glucose Normal
Protein 0.9 g/L <0.4
Lymphocytes 1.1 0-5

Which of the following is the likely diagnosis?

1. Autoimmune polyendocrine syndrome type 1


2. Guillain-Barre Syndrome
3. Hypokalaemic periodic paralysis
4. Subacute combined degeneration of the cord
5. Transverse myelitis
Explanation:

The likely diagnosis with the dissociated ascending motor polyneuropathy with no features of
sensory loss is Guillain-Barre, likely due to the covid-19 in the absence of any other obvious cause.
This has been rarely reported but the features here are typical of GBS, as is the LP findings with a
dissociated elevation of protein.

Hypokalaemic periodic paralysis is unlikely as the history is very different with a much shorter
episodic weakness.

The mild hypokalaemia with hyponatraemia is likely due to the bendrofluazide.

Reference:

1. McKean N, Chircop C. Guillain-Barré syndrome after COVID-19 vaccinationBMJ Case Reports CP


2021;14:e244125.

2. Guillain-Barre Syndrome Correct answer selected

An 82-year-old man with metastatic prostate cancer has been admitted to a specialist palliative care
inpatient unit.

He lives alone, and the request had been for a "social admission", as he had become acutely unable to
look after himself.

On admission, the nurse has found him to be fully alert but restless and distracted.

He was able to recall his date of birth and the current year but he was not able to recall the place or
his age.

When asked to count the months of the year backwards he was only able to state ‘December’ and
then ‘November’ but nothing further beyond that.

The nurse has therefore completed the ‘4AT’ (4 A’s Test) screening tool in light of the patient’s
confusion.
What would the ‘4AT’ score indicate for this patient?

1. Definite hypoactive delirium


2. Likely drug induced delirium
3. Probable delirium
4. Probable dementia rather than delirium
5. Unlikely delirium or cognitive impairment

Explanation:

The 4AT (4 A’s Test) is a screening tool for initial assessment of cognitive impairment and delirium.
It is one of the tools recommended by NICE (National Institute for Health and Care Excellence) and
SIGN (Scottish Intercollegiate Guidelines Network) for screening for delirium.

It is based on 4 factors: alertness, abbreviated mental test-4 (recall of age, date of birth, place/name
of the building, year); attention; acute change or fluctuating course.

A score of 4 or more, as in this case, suggests delirium.

The tool itself screens for probable delirium but does not sub-classify the type of delirium e.g.
hyperactive or hypoactive, or distinguish delirium from underlying cognitive impairment per se (this
may be established, for example, from the DSM-V criteria Diagnostic and Statistical Manual of
Mental Disorders).

Reference:

1. NICE (2021).. How should I assess a person with suspected delirium?


2. SIGN (2019). Risk reduction and management of delirium: a national clinical guideline.
3. The 4A Test: screening instrument for cognitive impairment and delirium 4AT.

4. Probable delirium Correct answer

A 67-year-old woman presented to the clinic with intermittent and recurrent cough, dyspnoea,
audible wheezing, and a low-grade fever. She was diagnosed as having sarcoidosis and was put on
steroids a few years back.

Her past medical history also consisted of hypertension, asthma since childhood, deep vein
thrombosis, and atrial fibrillation. She is a smoker and has been smoking 20 cigarettes a day for more
than 50 years.

On examination, her respiratory rate was 18 / minute with an oxygen saturation of 95%. Her blood
pressure was 100/70 mmHg, her jugular venous pressure was not elevated, and her heart rate was
92/minute, irregularly irregular with normal heart sounds. Her INR was 3.2 from the previous day.

Chest examination revealed a palpable rhonchus, a localised wheeze on forced expiration, and
localised prolonged inspiratory time in the right mammary and inframammary area.

A chest x-ray showed bilateral hilar lymphadenopathy.

A bronchoscopy was performed and no intra-bronchial tumour or foreign body was visualised.

What is the most likely diagnosis?

1. Acute attack of bronchial asthma


2. Acute bronchitis
3. Pancoast's tumour
4. Pulmonary thromboembolism
5. Right middle lobe syndrome
Explanation:

This is a case of an obstructive variety of middle lobe syndrome, caused by extrinsic compression of
middle lobe bronchus by the enlarged hilar lymph nodes secondary to sarcoidosis.

A persistent collapse of the middle lobe of the right lung is a hallmark of middle lobe syndrome. It
has an impact on people of all ages and is more common in females. It comes in two primary
varieties: obstructive and non-obstructive.

The obstructive variety is most frequently caused by malignancy.

The non-obstructive form is linked to bronchiectasis and inflammatory diseases. Treatment is always
focused on the underlying issue.

Pulmonary embolism is unlikely as the patient has no history of chest pain and her INR is within the
therapeutic range.

A diagnosis of acute bronchitis cannot explain her examination findings.

Pancoast's tumour involves the apical part of the lung and it causes compression of the sympathetic
trunk leading to Horner's syndrome.

Localised chest findings would not suggest an acute attack of bronchial asthma.

Reference:

1. Gudbjartsson, T., & Gudmundsson, G. (2012). Middle lobe syndrome: a review of


clinicopathological features, diagnosis and treatment. Respiration; international review of thoracic
diseases, 84(1), 80–86.

2. Right middle lobe syndrome Correct answer selected


An 84-year-old bedridden woman was brought to the ER with vomiting, acute abdominal pain, and
distension.

She has a long history of constipation and has been on a high-fibre diet.

In addition, she underwent an open laparotomy 15 years ago for a perforated gastric ulcer.

Her medications include omeprazole 20 mg/day, amlodipine 5 mg/day, and vitamin D 1000 I.U/day.

Her laboratory investigations are as follows:

Investigation Result Normal range


Haemoglobin 13.0 g/dl 11.5-16.6 g/dl
Total white cell count 7 x 109/L 4 -11 7 x 109/L
Serum urea 7 mmol/L 2.5-7.5 mmol/L
Creatinine 120 umol/L 60-110 umol/L
Serum sodium 132 mmol/L 135-145 mmol/L
Serum potassium 3.3 mmol/L 3.5-4.5 mmol/L
An x-ray of his abdomen was done and revealed the following:

What would be the best initial intervention for this patient?

1. Conservative management only


2. Flatus tube
3. Laparotomy
4. Nasogastric tube
5. Senna
Explanation:

In this case, the acute onset of symptoms and findings on abdominal radiograph suggest volvulus as
a diagnosis.

In addition, risk factors such as the long history of constipation, consumption of a high fibre diet, and
abdominal adhesions brought on by perforated viscus and previous exploratory laparotomy, provide
some additional clues to the diagnosis.

Risk factors include Intestinal malrotation, a congenital defect, an enlarged colon, Hirschsprung
disease, pregnancy, and abdominal adhesions.

A high-fiber diet and chronic constipation may also raise the risk.

The sigmoid colon and cecum are most frequently affected. Medical imaging, such as plain x-rays,
GI series, or CT scans, are frequently used for diagnosis.

A volvulus occurs when the mesentery that supports the loop of the intestine coils over itself,
obstructing the colon. The symptoms are usually acute and include abdominal pain, bloating,
vomiting, constipation, and bloody stool.

A section of the intestine may become ischaemic due to blockage of blood supply.

The best initial intervention would be the insertion of a flatus tube under sigmoidoscopic guidance or
with a barium enema.

The indications of laparotomy would be failed decompression, fever, leucocytosis and clinical
picture suggestive of intestinal ischaemia, perforation or peritonitis.

The nasogastric tube would give some symptom relief but would not deal with volvulus.

With this patient’s history of constipation, which is now complicated with volvulus, it is unlikely that
senna will be of any benefit.

Reference:

6. Le CK, Nahirniak P, Anand S, et al. Volvulus. [Updated 2022 Apr 30]. In: StatPearls [Internet].
Treasure Island (FL): StatPearls Publishing; 2022 Jan-.
7. Thornton, S. Sigmoid and Cecal Volvulus, Medscape, 2020.

8.
9. Flatus tube Correct answer selected
Subspecialty topics

A 75-year-old woman presents to the MAU on Sunday with recurrent episodes of right-hand
numbness and slurred speech which have occurred on approximately 10 occasions over the last 2
weeks.

The episodes are all very similar in nature, each lasting 10 to 40 minutes with complete resolution of
the weakness and numbness. They tend to occur at any time of day and come on quite suddenly.

In her past history, she has been treated for hypertension with amlodipine and also takes simvastatin.
She had a right hip replacement 3 years ago which was uneventful.

On examination, she feels that her symptoms have now entirely resolved and there are no
neurological deficits on examination. Blood pressure is 136/84 mmHg.

CT head scan is reported as normal.

Which of the following is the most appropriate approach to the management of this patient?

1. Admit and request specialist review


2. Admit, give antiplatelet therapy and specialist review
3. Discharge with no further follow up required
4. Discharge without antiplatelets and refer to TIA clinic
5. Give aspirin, discharge and organise TIA clinic referral

1.

Explanation:

This patient presents with transient loss of function which is typical of TIAs within the carotid
territory.

This is crescendo TLOF where 2 or more events occur within 1 week, signifying a bad prognosis.

The patient requires admission as there is a significant risk of a full-blown stroke, and also requires
appropriate specialist stroke review with risk factor assessment including carotid dopplers and echo.

Reference:

2. Nice: management for people presenting acutely with atrial fibrillation


3. Admit, give antiplatelet therapy and specialist review Correct answer selected

An 80-year-old woman is admitted following an hour history of weakness and loss of speech
affecting the left side of the body.

By the time she is admitted the weakness has resolved and her speech is improving.

She has a history of hypertension for which she takes amlodipine 10mg daily. She also takes
atorvastatin 10mg daily. Otherwise, she is well with no other history of note.

On examination, her blood pressure is 144/80 mmHg and her pulse is 68 bpm regular.

Which of the following most accurately suggests the approximate risk of this patient having a stroke
within the next week?

1. <1%
2. 5%
3. 10%
4. 25%
5. 40%
1.

Explanation:

The important issue here is to calculate this patient’s ABCD2 score. The ABCD2 score is a risk
assessment tool designed to stratify TIAs according to future risk of stroke.

A refers to age >60 =1

B refers to blood pressure with either systolic or diastolic >140/90 = 1

C – Clinical Features: Unilateral weakness with or without speech impairment =2. Speech
impairment =1

D – Duration: 10-59mins =1. >60 mins = 2

2 – Type 2 diabetes = 1

In this case, the patient scores 6 based on age, clinical features and duration. This suggests quite a
high risk of future stroke of about 12% at 7 days, with guidance suggesting the patient be admitted,
investigated and treated.

Reference:

2. Fothergill, A. Christianson, T, JH. Brown, R. D. Jr. Rabinstein, A, A. Validation and Refinement of


the ABCD2 Score: a population-based analysis. Stroke. 2009 Aug; 40(8): 2669–2673.
3. 10% Correct answer

A 75-year-old female was assessed in A&E having been brought into hospital by her husband in a
confused state.

They had been walking back from the shops and he stated that she started constantly asking where
they were and what they had just been doing. This continued in the emergency department but by the
evening post take ward round she was back to her normal self, although she couldn’t recall the events
of the day.

The husband who was with her the whole time did not observe any limb weakness or facial
asymmetry. Observations and bloods were unremarkable.

The most likely diagnosis is?

1. First presentation of dementia


2. Hypoglycemic event
3. Temporal lobe epilepsy
4. Transient ischaemic attack
5. Transient global amnesia
Explanation:

The nature of the symptoms – constant questions, then rapid full recovery with no memory of the
event – is strongly suggestive of Transient global amnesia.

Reference:

1. Spiegel, D, R. Smith, J. Wade, R, R. Cherukuru, N. Et al. Transient global amnesia: current


perspectives. Neuropsychiatr Dis Treat. 2017; 13: 2691–2703, doi: 10.2147/NDT.S130710.

2. Transient global amnesia Correct answer selected


A 70-year-old male is admitted complaining of involuntary movements on the left side of the body.
These symptoms have deteriorated over the last 3 weeks.

He has a 10-year history of type-2 diabetes for which he takes ramipril 10 mg/d, metformin 1g bd,
sitagliptin 100 mg/d and atorvastatin 20 mg daily.

On examination, his pulse is 80 bpm reg, blood pressure 136/88 mmHg. He has writhing movements
on the left limbs with some occasional left arm and leg flinging movements. Neurological
examination is otherwise normal, with flexor plantars and normal reflexes.

Investigations:

Investigation Result Normal range


FBC Normal
U+Es Normal
Glucose 25.9 mmol/L 3.5-5.5 mmol/L
HbA1c 88 mmol/mol <48 (10.2%))
CT Head Report Evidence of small vessel disease.
Nil acute
Blood ketones 0.5 mmol/L <0.6 mmol/L

Which of the following is the likely diagnosis?

1. Diabetic hemi-chorea hemi-ballism syndrome


2. Drug-induced chorea
3. Haemochromatosis
4. Stroke
5. Wilson’s disease

Explanation:
This is a typical history of the rare but well-described non-ketotic
hyperglycaemic (diabetic) hemi-chorea hemi-ballism syndrome characterised
by continuous, irregular, and involuntary jerky movements of one side of the
body, often the result of a focal lesion of the contralateral basal ganglia.

The condition is associated with poor glycaemic control, as in this case and,
improved glycaemic control is associated with improvement and resolution of
symptoms.

The pathophysiology of the disorder is associated with relative ischaemia,


possibly a result of hyperviscosity within the striatum.

A CT scan may be normal or reveal densities in the putamen/arcuate nucleus,


so an MRI is often required and demonstrates T1 hyperintensity in the
contralateral putamen.

Reference:

Al Montasir, A. and Sadik, M.H., 2013. Hemichorea-hemiballism in a nonketotic


diabetic patient. Journal of family medicine and primary care, 2(3), p.296.

Diabetic hemi-chorea hemi-ballism syndrome Correct answer selected

A 77-year-old woman attends A&E on a Saturday after suddenly developing dysarthria, diplopia,
numbness and weakness of the left hand which lasted approximately 2 hours.

The symptoms have now resolved but she describes the symptoms coming on suddenly, over
minutes.
In her past medical history, she is treated for hypertension, hyperlipidaemia and has stable
intermittent claudication not requiring any surgical intervention.

She takes amlodipine, simvastatin and aspirin. She is a smoker of 5 cigarettes daily.

On examination, her pulse is 88 bpm irregular, BP 150/85 mmHg but neurological and
cardiovascular examination is otherwise normal. There is no carotid bruit.

CT scan is normal.

Investigations show normal routine blood tests and an ECG reveals atrial fibrillation.

Which of the following is the most appropriate approach to the management of this patient?

1. Admit and arrange urgent specialist review


2. Admit, arrange urgent specialist review, echo and MRI as inpatient
3. Discharge with risk factor modification and review in TIA clinic
4. Discharge with risk factor modification, anticoagulation and review in TIA clinic
5. Discharge with risk factor modification and anticoagulation following outpatient MRI with TIA
clinic

Explanation:

This patient has presented with a typical TIA and requires risk factor assessment and
modification, particularly blood pressure review and smoking cessation, together with
anticoagulation – NOACS or warfarin are acceptable. The patient need not be admitted to
facilitate this. There is a 10% risk of completed stroke following TLOF and this can be
reduced by up to 80% with risk factor modification.
Reference:

Nice: Stroke and TIA

Discharge with risk factor modification, anticoagulation and review in TIA clinic Correct
answer

A 70-year-old man presents after a transient ischaemic attack involving weakness of the left arm and
lasting approximately 2 hours.

He has a past medical history of diabetes for which he takes metformin 1g bd, together with
simvastatin 40mg daily and ramipril 10mg daily.

On presentation, his weakness has now entirely resolved.

He has a pulse of 80 bpm regular and a blood pressure of 148/92 mmHg.

Which of the following denotes this man’s ABCD2 score?

1. 1
2. 3
3. 5
4. 6
5. 7

The ABCD2 score is a risk assessment tool designed to stratify TIAs according to future risk of
stroke.

A refers to age >60 =1

B refers to blood pressure with either systolic or diastolic >140/90 = 1

C – Clinical Features: Unilateral weakness with or without speech impairment =2. Speech
impairment =1

D – Duration: 10-59mins =1. >60 mins = 2

2 – Type 2 diabetes = 1

In our example, the age =1, blood pressure =1, Clinical features =2, duration =2 and he has diabetes
=1

The score is therefore 7 which is the maximal score indicating a high risk of a completed stroke 2, 7,
30 and 90 days after a TIA.

With an ABCD2 score of 6-7 there is an approximate 10% risk of stroke at 7 days and the
recommendation, in this case, is one of admission with observation.
Reference:

1. Fothergill, A. Christianson, T, JH. Brown, R. D. Jr. Rabinstein, A, A. Validation and Refinement of


the ABCD2 Score: a population-based analysis. Stroke. 2009 Aug; 40(8): 2669–2673.

7 Correct answer

A patient is admitted to the medical admissions unit. The patient is groaning, opens his eyes to
painful stimuli and tries to pull his arm away when pressure is applied to a nail bed.

Which of the following is the correct Glasgow Coma Scale score in this man’s case?

1. 4
2. 6
3. 8
4. 10
5. 13
Explanation:

GCS is based on the following scores:

1 2 3 4 5 6
Eyes Not open Open to pain Opens to voice Spontaneous N/A N/A
Verb No sounds Incomprehen Inappropriate words Confused disorientated Orientated N/A
al sible sounds converses
normally
Mot No Extends to Abnormal flexion to Flexion/withdrawal to Localises to pain Obeys
or movement pain pain pain commands
Therefore, in this man’s case, he opens his eyes to painful stimuli (E2), he makes incomprehensible
sounds (V2) and withdraws to pain (M4). Therefore he has an overall GCS of 8. It is very important
to be able to calculate the GCS accurately as it has a prognostic value.

Reference:

1. Glasgow Coma Scale


A 71-year-old woman is referred from primary care with various aches and pains. She has no past
history of note and takes no medication.

The GP encloses the following biochemistry:

Investigation Result Normal range


U+Es Normal
Calcium 2.36 mmol/L 2.2 - 2.6 mmol/L
Phosphate 0.8 mmol/L 0.7 - 1.4 mmol/L
PTH 18.2 nmol/L 3 - 6 nmol/L

Which of the following is the likely diagnosis?

1. Familial hypercalciuric hypercalcaemia


2. Primary hyperparathyroidism
3. Pseudohypoparathyroidism
4. Secondary hyperparathyroidism
5. Tertiary hyperparathyroidism
1.

Explanation:

This patient has a low calcium and elevated PTH concentration. The most likely explanation in a
woman of this age is vitamin D deficiency and hence secondary hyperparathyroidism. Vitamin D
deficiency is common and some studies suggest that it is present to a variable extent in up to 30% of
the general population.

Reference:

2. Amrein, K. Scherkl, M. Hoffmann, M. Neuwersch-Sommeregger, S. Et al. Vitamin D deficiency 2.0:


an update on the current status worldwide. European Journal of Clinical Nutrition volume 74,
pages1498–1513 (2020).

Secondary hyperparathyroidism Correct answer selected

A 72-year-old gentleman presents with a painful lower leg.

Examination of the left leg reveals thickening and bowing of the tibia which is slightly warmer to
touch over this than the right.
His investigations reveal an alkaline phosphatase of 450 iu/l (50-110), ALT of 22 iu/l (10-50) and a
calcium of 2.3 mmol/l (2.2-2.6).

Which of the following is the most appropriate investigation to establish the suspected diagnosis?

1. Bone biopsy
2. Bone densitometry
3. Bone scan
4. MRI tibia
5. X-ray of tibia

Explanation:

As about 90% of patients with Paget's disease are usually asymptomatic, the diagnosis is often
incidental based on radiological findings. In the early stages, the lesions seen are mostly osteolytic,
and in tubular long bones begin in the metaphysic and migrate down the shaft in a V-shaped 'cutting
cone'.
As the disease progresses, mixed lytic and sclerotic lesions are seen including trabecular thickening
and increased density. In later stages, most of the lesions are sclerotic.

Although isotope bone scanning is a useful investigation to assess the extent of disease and bone
biopsy it is used only occasionally to confirm the diagnosis, with X-ray of afflicted bone often the
most useful initial diagnostic test.

Reference:

1. Tuck, S, P. Layfield, R. Walker, J. Mekkayil, B. Francis, R. Adult Paget’s disease of bone: a review.
Rheumatology, Volume 56, Issue 12, December 2017, Pages 2050–2059,
https://doi.org/10.1093/rheumatology/kew430

2. X-ray of tibia Correct answer selected

The following results were obtained on a DEXA scan of the hip of a 55 year old female who was
concerned regarding her risk of osteoporosis since there was a strong family history of fragility
fractures.

BMD T score Z score


Neck 0.74 -1.5 -0.29
Trochanteric 0.73 +0.38 +1.1
Intertrochanteric 0.91 -1.29 -0.68
Total 0.83 -0.95 -0.11
Ward's triangle 0.50 -2.0 -0.21

Which of the following is the correct interpretation of this DEXA scan?

1. Normal bone mineral density


2. Osteomalacia
3. Osteopenia
4. Osteopetrosis
5. Osteoporosis

1.

Explanation:

A DEXA scan measures bone mineral density (BMD) at various sites, typically the lumbar spine and
hip are reported. The T score is the number of standard deviations of the patient's BMD and that of a
'young' control.

The Z score is the number of standard deviations between the BMD of the subject and that expected
for an aged and sex-matched control.

T scores greater than -1 are normal. T scores between -1 to -2.5 reflect osteopenia and T scores less
than -2.5 reflect osteoporosis.

This DEXA report of the hip reveals T scores between +0.38 and -2.0 which indicate osteopenia at
Ward's triangle and the intertrochanteric region.

Reference:

2. NOGG: Clinical guideline for the prevention and treatment of osteoporosis, 2017.
3. Osteopenia Correct answer selected

A 56-year-old woman is referred as she has been concerned about osteoporosis. She went through
the menopause uneventfully at the age of 50 and has no particular problems of note. She has a strong
family history of osteoporosis. A DEXA scan that she had obtained privately reveals a T score of -
2.3 at the hip and -2.0 at the lumbar spine. Her investigations reveal:

Investigation Result Normal range


Calcium 2.36 mmol/l 2.2 - 2.6 mmol/L
Alkaline Phosphatase 88 iu/L 50 - 110 iu/L
PTH 8.6 pmol/L 3 - 5.5 pmol/L
Vitamin D 12.7 nmol/L 50 nmol/L
Which of the following would be the most appropriate treatment for this patient?

1. Calcium and vitamin D supplements


2. Continuous combined HRT
3. Raloxifene
4. Teriparatide
5. Weekly alendronate
1.

Explanation:

This patient has secondary hyperparathyroidism and vitamin D deficiency, with osteopenia as
evidenced by the DEXA T score between -1 and -2.5. In this scenario, the most appropriate initial
treatment would be vitamin D replacement/Calcium treatment which would be expected to improve
bone mineral density in the context of hypovitaminosis D. This can be reviewed with serial scanning.

Reference:

2. CKS guidance

3. Calcium and vitamin D supplements Correct answer selected


A 74-year-old lady with metastatic renal cell carcinoma is admitted with symptoms of general
malaise and nausea and is noted to have corrected calcium 3.45mmol/l (2.2 - 2.6). You prescribe an
intravenous bisphosphonate infusion after rehydration.

Which of the following best describes the mode of action of bisphosphonates?

1. They block the effect of PTH on the renal tubules


2. They inhibit osteoblastic activity
3. They inhibit osteclastic activity
4. They stimulate osteoblastic activity
5. They stimulate osteoclastic activity
Explanation:

Bisphosphonates act as pyrophosphate analogues and inhibit osteoclastic activity in bone mineral and
induce apoptosis. They have no effect on PTJH or parathyroid- related protein.

Reference:

http://www.uptodate.com/contents/treatment-of-hypercalcemia
1. They inhibit osteclastic activity Correct answer selected

Six weeks after her hip fracture, an 80-year-old lady presents to your follow-up clinic.

She reports that she is mobilising well using a stick and her pain is well controlled.

She is taking alendronic acid, a calcium/vitamin D supplement and paracetamol that was prescribed
on discharge. She is otherwise fit and well.

She reports that things are going well, other than she finds she gets an upset stomach after taking the
alendronic acid.

She tells about her friend who has an injectable treatment which is given every six months and she
wonders if she would be eligible to try it.

What would you advise this patient?

1. She could be moved immediately to intravenous zoledronic acid


2. She could be moved immediately to subcutaneous denosumab
3. She should continue with the alendronic acid
4. She should discontinue antiresorptive treatment
5. She should try an alternative bisphosphonate treatment

Explanation:

This patient is clearly referring to denosumab, as it is given every 6 months. It is recommended for
secondary prevention of fractures when patients are unable to follow the instructions of alendronate
and either risedronate or etidronate, or have an intolerance or contraindication to those treatments.

Reference:

1. NICE: Denosumab for the prevention of osteoporotic fractures in postmenopausal women


Technology appraisal guidance [TA204]
2. She should try an alternative bisphosphonate treatment Correct answer selected

A 63-year-old man from North Africa presents to the geriatric clinic with recurrent falls over the past
6 months. He has a past history of rheumatoid arthritis and hypertension.

He denied any history of loss of consciousness, dizziness or vertigo and claimed that his falls are due
to loss of balance.

He has a recent history of a fractured neck of the femur after a trivial fall. His medications include
sulfasalazine, amlodipine, and recombinant human PTH.

Which of the following is a risk factor for osteoporosis in this patient?

1. Black ethnicity
2. Male sex
3. Recombinant human PTH (parathyroid hormone)
4. Rheumatoid arthritis
5. Sulfasalazine
1. Rheumatoid arthritis Correct answer selected

Explanation:

Rheumatoid arthritis (RA) is associated with osteoporosis due to active systemic inflammation,
immobilisation and the use of glucocorticoids. Osteoporosis occurs in two forms in RA: 1)
generalised bone loss with axial distribution including the spine, pelvis, hips, ribs and humerus, and
2) periarticular or localised bone loss in the proximity of the inflamed joints. According to several
studies, patients with RA have a higher rate of spinal or hip fractures than people with primary
osteoporosis.

Other risk factors include female sex, white ethnicity, lack of physical activity, alcohol /smoking,
hyperparathyroidism, prior fracture, vitamin D deficiency, and some medications such as prolonged
use of glucocorticoids.

Reference:

2. Christodoulou C, Cooper C. What is osteoporosis? Postgraduate Medical Journal 2003;79:133-138.


BMJ.
3. Pouresmaeili F, Kamalidehghan B, Kamarehei M, Goh YM. A comprehensive overview on
osteoporosis and its risk factors. Ther Clin Risk Manag. 2018 Nov 6;14:2029-2049.
A 65-year-old female has been commenced on denosumab for the treatment of osteoporosis.

Which of the following most appropriately describes the mechanism of action of denosumab?

1. Increases bone remodelling


2. Increases calcium reabsorption from the gut
3. Inhibits osteoclast function
4. Reduces osteoblast function
5. Reduces renal calcium excretion
Explanation:

Denosumab is a RANKL inhibitor thereby inhibiting osteoclast function and survival and hence
improving bone thickness.

Reference:

1. RANKL inhibitor

2. Inhibits osteoclast function Correct answer selected


You see an 82-year-old woman who has a history of frequent falls at home. She has a long history of
illnesses, including osteoarthritis, chronic obstructive pulmonary disease, angina, heart failure, and a
recent fractured left hip that was fixed with a dynamic hip screw.

She takes a Combivent inhaler, isosorbide mononitrate, lisinopril, aspirin, and lactulose, in addition
to other drugs.

On examination, she was found to be fragile and to be at constant risk of falling after an examination.

What measures should you take to reduce the patient's risk of further fractures?

1. Perform a DEXA scan before starting treatment


2. Prescribe calcium supplementation only
3. Prescribe calcium with vitamin D3
4. Prescribe risedronate weekly and calcium/vitamin D3 daily
5. Prescribe Strontium ranelate daily

Explanation:

According to the NHS guidelines, patients who have experienced fractures and are osteopenic (T
score of -1 to 2.5) should receive osteoporosis treatment.
Given her osteoporosis risk factors, it is almost certain that this patient is at least osteopenic. As she
had a fragility fracture, she would qualify for treatment.

Although a DEXA scan is the investigation of choice to make a diagnosis of osteoporosis, in this
case, it is neither necessary (because of their history of low impact fracture), nor appropriate in view
of her age and frailty. In addition, osteoarthritis can interfere with the scan result.

Calcium alone is not effective at reducing hip fractures.

Calcium with vitamin D3 helps in primary prevention but failed to show any benefit in the secondary
reduction of fractures.

Bisphosphonates have been shown to be effective at reducing hip fractures. Strontium is used for
patients who are unable to take bisphosphonates. In addition, it can cause thromboembolism and is
therefore not regarded as a first-line drug.

Reference:

1. NHS: Osteoporosis, 2019.


2. Whitaker Elam, R. E. Osteoporosis Treatment & Management, Medscape, 2021.

3. Prescribe risedronate weekly and calcium/vitamin D3 daily Correct answer selected

A 67-year-old lady with newly diagnosed osteoporosis re-presents to clinic with intolerance to
alendronate 70 mg weekly after 4 weeks of persistence. There is a history of previous fracture at the
left wrist when she fell off the fourth step of a ladder 4 years ago. She is also taking calcium and
vitamin D supplementation. She underwent menopause at age 40 and did not take hormone
replacement therapy. There is a family history of osteoporosis and hip fracture in her late mother.

Other secondary causes of osteoporosis have been excluded.

Her DEXA scan T scores are:

Investigation Result
AP Spine -3.9
Total Hip -3.3
Which treatment is appropriate as per NICE guidelines?

1. Intravenous zoledronic acid


2. Continue calcium and vitamin D supplementation alone
3. Denosumab
4. Ibandronate
5. Teriparatide

Explanation:

This question refers to the NICE guidelines for the secondary prevention of osteoporotic fractures
which relates only to treatments for the secondary prevention of fragility fractures in postmenopausal
women who have osteoporosis and have sustained a clinically apparent osteoporotic fragility
fracture.

Osteoporosis is defined by a T-score of -2.5 standard deviations (SD) or below on dual-energy X-ray
absorptiometry (DXA) scanning.

Alendronate is recommended as a first-line treatment option for the secondary prevention of


osteoporotic fragility fractures in postmenopausal women who are confirmed to have osteoporosis.

In this case, the patient has severe osteoporosis, a previous fracture and is intolerant of alendronate.

Therefore according to the guidance, the patient should initially try an alternative bisphosphonate
initially.
Then, if intolerant an alternative should be considered, such as teriparatide, denosumab or IV
bisphosphonates.

Reference:

1. NOGG: Clinical Guidelines 2017


2. SIGN: Management of osteoporosis and the prevention of fragility fractures, 2020.

3. Ibandronate Correct answer

A 67-year-old woman comes to see you at the metabolic bone clinic. She has had 2 vertebral
fractures in the past.

Her GP has given her alendronate and risedronate but she developed oesophagitis with both.

She was taking strontium ranelate which was withdrawn and now continues on raloxifene together
with calcium and vitamin D.

Her DEXA scan was repeated and shows that her T-score at the lumbar spine has fallen from -2.8 to
-3.5 and at the neck of the femur it is stable at -3.6.

Which of the following management options would you select for the management of this patient?

1. Continue current treatment unchanged


2. Start IV zolendronate
3. Start denosumab
4. Start ibandronate
5. Start teriparatide

Explanation:

NICE guidelines recommend the use of teriparatide for secondary osteoporosis in women who are
unable to take bisphosphonates or have a contraindication to, or are intolerant of, alendronate and
either risedronate or etidronate; or who have had an unsatisfactory response to treatment with
bisphosphonates and who are 65 years or older and have a T-score of - 4.0 SD or below, or a T-score
of - 3 SD or below plus more than two fractures, or who are aged 55-64 years and have a T-score of -
4 SD or below plus more than two fractures.
Reference:

1. https://www.nice.org.uk/guidance/ta161

Start teriparatide Correct answer

A 68-year-old Caucasian woman attends clinic with a fractured humerus following an innocuous fall.
She has no other risk factors. Her total hip T-score is -3.4 and spine T-score is -3.1.

Her FRAX score for hip fracture is 12% and for a major fracture is 30%.

Which of the following would be the most appropriate approach to the management of this patient?

1. Alendronate
2. Calcitriol
3. Calcium and vitamin D supplement
4. Denosumab
5. No treatment required
1.

Explanation:

No treatment is an inadvisable option because plentiful data show that patients with osteoporosis and
a previous fragility fracture will have a reduced risk of a new fracture if treated with pharmacological
therapy.

Calcitriol is incorrect. It has been approved by the FDA for use in renal dialysis patients and
hypoparathyroidism but there are no reliable data to demonstrate its use in the reduction of
osteoporotic fractures.

Alendronate is the correct answer. This agent is approved for the prevention and treatment of
postmenopausal osteoporosis, and to increase bone mass in men with osteoporosis, and for treatment
of osteoporosis in men and women taking glucocorticoids.

Reference:

2. Cosman, F. de Beur, S. J. Leboff, M, S. Lewiecki, E, M. Et al. Clinician’s Guide to Prevention and


Treatment of Osteoporosis, Osteoporos Int. 2014; 25(10): 2359–2381. Published online 2014 Aug
15. doi: 10.1007/s00198-014-2794-2.
Alendronate Correct answer selected

A 70 year old man with rheumatoid arthritis is on a long term steroid therapy (prednisolone-7.5 mg
per day). At what range of T scores on bone mineral densitometry (BMD) will you consider starting
him on bisphosphonates?

1. Start bisphosphonate irrespective of T score


2. T score < -1
3. T score < -1.5
4. T score < -2
5. T score< -2.5
Explanation:

This gentleman has two clinical risk factors (CRFs) for the development of osteoporosis- rheumatoid
arthritis and usage of steroids. Both of these are moderate risk factors for the development of
osteoporosis in contrast with smoking and alcohol intake which are considered weak CRFs.

A parental history of hip fracture and previous fragility are generally classified as strong risk factors
for osteoporosis development.

Guidelines suggest that with this long term dose at 7.5 mg/d, the patient should be commenced on a
bisphosphonate.

Reference:

1. NICE: Guidelines

2. Start bisphosphonate irrespective of T score Correct answer selected


A new experimental drug is being introduced for the treatment of osteoporosis. It has been shown to
act by inhibiting the production of RANKL factor. Which of the following cells are specifically
targeted by this class of agent?

1. Lining cells
2. Marrow cells
3. Osteoblasts
4. Osteoclasts
5. Osteocytes

1.
Explanation:

The communication between osteoblasts and osteoclasts seems to occur as a result of locally
produced signals which include osteoclast differentiation factor called RANK ligand (RANKL) and
osteoprotegerin (OPG). RANKL is a protein derived from osteoblasts, which binds to the RANK
receptor and activates NF-kB on the surface of osteoclast precursors leading to increased osteoclast
production.

OPG in contrast binds to RANKL and inhibits its interaction with RANK receptor. Therefore
inhibition of RANKL factor leads to downregulation of osteoclast production.

Reference:

2. Ono, T. Hayashi, M. Sasaki, F. Nakashima, T. RANKL biology: bone metabolism, the immune
system, and beyond. Inflammation and Regeneration volume 40, Article number: 2 (2020).

3. Osteoclasts Correct answer selected

A 77-year-old man is referred by the GP with polyuria and fatigue.

His calcium has been noted to be 3.1 mmol/l (2.2-2.6) with a PTH of 13.5 pmol/l (3.5-6).

He has a past history of ischaemic heart disease, having had previous stenting and associated NYHA
stage 3 heart failure.

Repeat investigations confirm the GP’s results.

He has a urea of 13.8 mmol/l (3-8), with a creatinine of 168 micromol/l (50-100).
He is adamantly opposed to surgical intervention and at the MDT, the surgeons feel that surgical risk
is rather high.

Which of the following would be the most appropriate approach to the management of this patient?

1. Intra-tumoural alcohol injection


2. IV zoledronate
3. Oral cinacalcet
4. Subcutaneous calcitonin
5. Weekly subcutaneous denosumab

1.

Explanation:

This patient is not considered suitable for surgical intervention due to CV risk, and with the patient
being opposed to surgery, the most appropriate approach to long term management would be the
calcimimetic, cinacalcet. It acts like calcium on the calcium-sensing receptor, thereby reducing PTH
levels.

Reference:

2. Cinacalcet
3. Oral cinacalcet Correct answer

A 72-year-old female patient attends to discuss her Osteoporosis (T score of -3.0) which was
diagnosed 16 months previously following a wrist fracture.

At that time she was commenced on once-weekly Alendronate plus calcium and Vitamin D
supplements.

She has not had other fractures, is not on oral steroid medication and does not suffer from any other
relevant medical condition.

The patient enquires as to whether she can have another DXA scan to see if the treatment is working.

What is the advice for repeating a DXA scan?

1. DXA is repeated 6 M after initiation of treatment


2. DXA scan is repeated every year
3. Review DXA every 5 years
4. Review DXA 3-5 years from previous scan if it is likely to influence management
5. Review DXA 5-7 years from previous scan
Explanation:

Review the DXA scan 3-5 years after previous scan if it is likely to influence management. DXA
scans are of limited value in assessing response to treatment.

This is because the changes in BMD in response to treatment are small and slow and similar in
magnitude to the error of the DXA measurement.

The decision as to whether and when to repeat a DXA scan will depend on the initial results and the
individual patient's circumstances.

Reference:

1. NOGG Guidelines 2017

2. Review DXA 3-5 years from previous scan if it is likely to influence management Correct answer
An 80-year-old male is reviewed after having been admitted to hospital with a fall. He has been
getting rather weaker over the last few years and does find some difficulty rising from a chair.

He lives alone since his wife died two years ago but has family support and carers twice daily.

He is treated with amlodipine 5mg daily, simvastatin 20mg daily and aspirin 75mg daily.

His blood pressure is 138/70 mmHg without a postural fall.

The only noticeable result amongst his investigations is the following:

25OH Vitamin D 8 nmol/l (> 50 nmol/L)

Which of the following clinical features would you expect to find in association with this?

1. Distal myopathy
2. Neuropathy
3. Pale conjunctivae
4. Proximal myopathy
5. Tetany on inflation of a blood pressure cuff
Explanation:

This patient has severe vitamin D deficiency as indicated by the 25OHD vitamin D concentration
less than 30 nmol/l. It has been demonstrated that below this level there is considerable impact on
bone mineralisation, plus osteomalacia can occur.

In conjunction with the effect on bone, vitamin D is associated with calcium influx into muscle, thus
vitamin D deficiency is associated with proximal myopathy.

This may contribute to the difficulty rising from the chair that this patient experiences and also the
issue of increasing falls.

Evidence suggests a reduction in falls following appropriate treatment.

BMJ 339:b3692 (2009) Fall prevention with supplemental and active forms of vitamin D: a meta-
analysis of randomised controlled trials.

Although we are not told about his calcium level, one would assume from the results that these are
normal. However, it would also be important to assess his calcium intake as the co-prescription of a
calcium supplement may be required if his nutritional intake is less than 800mg daily.

Vitamin D deficiency is very common in the elderly with virtually every institutionalised elderly
person being deficient.

The reason for this is two-fold. Firstly, the elderly, particularly the institutionalised person is not
exposed to sunlight to manufacture vitamin D and secondly the manufacture of vitamin D declines as
we age. Current NOS guidelines recommend that all people over the age of 65 take a daily
supplement containing 10mcg (400 IU) of vitamin D.

Reference:

1. Francis, Prof. R et al, (2013) Vitamin D and Bone Health: A Practical Clinical Guideline for Patient
Management. The National Osteoporosis Society.
2. Saran, T. Api, C. Kullaya, T. Kearkiat, P. Et al. Vitamin D supplement on prevention of fall and
fracture A Meta-analysis of Randomized Controlled Trials. Medicine: August 21, 2020 - Volume 99
- Issue 34 - p e21506.

3. Proximal myopathy Correct answer selected

A 65-year-old gentleman presents with deformity and some discomfort in his tibia. His bloods show
raised alkaline phosphatase with normal calcium and phosphate levels. His skull and spinal X rays
confirm the diagnosis of Paget's disease. Which one of the following is the most appropriate
treatment for his underlying condition?

1. IV pamidronate
2. IV zoledronate
3. Oral calcium carbonate and vitamin D
4. Oral risedronate
5. Oral cinacalcet

Explanation:

The prevalence of Paget's disease is about 2% over the age of 55 years in the UK. Only 1 out of 10
patients are symptomatic.

The indications for treating Paget's disease include the presence of symptoms including pain,
deformity, complications (such as deafness, spinal cord compression) and hypercalcaemia.

Bisphosphonates form the mainstay for the treatment.

The bisphosphonates licensed for treatment in Paget's disease include pamidronate, risedronate,
etidronate, tiludronic acid and zolendronate.
Out of these zolendronate has the longest duration of action, often only a single dose is required for
treatment purposes and has greater efficacy in reducing alkaline phosphatase vs oral risedronate.

Reference:

1. Tuck, S, P. Layfield, R. Walker, J. Mekkayil, B. Francis, R. Adult Paget's disease of bone: a review.
Rheumatology, Volume 56, Issue 12, December 2017, Pages 2050–2059,
https://doi.org/10.1093/rheumatology/kew430.

2. IV zoledronate Correct answer

A 71-year-old man presents with a two-month history of tiredness and lethargy. Six months ago he
was treated for paroxysmal atrial fibrillation with amiodarone 200 mg daily.

On examination, he appeared to be euthyroid with no goitre palpable.

Investigations revealed:

Investigation Result Normal range


Serum-free T4 26.2 pmol/L 9-23 pmol/L
Serum total T3 0.8 nmol/L 3-5.5 nmol/L
Serum TSH 9 mU/L 0.5-5 mU/L
Which of the following is the likely diagnosis?

1. Amiodarone-induced hypothyroidism
2. Amiodarone induced thyrotoxicosis
3. Hashimoto's hypothyroidism
4. Sick euthyroid syndrome
5. TSH secreting pituitary adenoma
Explanation:

The results show increased T4, low T3 but elevated TSH, indicating hypothyroidism. This suggests
amiodarone-induced hypothyroidism which inhibits the peripheral conversion of T4 to T3.

Reference:

1. ETA Guidelines

2. Amiodarone-induced hypothyroidism Correct answer selected


A 70-year-old woman presents with locally advanced cervical cancer.

The cancer was found to have penetrated the spinal cord and the patient is unable to move around in
bed. She experiences excruciating pain when being moved.

She describes the pain as sharp and shooting down the left leg. She is also experiencing paraesthesia
in both lower limbs.

She is currently on morphine sulphate tablets (MST) 120 mg bd and 40 mg Oramorph for incidental
pain.

What advice would you give to this patient?

1. Add dexamethasone 5 mg daily


2. Add pregabalin 150 mg bd
3. Add pregabalin 75 mg daily
4. Increase MST to 150 mg bd
5. Increase Oramorph to 60 mg prn
Explanation:

The patient is experiencing typical neuropathic pain and as such adjuvant pain therapy needs to be
started. Pregabalin works well on neuropathic pain but needs to be started slowly because some
people become too drowsy. The dose can then be titrated upwards as necessary up to a maximum
dose of 300 mg bd.

Reference:

1. SIGN (2013). SIGN 136: Management of chronic pain: a national clinical guideline.
2. Add pregabalin 75 mg daily Correct answer

You are asked to advise on the management of a elderly gentleman with end stage renal failure.

The renal consultant has explained to the family that the gentleman's prognosis is now likely hours to
a few days.

Which would be your first line subcutaneous PRN opioid of choice in this situation?

1. Fentanyl
2. Diamorphine
3. Hydromorphone
4. Morphine
5. Oxycodone
1.

Explanation:

Fentanyl is metabolised to an inactive metabolite and therefore active metabolites will not
accumulate with renal failure predisposing to opioid toxicity.

Alfentanil may be used for patients requiring higher strengths of opioids allowing a higher equivalent
dose to be given in a smaller volume and used in a portable syringe driver.

Oxycodone or diamorphine injections may be used in the short term if fentanyl is not available.

Reference:

2. Smith, H, S. Opioid Metabolism. Mayo Clin Proc. 2009 Jul; 84(7): 613–624. doi: 10.1016/S0025-
6196(11)60750-7.

3. Fentanyl Correct answer selected


A 70-year-old man with cancer of the prostate with bone metastases is currently taking morphine
sulphate tablets 60 mg twice daily.

He has normal liver and renal function. He is swallowing well. His pain is currently well controlled
except whilst bathing. You agree with the patient that a dose of morphine to cover bathing is needed.

How would you prescribe morphine for incident pain?

1. 10 mg Oramorph orally 20–30 minutes before bathing


2. 10 mg Oramorph subcutaneously 60 minutes before bathing
3. 20 mg morphine subcutaneously 20–30 minutes before bathing
4. 20 mg Oramorph orally 20–30 minutes before bathing
5. 20 mg Oramorph orally 60 minutes before bathing
Explanation:

For incident pain related to movement or activity, a patient on morphine needs to receive a dose
equivalent to 1/6th the total daily dose of morphine, 20–30 minutes before the anticipated movement.

Reference:

1. NICE (updated 2016). CKS: Palliative cancer care – pain.


2. NICE (2012, updated 2016). Clinical guideline [CG140]: palliative care for adults: strong opioids for
pain relief.

3. 20 mg Oramorph orally 20–30 minutes before bathing Correct answer

An 86-year-old woman known to have rectal cancer with liver metastases is admitted to the hospice
from home following referral from the community palliative care team for end-of-life care.

When she was asked about relevant family members on her admission, she had stated that she had no
family members.
She did have visitors to the hospice during her admission.

She died in the hospice 6 days later.

You complete a death certificate confirming her cause of death as metastatic colorectal carcinoma.

Who should register her death?

1. A GP who had not been involved in the patient’s care


2. District nurse
3. The deceased’s friend who she had put in charge of organising her funeral
4. The deceased’s GP
5. The deceased’s next-door neighbour

1.

Explanation:

The individuals that can register a death are:

2. relatives of the deceased,


3. someone present at the death,
4. an administrator from the hospital,
5. the person making arrangements with the funeral directors.
Deaths should be registered within 5 calendar days and can be registered at any register office,
although it is recommended to use the one serving the district in which the person died.

Reference:

6. https://www.gov.uk/after-a-death/register-the-death

7. The deceased’s friend who she had put in charge of organising her funeral Correct answer

In 2010, the General Medical Council published 'Treatment and care towards the end of life: good
practice in decision making' Please read the following statements carefully and decide which one is
correct.

1. If you are involved in decisions about treatment and care towards the end of life, you must be aware
of the Human Rights Act 1968.
2. It is unrealistic to work on the presumption that every adult patient has the capacity to make
decisions about their care and treatment.
3. Medical staff must assess their (patients') capacity to make each decision at a predetermined
appointment and this process need not be repeated.
4. The Independent Mental Capacity Advocate will have authority to make enquiries about the patient
and contribute to the decision by representing the patient's interests, but cannot make a decision on
behalf of the patient.
5. When consenting, if a patient does not want to know the potential consequences and risks of a
procedure, yet would still like it to be carried out, the procedure should not go ahead until a full
explanation of risks has been given.
Explanation:

Answer A should read Human Rights Act 1988. Answer B should state that "all healthcare
professionals should work on the presumption. etc etc",

Answer C should be "You must assess capacity to make each decision at the time it needs to be
made.",

Answer E: If a patient insists that he/she does not want even the basic information, you must explain
the potential consequences of carrying out an investigation or treatment if their consent may be open
to subsequent legal challenge. You must record the fact that the patient declined relevant
information. The investigation/procedure can however still go ahead.

Reference:

1. GMC: treatment and care towards the end of life

2. The Independent Mental Capacity Advocate will have authority to make enquiries about the patient
and contribute to the decision by representing the patient's interests, but cannot make a decision on
behalf of the patient. Correct answer
Mr Thomas recently died from pancreatic cancer and was a patient of the hospice. You receive a
telephone call from the executor of his will (a local solicitor). The executor needs information about
the patient’s place and date of death so he can deal with the person's estate. The death was expected
and the GP issued the death certificate but the practice is closed for teaching this afternoon.

The executor, a local solicitor, can bring a signed written request for the information to the office
(and photo identification to verify who he is) along with written proof that he is the executor of the
Will. What is the most appropriate response to this request?

1. Confirm that the patient was known to the hospice but refer the executor to the Coroner to obtain the
other information required
2. Confirm that the patient was known to the hospice but refer the executor to the registrar of birth and
deaths to obtain the information required
3. Give the executor a copy of the patient's hospice records
4. Give the executor only the place and date of death
5. Inform the executor that you cannot disclose confidential information about a patient who is
deceased
1. Give the executor only the place and date of death Correct answer selected

Explanation:

Give the executor only the place and date of death

The Access to Health Records Act regulates access to medical records of the deceased. You should
keep medical records confidential, in particular parts of the record that patients did not want
disclosed. However, you may need to share confidential information if asked by the patient's
representative - such as the executor of the will. In this situation, you should only share information
that is needed.

Reference:

2. Health Records Act. Office of Public Sector Information. London: Her Majesty's Stationery Office,
1990

3. Give the executor only the place and date of death Correct answer selected

Complicated grief is recognised according to the DSM classification by the following features:
1. Continued frequent episodes of "grief bursts and pangs" for more than 6 months after bereavement
2. Inability to accept the person has died for at least 4 months after bereavement
3. Persistent visual hallucinations involving the deceased in the first year after bereavement
4. Persistent symptoms of grief and impaired social functioning for at least 6 months following
bereavement
5. The existence of minimal observed distress in the first year after bereavement
Back

Complicated grief is recognised according to the DSM classification by the following features:

1. Continued frequent episodes of "grief bursts and pangs" for more than 6 months after bereavement
2. Inability to accept the person has died for at least 4 months after bereavement
3. Persistent visual hallucinations involving the deceased in the first year after bereavement
4. Persistent symptoms of grief and impaired social functioning for at least 6 months following
bereavement
5. The existence of minimal observed distress in the first year after bereavement
Back
1.
Explanation:

DSM classification proposes the following criteria for complicated grief:

Criterion A: Person has experienced the death of a significant other and response involves three of
the four following symptoms, experienced at least daily or to a marked degree:

2. Intrusive thoughts about the deceased.


3. Yearning for the deceased.
4. Searching for the deceased.
5. Excessive loneliness since the death.
Criterion B: In response to the death, four of the eight following symptoms are experienced at least
daily or to a marked degree:

6. Purposelessness or feelings of futility about the future.


7. Subjective sense of numbness, detachment, or absence of emotional responsiveness.
8. Difficulty acknowledging the death (e.g., disbelief).
9. Feeling that life is empty or meaningless.
10. Feeling that part of oneself has died.
11. Shattered worldview (e.g., lost sense of security, trust, control).
12. Assumption of symptoms or harmful behaviours of, or related to, the deceased person.
13. Excessive irritability, bitterness, or anger related to the death.
Criterion C: The disturbance (symptoms listed) must endure for at least 6 months.

Criterion D: The disturbance causes clinically significant impairment in social, occupational, or other
important areas of functioning.

These criteria have not been formally adopted, and thus there is no formal diagnostic category for
prolonged grief disorders in the DSM. However, these criteria help in specifying symptoms, the
severity of symptoms, and how to distinguish complicated grief from normal grief. Not all health
care professionals agree that the duration of "at least 6 months" is the most accurate number,
suggesting that the time period may be too short and that 6 months to 2 years may be more accurate.

Grief bursts and pangs may continue to occur up to 2 years after bereavement but in normal grief will
be lessening in intensity and frequency. Visual hallucinations are frequently experienced in the early
stages of bereavement.

Reference:

14. NIH: Grief, Bereavement, and Coping With Loss (PDQ®)–Health Professional Version.

15. Persistent symptoms of grief and impaired social functioning for at least 6 months following
bereavement Correct answer selected
An 86-year-old lady with metastatic endometrial cancer is being cared for on the medical ward. Her
medical team believe that she is likely to be in the last days of life and have asked for your input as
her family have been asking about whether she would benefit from a fluid drip.

According to the NICE Clinical Guideline 31 on the care of the dying patient, what approach to the
use of assisted hydration should be taken?

16. Clinically assisted hydration at the end of life should be offered to every patient as it will prolong
life
17. Clinically assisted hydration should not be offered as it is likely to cause more harm than benefit
18. Clinical assisted hydration should not be offered as it will likely hasten death
19. Clinical assisted hydration may relieve distressing symptoms but may also cause other problems
20. The risks or benefits of clinically assisted hydration do not need to be discussed
21.
Explanation:

NICE guidelines recommend the use of teriparatide for secondary osteoporosis in women who are
unable to take bisphosphonates or have a contraindication to, or are intolerant of, alendronate and
either risedronate or etidronate; or who have had an unsatisfactory response to treatment with
bisphosphonates and who are 65 years or older and have a T-score of - 4.0 SD or below, or a T-score
of - 3 SD or below plus more than two fractures, or who are aged 55-64 years and have a T-score of -
4 SD or below plus more than two fractures.

1. Clinical assisted hydration may relieve distressing symptoms but may also cause other problems
Correct answer selected

Explanation:

NICE guidance on the management of the dying patient advises that the healthcare team should
discuss the risks and benefits of clinically assisted hydration with the dying person and those
important to them.

Further to that, for someone who is in the last days of life: clinically assisted hydration may relieve
distressing symptoms or signs related to dehydration, but may cause other problems.

It is uncertain if giving clinically assisted hydration will prolong life or extend the dying process and
it is uncertain if not giving clinically assisted hydration will hasten death.

Reference:

2. NICE (2015) Care of dying adults in the last days of life. Clinical Guideline 31
According to the NICE guidelines for the care of dying adults in the last days of life (NG31) 2015,
how frequently should you review the dying person's hydration status, and review the possible need
for starting clinically assisted hydration, respecting the person's wishes and preferences?

1. Daily
2. Every nursing shift
3. Hourly
4. Once, at the point when medication is rationalised and the patient is thought to be close to death
5. Only when the next of kin express concerns
1.
Explanation:

According to the NICE Guideline for the Care of Dying Adults in the Last Days of Life (NG31)
2015, you should assess, preferably daily, the dying person's hydration status, and review the
possible need for starting clinically assisted hydration, respecting the person's wishes and
preferences.

Reference:

2. NICE guideline [NG31] 2015. Care of dying adults in the last days of life

3. Daily Correct answer

An 81-year-old lady with metastatic breast cancer has deteriorated, without reversible cause, and is
felt to be in the last hours to days of life.

Her usual medication is levothyroxine, aspirin and letrozole.

She is agitated and variably rousable. She has repeatedly said that she 'does not want to be sedated'.
She has now developed audible chest secretions.
On examination, she has audible secretions bilaterally which appear to be largely transmitted sounds
from her upper airways.

What medication is most appropriate to help reduce this symptom?

1. Dexamethasone
2. Furosemide
3. Glycopyrronium
4. Hyoscine hydrobromide
5. Salbutamol

1. Glycopyrronium Correct answer selected

Explanation:

The efficacy of hyoscine and glycopyrronium are similar in the management of 'death rattle'.

Glycopyrronium does not cross the blood brain barrier and therefore is not as associated with
sedation and delirium.

Dexamethasone may be useful in lymphangitis or large volume malignant thoracic disease (not
apparent in the case described, and would have a limited role in the last hours of life).
Salbutamol may have a role in bronchospasm from any cause, which is not suggested here.

Furosemide can be given subcutaneously at the end of life for symptom control, but the absence of
heart failure medication in the usual medication list suggests death rattle is more likely than
pulmonary oedema in this case.

Reference:

2. Twycross R., Wilcock A., Howard P. Palliative care formulary 2014 Fifth Edition

The junior doctor has just started their first job with you in the hospice in patient unit.

You have asked them to verify and certify death for a patient with advanced cancer who was an
expected death on your ward.

The junior doctor is requesting some guidance regarding identification of death. Which of the
following statements is true?

1. After the necessary observation period, there is no requirement to check for absence of papillary
responses to light or to check for an absent corneal reflex
2. In a hospital setting, the absence of cardiac function must be supplemented by one or more of the
following – asystole on ECG monitoring; absence of pulsatile flow using direct intra-arterial pressure
monitoring or/and absence of contractile activity using echocardiography
3. The deceased should be observed by the person responsible for confirming the death for a minimum
of two minutes to establish that irreversible cardio-respiratory arrest has occurred
4. The deceased should be observed by the person responsible for confirming the death for a minimum
of five minutes to establish that irreversible cardio-respiratory arrest has occurred
5. The absence of cardiac function can be confirmed by absence of a radial pulse on palpation
Explanation:

Please refer to the Academy of Royal Colleges guidance 'A code of practice for the Diagnosis and
Confirmation of Death'.

Reference:

1. 'A code of practice for the Diagnosis and Confirmation of Death'.

2. The deceased should be observed by the person responsible for confirming the death for a minimum
of five minutes to establish that irreversible cardio-respiratory arrest has occurred Correct answer
selected
You are caring for a patient who has no family or friends.

Which of the following circumstances would warrant the involvement of an Independent Mental
Capacity Advocate (IMCA)?

1. The patient has end-stage dementia and you and the team feel antibiotics if the patient develops a
chest infection inappropriate
2. The patient has cerebral palsy and lacks capacity and requires major surgery which has an associated
mortality risk
3. The patient has end-stage renal failure and wishes to stop dialysis
4. The patient with metastatic cancer of the lung and is now being moved into long term care after a
failed discharge to home and the patient realises this is the safest option for him
5. The patient is confused due to resistant hypercalcaemia from end-stage metastatic breast cancer and
the oncologists wish to stop chemotherapy
Explanation:

An IMCA should be sought when decisions have to be made about:

Serious medical treatment e.g. chemotherapy, major surgery, with-holding or withdrawing artificial
hydration and nutrition:

1. Where benefits and possible burdens are finely balanced


2. When a decision between choice of treatment is finely balanced
3. When the treatment proposed is likely to have serious consequences
4. Moving the patient into long term care
5. Adult protection (in England)
The role of the IMCA is to support the person for whom the decision is being made by obtaining as
much information as possible from that person and other sources about their wishes, feelings, beliefs
and values. The IMCA will not be the final decision maker, but the decision-maker must take into
account the information provided by the IMCA.

Serious medical treatment is that which involves:

6. giving new treatment


7. stopping treatment that has already started
8. withholding treatment that could be offered
Reference:

9. Mental Capacity Act (MCA)

10. The patient has cerebral palsy and lacks capacity and requires major surgery which has an associated
mortality risk Correct answer selected
A 76-year-old woman who is known to have a squamous cell carcinoma of the lung and brain
metastases is admitted to the hospice for 2 weeks of respite care while her family are on holiday.

She is a little sleepy and nursed mainly in bed due to a left hemiplegia and poor sitting balance, but
not confused and eating and drinking normally.

You perform routine blood tests which show the following:

Investigation Result Normal range


Sodium 120 mmol/L 133-145 mmol/L
Potassium 4.5 mmol/L 3.5-5.3mmol/L
Urea 8.1 mmol/L 2.5-7.8 mmol/L
Creatinine 63 mmol/L 70-120 mmol/L
She is taking the following medication:

Paracetamol 1g qds
Dexamethasone 2 mg od
Furosemide 20 mg od
Fluoxetine 20 mg od
Digoxin 0.125 mg od
Duovent inhaler bd
Which of the following should be next undertaken on this patient?

1. Give hypertonic saline


2. Measure ectopic ACTH
3. Measure random cortisol
4. Measure serum and urine osmolarity
5. Short synacthen test

1. Measure serum and urine osmolarity Correct answer

Explanation:
The likely cause for the hyponatraemia is SIADH which could be due to a multitude of factors
including drugs but also the tumour itself. The starting point would be the measurement of the urine
and serum osmolality.

You would expect a euvolaemic hyponatraemia. This is likely chronic hyponatraemia and, therefore,
hypertonic saline should not be used and fluid restriction employed.

Even though the patient is on furosemide, the starting point remains the above assessment.

Reference:

2. Yasir, M. Mechanic, O, J. Syndrome of Inappropriate Antidiuretic Hormone Secretion. StatPearls,


2021 Jan.

A 77-year-old lady is referred to the outpatient clinic with worsening lymphoedema of her right arm
due to radiotherapy post-mastectomy for breast cancer-causing discomfort.

She has a medical history of cardiac failure and osteoporosis.

She is frail and wheelchair-bound and finds coming to clinics regularly difficult.

You refer her to the lymphoedema service for review and she asks what treatment they are likely to
suggest.

What would you tell her is the main goal of lymphoedema management in her case?

1. To decrease arm swelling on average by 40% with twice-daily massage and compression
2. To educate the patient in active self-management of the lymphodema
3. To prevent the risk of cellulitis developing in the arm
4. To reduce the chance of a thrombosis developing in the arm
5. To restore and maintain arm function as much as possible

Explanation:

The goals are to keep the skin soft and infection-free and maintain limb function.

Complex decongestive Therapy (CDT) is the recognised treatment and includes:

1. Skincare
1. Exercise and maintenance of mobility
1. Manual lymphatic drainage
1. Compression
Whilst the other options are goals of treatment to a lesser extent not all patients achieve a significant
reduction in swelling but the other aspects of care remain important. This lady because of her general
condition may be well enough to participate in an active multidisciplinary programme which is
suitable for many and her concurrent cardiac failure may limit the use of lymphatic drainage.

Reference:

1. https://www.lympho.org/
2. Palliative adult network guidelines [BOOK].

3. To restore and maintain arm function as much as possible Correct answer

You are called to the oncology clinic to review an 88-year-old lady who was recently diagnosed with
large fungating breast cancer. The intent is to commence hormonal therapy but your opinion is
sought concerning the management of the malodorous wound.

The wound is inflamed and malodorous with widespread areas of purulent discharge. Despite this,
the lady denies pain and is not distressed by the lesion.

What would you suggest for wound management?

1. Oral metronidazole
2. Local radiotherapy
3. Surgical debridement
4. Topical metronidazole gel
5. Wound swab with activated charcoal dressings
Explanation:

The description of an inflamed wound with purulent discharge suggests a clinically infected wound
and since the patient is not systemically unwell or distressed by the wound, it would be appropriate
to take a swab and treat with wound cleansing and absorbant dressing to minimise odour (such as
activated charcoal or silver dressings) pending antibiotic sensitivity rather than start antibiotics
blindly.

The use of metronidazole gel although this is a moist wound is a possible option if sensitivities
confirm it would be appropriate.

The use of radiotherapy before the trial of hormones in an elderly lady would not be indicated and
surgical debridement is best for wounds with dry necrotic areas.

Reference:

1. Naylor W. (2002) Malignant wounds: aetiology and principles of management. Nursing Standard
16:52; 45-53

2. Wound swab with activated charcoal dressings Correct answer selected


You see an 80-year-old man who is severely agitated due to Alzheimer's disease. His symptoms have
deteriorated over the last 2 months and he has been admitted due to behavioural issues that distress
him. There are no features to suggest acute delirium. Your consultant asks you to prescribe an
appropriate antipsychotic to alleviate this man's distress. Which of the following antipsychotic
available on the hospital formulary do you prescribe?

1. Flupenthixol
2. Haloperidol
3. Quetiapine
4. Risperidone
5. Sulpiride

Explanation:
In the absence of delirium and in this scenario of acute agitation associated with dementia, guidance
would suggest the use of an atypical anti-psychotic such as risperidone and olanzapine. NICE
guidelines state

'Of all agents currently used for behavioural and psychological symptoms of dementia, atypical
antipsychotics have the strongest evidence base, although their benefits are moderate.

Any such benefits must be balanced against the risk of adverse events, including mortality. The
mortality findings among individual antipsychotic agents seem to be consistent with the tolerability
profile of individual atypical antipsychotics in the CATIE-AD trial, where olanzapine and
risperidone were more efficacious than either quetiapine or placebo, but quetiapine and placebo were
better tolerated (Kales et al, 2015)'

Reference:

1. Good article to review from Southern Health NHS

2. Risperidone Correct answer selected

A 89-year-old lady with inoperable rectal cancer is admitted to the hospice with severe symptoms of
tenesmus.

She has not passed formed stools for the last month but passes liquid stool several times a day when
the rectal sensation becomes worse.

A plain abdominal x-ray shows a non-specific gas pattern with no faecal loading.

She has a past medical history of atrial fibrillation, hypertension, stage 3 chronic renal disease and
type 2 diabetes mellitus and is on the following medication:
Digoxin 0.125mg od
Glicazide 80mg bd
Co-codamol 8/55 2 tablets qds
Timolol 0.25% eye drops to left eye bd

Which of the following would be the best medication to commence initially?

1. Amitriptyline
2. Codanthramer
3. Diclofenac
4. Hyoscine Butylbromide
5. Nifedipine

Explanation:

There are limited reports relating to the treatment of tenesmus but positive effects have been reported
with the use of nifedipine, antispasmodics, topical anaesthetics and neuropathic agents.

In this case, in view of the atrial fibrillation, impaired renal function and the presence of glaucoma a
trial of nifedipine (or another calcium antagonist like diltiazem) would be the best option.

Use of a stimulant laxative with probable partial rectal obstruction is likely to exacerbate symptoms.

Reference:

1. https://www.ruh.nhs.uk/For_Clinicians/departments_ruh/Palliative_Care/documents/palliative_care_
handbook.pdf
2. Nifedipine Correct answer selected

An 81-year-old lady with Parkinson's disease and locally advanced carcinoma of pancreas has been
admitted with a 4-day history of jaundice and severe pruritus.

Her jaundice is found to be secondary to biliary obstruction from the pancreatic tumour and multiple
attempts at biliary stenting have been unsuccessful.

Her overall condition has progressively deteriorated such that she is now bedbound but is still eating
and drinking and managing oral medication (selegiline). Her main symptom remains severe pruritus.

According to the Cochrane review of pharmacological interventions for pruritus in palliative care,
what would be the most appropriate drug to commence for this symptom?

1. Chlorpheniramine
2. Danazol
3. Paracetamol
4. Rifampicin
5. Sertraline
Explanation:

Whilst Danazol is used for pruritus secondary to cholestasis the effect is generally not seen for 5-10
days and given this patient's deteriorating condition is less appropriate.

Paracetamol should be avoided if possible in the context of severely deranged liver function.

Chlorpheniramine (Piriton) generally has no effect other than sedation in pruritus secondary to
cholestasis as the mechanism of itch is not secondary to mast cell degranulation.

Sertraline is used for pruritus secondary to cholestasis but would be contraindicated in this patient
because of the potential for a drug interaction with Selegiline producing serotonin syndrome or
neuroleptic malignant syndrome.

It was also felt to have limited evidence (1 trial of 12 participants) compared to rifampicin in this
context.

Reference:

1. Siemens W, Xander C, Meerpohl JJ, Buroh S, Antes G, Schwarzer G, Becker G. Pharmacological


interventions for pruritus in adult palliative care patients. Cochrane Database of Systematic Reviews
2016, Issue 11. No.: CD008320. DOI: 10.1002/14651858.CD008320.pub3

2. Rifampicin Correct answer selected


You see a 74-year-old lady in the outpatients, who has a diagnosis of Chronic Obstructive Pulmonary
Disease. She has been experiencing repeated episodes of exacerbations of her breathlessness
requiring admission.

On examination she is afebrile, heart rate 90 bpm regular, blood pressure 150/90 mmHg. Her JVP is
not raised, percussion of her chest is resonant and there is generally reduced breath sounds
throughout both lung fields. There is no peripheral oedema.

Current medication includes salbutamol 5mg nebulised qds, ipratropium bromide 500mcg nebulised
qds and maintenance prednisolone 10mg od. What would be the most suitable next step in her
management?

Furosemide 40mg od
Increase prednisolone to 40mg od
Lorazepam 0.5mg as required
Nebulised Morphine 5mg 4 hourly
Oral immediate release morphine 2.5mg 4 hourly
Explanation:

There are no signs of heart failure indicating that diuretics may be appropriate, or that increasing the
steroid medication is likely to improve symptoms. There is no evidence for nebulised morphine in
the treatment of breathlessness or indeed benzodiazepines, but these can be used in anxiety
associated with breathlessness.

See the linked systematic reviews on opioids and benzodiazepines for the symptomatic management
of breathlessness in advanced disease.

Reference:

1. Benzodiazepine
2. Opioid

3. Oral immediate release morphine 2.5mg 4 hourly Correct answer selected

An 86-year-old man with COPD has recently been referred to community palliative care. His last
spirometry showed an FEV1 of <30% predicted and his activities of daily living are limited by
breathlessness. But he can, for example, walk to his local shop to buy a newspaper (about 10-15
minutes walk at his pace) and stops there for 30 minutes before walking back home.
You want to refer him to the breathlessness management programme at the hospice, the referral form
asks for the MRC (Medical Research Council) grade of breathlessness.

How would you classify this patient's breathlessness on the MRC scale?

1. MRC grade 1 breathlessness


2. MRC grade 2 breathlessness
3. MRC grade 3 breathlessness
4. MRC grade 4 breathlessness
5. MRC grade 5 breathlessness

1.

Explanation:

The MRC breathlessness scale is used as a standardised marker of the severity of breathlessness and
its impact on activity.

It is often used as one of a number of triggers to indicate that a patient with non-malignant
respiratory disease may be appropriate for supportive and palliative care.

A patient who walks slower than contemporaries on level ground because of breathlessness, or has to
stop for breath when walking at own pace would be classified as MRC grade 3.
Reference:

2. MRC dyspnoea scale

3. MRC grade 3 breathlessness Correct answer selected

A GP has asked for advice about a 70-year-old man with known lung cancer and bone metastases
who has gone "off legs".

He has had generally reduced mobility for a week and has no back pain, his light touch sensation is
normal and there is no “definite” power deficit in his legs.

He has a tendency to constipation, which hasn’t changed, and no urinary symptoms.

Which clinical feature makes spinal cord compression least likely in this context?

1. Absence of back pain


2. Absence of bowel or bladder symptoms
3. Absence of power deficit
4. Intact light touch sensation
5. Symptoms at the lumbro-sacral level
1.

Explanation:

Back pain is the commonest early feature of impending malignant spinal cord compression (MSCC).
Whilst the absence does not mean you could exclude MSCC from clinical assessment alone, it makes
it unlikely because it is present in 95% of cases.

Pinprick sensation may be lost before light touch sensation in evolving cord compression and normal
light touch sensation alone is not therefore reassuring.

Power may appear intact initially in impending/evolving cord compression.

Bowel and bladder are late features and may not be present initially or depending on the level of
compression.

MSCC is less common at the lumbro-sacral level compared to thoracic, but lumbro-sacral is the
second most common site of MSCC.

Reference:

2. Watson M., Armstrong P., Back I., Gannon C., Sykes N., Palliative adult network guidelines (fourth
edition)
3. Absence of back pain Correct answer selected

A prescription that you completed for an in-patient due to go home tomorrow has been returned to
you by the pharmacist as you have not prescribed one of the drugs correctly in accordance with
prescription requirements for controlled drugs according to the Misuse of Drugs Regulations.

Which of the below drugs should have been prescribed as a "controlled drug"?

1. Co-codamol 30/500 2 tablets qds


2. Diazepam 5mg tds
3. Midazolam 2.5mg as required
4. Oramorph (10mg/5ml concentration) 2.5mg as required
5. Trazodone 150mg nocte
Explanation:

Midazolam (and temazepam) are schedule 3 controlled drugs. Other schedule 3 drugs are
buprenorphine and phenobarbital both of which should be prescribed as a CD. Oramorph 10mg/5ml
strength is a schedule 5 drug and as such does not have to be prescribed according to CD
requirements. However Sevredol and concentrated Oramorph (10mg/ml) are schedule 2 drugs as are
all other strong opioids.

Reference:

1. Royal Pharmaceutical Society. Medicines, Ethics and Practice. The Professional guide for
pharmacists. Edition 35. July 2011 [book].
2. NICE - Controlled drugs and drug dependence. Regulations and classification.

3. Midazolam 2.5mg as required Correct answer


An 83-year-old lady with locally metastatic breast cancer is known to the community palliative care
team.

Her liver capsule and bone pain have been well controlled with morphine sulphate tablets (MST)
45mg twice daily for several months but the district nurse has asked for a review because her pain
has escalated over the past week.

Although there has been no change in the character or nature of either pain, the patient reports using
Oramorph 15mg 5-6 times daily.

To assist with your medication review the patient shows you all of the tablets she currently takes out
of a kitchen cupboard and surprisingly there are more unused boxes of MST than you expected to
find.

The patient is supported by the district nurse who visits daily and until recently her neighbour has
done her weekly shopping and collected all her medication prescriptions from the local chemist (but
has gone away to look after an unwell relative).

What is the most appropriate next step in managing her pain?

1. Buphrenorphine patch 10mcg/h patch


2. Fentanyl 25mcg/hour patch
3. MST 90mg twice daily
4. Oxynorm (oxycodone) 25mg twice daily
5. Tramadol 100mg qds
Explanation:

There is a suspicion of poor compliance by the patient (intentional or unintentional) as there is more
unused MST than would be expected if all doses had been taken.

Any concerns or worries about medication should be explored to try and elicit any underlying
reasons why this may be the case.

The fact that pain has escalated and medication appears unused since the neighbour has been away,
suggests they had a pivotal role in prompting/overseeing the patient’s medication.

Medication compliance is a recognised indication for Fentanyl patches and a patch at an


approximately equivalent dose (which the district nurse could change every three days) would be the
most appropriate next step rather than increasing the opioid dose or changing the opioid used.

Note, a Fentanyl patch would not be appropriate if there had been escalating pain despite complete
compliance unless there was another indication for its use such as nausea and vomiting.

Reference:

1. Watson M, Lucas C, Hoy A, Back I and Armstrong P. Palliative Adult Network Guidelines. Third
Edition 2011.
Available at https://www.book.pallcare.info
2. Opiate Conversion Doses

3. Fentanyl 25mcg/hour patch Correct answer selected


4.

A 71-year-old gentleman was seen with a one-week history of a rash. This started on his forearms
and spread to the trunk and then the legs.

The rash was very pruritic in nature and was interfering with his sleep.

He had a chest infection 4 weeks prior to the onset of the rash and was treated with Augmentin for 7
days. He was known to have diabetes which was diet-controlled. He did not have any personal
history of skin disease. His son had mild asthma and eczema.
On examination, he was apyrexial, blood pressure 160/70 mmHg, pulse 85 bpm.

Full Blood Count, Urea and Electrolytes, C-reactive Protein and CXR were normal. Urinalysis
revealed a trace of protein.

His
rash
is as
show
n.

Whic
h of
the
follo
wing
is the
likely
diagn
osis?

1.
Conta
ct
allerg
ic
derm
atitis
2. Drug
reacti
on to
Aug
menti
n
3.
Gutta
te psoriasis
4. Henoch-Schönlein
5. Lichen planus
Explanation:

This patient presented with a generalised itchy rash. The photograph shows a lichenoid rash affecting
the forearm and is typical of lichen planus.

Lichen planus typically consists of polygonal, purple, flat top papules which when examined closely
show lacy white streaks, called Wickham’s striae. Wickham’ striae are also often found on the buccal
mucosa and are usually asymptomatic.

Lichen planus is intensely pruritic in nature. It is often self-limiting with an average duration of 18
months.

The treatment is for symptomatic relief with emollients and potent topical steroids.
In severe cases, there can be nail involvement, genital involvement and scarring alopecia. In severe
cases, treatment with oral steroids can be helpful.

Many drugs including antihypertensives, thiazides and antimalarials can induce a Lichen planus like
a rash.

Henoch-Schönlein purpura has a purpuric appearance and usually affects the legs and buttocks and is
not usually pruritic.

Guttate psoriasis can follow infections but has a more scaly appearance typical of psoriasis. It would
be unusual to develop a reaction to the Augmentin 3 weeks after the drug was stopped.

Reference:

1. Gorouhi, F. Davari, P. Fazel, N. Cutaneous and Mucosal Lichen Planus: A Comprehensive Review
of Clinical Subtypes, Risk Factors, Diagnosis, and Prognosis. ScientificWorldJournal. 2014; 2014:
742826. doi: 10.1155/2014/742826

Lichen planus Correct answer selected

A 72-year-old man presented with this crusting lesion on the right side of his face. A cousin that he
hadn’t seen for a number of years noticed this and suggested he ought to seek medical advice.

He was unaware of the lesion which was painless and crusted as shown, being approximately 1.5 cm
in diameter with an irregular margin and slightly raised.

He is not sure of how long the lesion has been there having noticed crusting for roughly 1 year. This
may have gradually enlarged.
In his medical history, he was diagnosed with type 2 diabetes 8 years ago and takes amlodipine 10mg
daily, metformin 500mg tds, gliclazide 80mg daily and simvastatin 40mg daily.

Which of the following is the likely diagnosis?

1. Actinic keratosis
2. Basal cell carcinoma
3. Drug reaction
4. Hutchinson’s lentigo
5. Squamous cell carcinoma
Explanation:

This lesion is suspicious of a squamous cell carcinoma being more apparent on sun-exposed skin.

The scaly redness with an irregular margin is typical. SCC is the second commonest skin cancer
behind basal cell carcinoma which may be differentiated by the typically more raised, pearlier
appearance and by the fact that metastasis is more common.

Generally, treatment relies on surgical excision.

Reference:

1. Squamous cell carcinoma – PatientUK


2. Squamous cell carcinoma Correct answer

A 70-year-old lady presented with a 2-month history of a pruritic rash affecting her elbows and
knees. These appeared acutely on her elbows and a week later started to involve her knees and
buttocks.

The eruption was associated with intense pruritus and the pruritus was worse at night. She had a past
history of eczema but was on no regular medication.

There was a family history of heart disease. An examination is shown in the photo.

What is the most likely diagnosis?


1. Dermatitis herpetiformis
2.

3.
4. Eruptive xanthomas
5. Henoch-Schönlein purpura
6. Infected eczema
7. Psoriasis

Explanation:
This young patient presents with a pruritic rash on the extensor areas. The main differential diagnosis
would be dermatitis herpetiformis and psoriasis. In dermatitis herpetiformis, there is formation of
vesicles which are characteristically very pruritic and usually have been excoriated, so you tend to
see excoriation marks rather than vesicles.

Psoriasis is characterised by well-defined, scaly erythematous plaques.

The distribution would fit with eruptive xanthomas but these tend to be asymptomatic.

Henoch-Schönlein purpura is a purpuric rash affecting primarily the lower limbs and infected
eczema tends to be more inflamed, scaly and weepy.

Reference:

1. Dermnetnz: Dermatitis herpetiformis

2. Dermatitis herpetiformis Correct answer selected


3.

This 70-year-old man came along to clinic complaining about these lesions that his daughter noted
on his head. His daughter suggested that he came to see you about it.

He has played golf most of his life but usually wears sun creams and hats.

These lesions are non-painful occasionally itchy, scaly and sometimes catch on his comb. He caught
the one lesion on the right side of the forehead with a comb and it bled a little.
Which of the following is the likely diagnosis?

1. Actinic keratosis
2. Eczema
3. Malignant melanoma
4. Multiple squamous cell carcinoma
5. Psoriasis

Explanation:

This caucasian man has numerous scaly, relatively flat lesions on his forehead which are typical of
actinic keratoses otherwise known as solar keratoses.
As the name implies, they are a consequence of sun damage and are particularly common in elderly
fair-skinned people and can occur on any sun-exposed area, particularly the face, lips, head and
hands.

They are pre-malignant conditions that can develop into squamous cell carcinoma.

Treatment includes a number of approaches including freezing but topical creams are usually
effective first-line agents including diclofenac, 5-fluorouracil and imiquimod.

Reference:

1. Actinic Keratoses: PCDS

2. Actinic keratosis Correct answer selected

A 70-year-old woman with type-2 diabetes has been attending medical outpatients for some time and
you have been checking her blood pressure.

She is obese and her blood pressure recordings have ranged between 160-175 mmHg systolic and
95-105 mmHg diastolic.

You have given her advice on weight management and exercise yet she hasn’t really made much
progress. You elect to treat her with an ACE inhibitor and commence lisinopril.

What target level of blood pressure are you aiming to achieve in this patient when monitoring the
blood pressure in the outpatient's clinic?/

1. < 130/80
2. < 135/85
3. < 140/80
4. < 140/90
5. < 150/90

Explanation:

This patient has persistently elevated blood pressure and has received appropriate education and
advice relating to lifestyle and dietary intervention.

As the blood pressure is unaffected the most logical choice of an ACE inhibitor has been selected as
the most appropriate anti-hypertensive.

The target to which we should aim would be <140/90 mmHg when monitoring BP in the clinic
according to NICE.

When using ambulatory BP or home blood pressure measurements use below 135/85 mmHg for
adults aged under 80, below 145/85 mmHg for adults aged 80 and over.

Reference:

1. NICE: measuring blood pressure


2. < 140/90 Correct answer selected

A 75-year-old Afro-Caribbean male has recently been diagnosed with hypertension.

Other than hypertension, he also has a 6-year history of well-controlled type 2 diabetes for which he
takes metformin.

His blood pressure recordings are between 160-180 systolic and diastolic blood pressure is around
90-110 mmHg. His BMI is 31.2 kg/m2.

Which of the following would be the most appropriate medication to commence for this patient?

1. Amlodipine
2. Atenolol
3. Bendrofluazide
4. Lisinopril and bendrofluazide dual therapy
5. Losartan
Explanation:

Hypertension in Afro-Caribbean patients is typically associated with a low renin level and in
consequence, does not respond that well to agents such as ACE inhibitors which rely on an activated
renin-angiotensin-aldosterone system.

However, ARBs appear to be superior. Beta-blockers are less effective as they lower renin
concentrations and again, if already low levels of renin, are less effective.

Therefore, first-line drug treatment for Afro-Caribbean patients with hypertension and type 2
diabetes remains an ARB with calcium channel blocker (CCB) as a second line option.

Dual therapy was advocated but no longer with the approach of starting with a single agent.

Reference:

1. NICE: starting antihypertensive drug treatment


2. Brewster, L. M. Seedat, Y. K. Why do hypertensive patients of African ancestry respond better to
calcium blockers and diuretics than to ACE inhibitors and ß-adrenergic blockers? A systematic
review BMC Med. 2013; 11: 141. Published online 2013 May 30. doi: 10.1186/1741-7015-11-141.

3. Losartan Correct answer


A 78-year-old female was referred to the clinic by her GP. She has a history of chronic atrial
fibrillation, stage 3 chronic kidney disease stage 3 and hypertension. She was being treated with
warfarin, bendrofluazide, ramipril, digoxin and simvastatin.

On examination, she appeared well and clinically euthyroid. Her pulse was 56 bpm irregular and her
blood pressure was 164/82 mmHg.

Which of the following would be the most appropriate approach to the further management of this
patient’s blood pressure?

1. Amlodipine
2. Doxazosin
3. Metoprolol
4. Spironolactone
5. Verapamil
Explanation:

In this elderly woman who has isolated systolic hypertension as revealed by the diastolic below 90
mmHg, the most appropriate antihypertensive would be the addition of the calcium antagonist.

She already has a relative bradycardia, so the addition of a beta-blocker and verapamil may
exacerbate the bradycardia.

Studies such as Syst-Eur and HOT demonstrated the reduced cardiovascular impact of CCB in the
elderly with ISH.

In this patient, one would argue that the CCB should have been first-line treatment possibly in
addition with ACEI.

Reference:

1. Celis, H., Fagard, R. H., Staessen, J. A., & Thijs, L. (2001). Risk and benefit of treatment of isolated
systolic hypertension in the elderly: evidence from the Systolic Hypertension in Europe Trial.
Current opinion in cardiology, 16(6), pp. 342-348.
2. NICE: starting antihypertensive drug treatment

3. Amlodipine Correct answer selected


A 72-year-old man attends outpatients as he has been slightly concerned about his blood pressure. He
has been checking his blood pressure at home using an automated machine and found readings of
>160 mmHg systolic.

He is generally otherwise fit and well, and other than a hip replacement for osteoarthritis, has not
needed to seek medical advice.

His BMI is 26.5 kg/m2 and his blood pressure is sustained between 160-170 mmHg systolic with
diastolic recordings of 80-90 mmHg.

No abnormalities are noted on examination. Investigations reveal normal U+Es.

Which of the following is the most appropriate management for this man’s blood pressure?

1. Amlodipine
2. Atenolol
3. Bendrofluazide
4. No treatment necessary
5. Ramipril
Explanation:

This man has isolated systolic hypertension (ISH) with a systolic blood pressure above 140 mmHg
yet diastolic blood pressure less than 90 mmHg.

This type of blood pressure is typically found in older patients and is associated with co-morbidities,
particularly stroke.

The most appropriate treatment for ISH is a Calcium Channel Blocker (CCB) such as amlodipine.

Reference:

1. NICE: starting antihypertensive drug treatment


2. Amlodipine Correct answer selected

A 76-year-old woman attends clinic for her diabetic annual review.

The only thing that she is concerned about is a tickly and irritating cough that she has been aware of
for the last few months.

Her glycaemic control is generally good, with an HbA1c of 7.2% and she takes Metformin 500 mg
tds.
She has a history of hypertension associated with her diabetes and takes atenolol 50mg daily,
lisinopril 20mg daily and amlodipine 5 mg daily. She also takes simvastatin 40mg daily and aspirin
75mg daily.

Her blood pressure is 132/74 mmHg.

Which of the following is likely to be responsible for her cough?

1. Amlodipine
2. Aspirin
3. Atenolol
4. Lisinopril
5. Metformin

Explanation:

ACE inhibitor-induced cough is very common occurring in up to 30% of users and is typically a mild
tickly and irritating cough.
It is one of the more common side effects associated with ACEi and usually resolves on withdrawal
of the drug. It is a consequence of ACEi induced mast cell degranulation.

Stopping the ACE inhibitor and switching to an alternative antihypertensive such as an angiotensin
receptor blocker (ARB), although cough can still occur with this drug, in this case, would probably
be the best option.

Reference

1. NICE: starting antihypertensive drug treatment


2. ACE inhibitors medicine leaflet

3. Lisinopril Correct answer selected

A 72-year-old man with a five-year history of obesity and type-2 diabetes is diagnosed with
hypertension.

His blood pressure measurements reveal recordings of 150-160 mmHg systolic and 88-94 mmHg
diastolic.

He has been advised for many years on weight loss and lifestyle changes but to little avail.

His current medication includes simvastatin 40 mg daily and metformin 1g bd.

Which of the following would be most appropriate regarding the management of his blood pressure?

1. Continue with lifestyle measures alone


2. Commence amlodipine
3. Commence atenolol
4. Commence bendrofluazide
5. Commence ramipril

Explanation:

The most appropriate pharmacological treatment in this man with type 2 diabetes and hypertension
would be the introduction of an ACE inhibitor, such as ramipril as a first-line treatment. However, it
is important to continue to reinforce lifestyle advice - patients who achieve weight loss through diet
are likely to have a reduction in systolic BP of at least 10 mmHg, and exercise has been shown to
achieve similar results.

Reference:

1. NICE: starting antihypertensive drug treatment


2. Commence ramipril Correct answer selected

A 71-year-old male is seen because he is told that he had high blood pressure when he went to a Well
Man service at his local superstore car park.

He keeps himself fit and does weight training. He is categorised as obese with a BMI of 34kg/m2, yet
his waist circumference is 90cm. The card that he received in the clinic states that his blood pressure
is 166/88 mmHg.

When taking his blood pressure, you realise that you need an oversize cuff to measure this properly.

What effect does the use of an undersized cuff have on the estimation of blood pressure?

1. No effect on estimation of blood pressure


2. Modest underestimation of blood pressure (<10mmHg)
3. Marked underestimation of blood pressure (~20 mmHg)
4. Modest overestimation of blood pressure (<10 mmHg)
5. Marked overestimation of blood pressure (~20 mmHg)
Explanation:

1. A standard cuff with a bladder measuring 12 x 26 cm is used for the majority of adult arms
2. A large cuff with a bladder measuring 12 x 40 cm is used for obese arms
3. A small cuff with a bladder measuring 12 x 18 cm is used for lean adult arms and children
4. The use of a cuff with a bladder too small (undercuffing) will result in an overestimation of BP as
much as 30 mmHg in obesity.
5. The use of a cuff with a bladder too large (overcuffing) will result in an underestimation of BP up to
30 mmHg.
6. Undercuffing is more common than overcuffing.
Reference:

7. Hypertension recommendations for conventional, ambulatory and home blood pressure measurement
8. NICE: starting antihypertensive drug treatment
9. The blood pressure cuff - Doctors practical guide

10. Marked overestimation of blood pressure (~20 mmHg) Correct answer


A 55-year-old female with high blood pressure returns to your clinic for a further blood pressure
check.

Initially, her blood pressure was 160/88 mmHg, and on this occasion, it is 162/90 mmHg.

She has a BMI of 29.6 kg/m2, a pulse of 62 bpm regular.

What might you find on examination of this patient that could suggest that she has target organ
damage associated with the hypertension?

1. Heart murmur
2. Ketones ++ on urine dipstick
3. Kayser-Fleischer rings in the eye
4. Left ventricular hypertrophy on the ECG
5. Visual field defect
Explanation:

In accordance with NICE guidelines, all people with hypertension should be offered the following
investigations in order to assess target organ damage:

1. Arrange for a 12-lead electrocardiograph to be performed.


2. Test for the presence of protein in the urine by sending a urine sample for estimation of the
albumin:creatinine ratio and test for haematuria using a reagent strip
3. Take a blood sample to measure plasma glucose, electrolytes, creatinine, estimated glomerular
filtration rate, serum total cholesterol and HDL cholesterol
4. Examine the fundi for the presence of hypertensive retinopathy

Evidence of target organ damage includes:

5. ECG - Left ventricular hypertrophy is due to increased strain on the left ventricle due to higher
pressures.
6. Urine dipstick - the presence of leakage of protein into the urine (albuminuria or proteinuria), or
reduced renal function.
7. Blood test - evidence of reduced renal function
8. Fundoscopy - evidence of hypertensive retinopathy (flame haemorrhages, microaneurysms, cotton
wool spots)
Reference:

9. NICE: Hypertension

10. Left ventricular hypertrophy on the ECG Correct answer selected


A 73-year-old male with a recent history of hypertension and type-2 diabetes attends clinic
complaining of vague proximal muscle aches and pains. He has encountered these problems over the
last few months.

He is currently taking aspirin, metformin, simvastatin, losartan, omeprazole and amlodipine as


regular therapies.

On Examination, there is little to find other than some tenderness of the shoulder girdle

Which of the following is most likely to have caused his presentation?

1. Amlodipine
2. Losartan
3. Metformin
4. Omeprazole
5. Simvastatin
Explanation:

The presentation is suggestive of a statin-induced myositis. This occurs in approximately 5% of


those treated with statins and usually presents with vague muscle aches and pains.

CPK is often elevated and the patients respond to withdrawal of the offending drug.

Myositis is also associated with fibrates and can be exacerbated with the concomitant use of a statin
and fibrate.

Alternative agents to treat hyperlipidaemia, such as ezetimibe, could be used.

Reference:

1. NICE: simvastatin
2. Selva-O'Callaghan, A. Alvardo-Cardenas, M. Pinal-Fernandez, I. Et al. Statin-induced myalgia and
myositis: an update on pathogenesis and clinical recommendations. Expert Rev Clin Immunol. 2018
Mar; 14(3): 215–224. Published online 2018 Feb 23. doi: 10.1080/1744666X.2018.1440206

3. Simvastatin Correct answer selected

A 77-year-old man visited his GP to discuss tetanus vaccination. It had been some years since his
blood pressure had last been taken, so the GP decided to check it. The reading was 158/116 mmHg.

The patient was advised on lifestyle modification in order to help lower his blood pressure, yet on
review a couple of months later, he was still hypertensive with a reading 156/112 mmHg. The GP
initiated antihypertensive medication.
Four weeks later, he presented with new-onset problematic constipation. He described the stools as
hard, but normal in colour and odour.

He had always taken care to eat a high fibre diet, was a non-smoker and consumed six units of
alcohol weekly.

On examination, he was apyrexial, had a heart rate of 80 bpm, blood pressure of 148/106 mmHg and
a respiratory rate of 14 breaths per minute. Hard faeces could be felt on digital rectal examination.

Which drug would be most likely to cause this man’s problems?

1. Atorvastatin
2. Bendroflumethiazide
3. Enalipril
4. Propanolol
5. Nifedipine

Explanation:

Nifedipine, along with other calcium channel blockers, is known to cause constipation. There is
evidence to suggest that this is because it reduces the myoelectric response to eating.

If constipation does not resolve once it has been stopped, another cause should be sought. It is
notable that this man has constipation despite a high fibre diet.

Other antihypertensive medications are unlikely to cause constipation.


Reference:

1. NICE: nifedipine
2. Medicine leaflet: Nifedipine

3. Bassotti G, Calcara C, Annese V, Fiorella S, Roselli P, Morelli A. Nifedipine and verapamil inhibit
the sigmoid colon myoelectric response to eating in healthy volunteers. Dis Colon Rectum 1998
Mar; 41(3):377-80.

4. Nifedipine Correct answer selected

A 72-year-old man presented to his GP with polydipsia, sight problems and general fatigue. He had a
history of impaired glucose tolerance. He had given up smoking five years previously and was
teetotal.

He was taking atenolol and amlodipine.

On examination, he was apyrexial, had a heart rate of 78 bpm, a blood pressure of 138/94 mmHg and
a respiratory rate of 14 breaths per minute.

Fundoscopy showed background retinopathy and a fasting blood glucose reading was 13.4 mmol/L
(normal is 3.5-5.5).

A formal sample was sent, and repeat readings confirmed a diagnosis of type-2 diabetes mellitus. He
was prescribed metformin, simvastatin and aspirin.

A full blood count showed the following:

Investigation Result Normal range


Hb 14.3 g/dL 13-18 g/dL
MCV 88 fL 76-96 fL
Platelets 347 x 109/L 150-400 x 109/L
WCC 8.1 x 109/L 4.0-11.0 x 109/L
RCC 5.9 x 1012/L 4.5-6.5 x 1012/L
One month later, he returned for review. His symptoms had resolved and on examination, he was
apyrexial, had a heart rate of 82 bpm, a blood pressure of 158/114 mmHg and a respiratory rate of 16
breaths per minute. Fasting blood glucose was 7.2 mmol/L. He was prescribed amlodipine and
atenolol.

Investigation Result Normal range


Hb 13.3 g/dL 13 -18 g/dL
MCV 104 fL 76-96 fL
Platelets 327 x 109/L 150-400 x 109/L
WCC 8.1 x 109/L 4.0-11.0 x 109/L
RCC 4.8 x 1012/L 4.5-6.5 x 1012/L
Three months later, on further review, the patient was feeling well. His blood pressure was 144/92
mmHg and his HbA1C was 6.9%.

What is the most likely diagnosis?

1. Alcoholism
2. Hypothyroidism
3. Sideroblastic anaemia
4. Spherocytosis
5. Vitamin B12 deficiency

Explanation:

Metformin is known to cause vitamin B12 deficiency, so should always be suspected as a cause of
raised MCV or megaloblastic anaemia.

It is thought to inhibit vitamin B12 absorption from the ileum by affecting the calcium-dependent
membrane action.

When metformin is stopped, the vitamin B12 deficiency resolves; if it does not, an alternative cause
must be sought.

While this is a mild case, long term metformin therapy has been known to cause a much more severe
picture, with deterioration in mental state and peripheral neuropathy.

Reference:

1. NICE: metformin hydrochloride


2. Medicine leaflet: Metformin
3. Pernicious anaemia and B12 deficiency
4. Metformin-related vitamin B12 deficiency

5. Vitamin B12 deficiency Correct answer selected

A 55-year-old female presents with menopausal symptoms of flushing and sweats which have been
problematic over the last six to twelve months.

Her last period was over 12 months ago. Her life has been quite affected by this problem and she is
desperate to improve these symptoms.

She is otherwise well and has no medical history of note. There is no other family history of note.
Her BMI is 28.8 kg/m2 and her blood pressure is 122/76 mmHg.

Which of the following would be the most appropriate approach to managing this patient’s
symptoms?

1. Combined Hormone replacement therapy


2. Plant oestrogens
3. St John’s Wort
4. Vaginal oestrogen cream
5. Venlafaxine
Explanation:

This post-menopausal woman complains of disabling sweats and the most appropriate approach to
the management of this would be the use of combined HRT containing oestrogen and progestogens.

After menopause, women stop producing the hormones oestrogen and progesterone, and this is what
causes the symptoms that she is experiencing.

Therefore, replacing these hormones with HRT would be beneficial for her. There appears to be no
contra-indication to HRT and so commencing a low dose combined HRT (oestrogen with
progestogen) to begin with would be appropriate, which can be titrated according to her symptoms.

Some contraindications for HRT include a history of breast, ovarian or endometrial cancer, previous
clots such as a DVT or strokes, untreated high blood pressure, and liver disease.

Reference:

1. http://www.nhs.uk/Conditions/Hormone-replacement-therapy/Pages/Introduction.aspx

2. Combined Hormone replacement therapy Correct answer selected


A 55-year-old female presented with acute onset of weakness in the left arm which resolved after 2
hours.

She was diagnosed with a stroke mimic. Which of the following is true regarding stroke mimics?

1. A low National Institutes of Health Stroke Scale score and the presence of diabetes are suggestive of
stroke mimic
2. Migraine is the commonest condition mimicking acute stroke
3. Patients with migraine have a lower risk of stroke than those without migraine
4. Seizure is an unusual cause for stroke mimic
5. Systolic blood pressure above 160 mmHg is a predictive indicator of stroke mimic
Explanation:

Stroke mimic is where other conditions are the cause of stroke-like presentations.

It may account for roughly 30% of all presentations potentially considered to be a stroke.

The commonest of these is seizures with studies revealing it to account for around 21%. The next
most common mimic is hypoglycaemia and sepsis.

A low NIHSS score (<5) together with a blood pressure <140 mmHg systolic, history of diabetes or
a lack of history of arrhythmia are predictors of stroke mimic.

Reference:

1. Okano, Y., Ishimatsu, K., Kato, Y., Yamaga, J., Kuwahara, K., Okumoto, K., & Wada, K. (2018).
Clinical features of stroke mimics in the emergency department. Acute medicine & surgery, 5(3), pp.
241-248.

2. A low National Institutes of Health Stroke Scale score and the presence of diabetes are suggestive of
stroke mimic Correct answer selected

An 85-year-old man was rushed to the emergency room after experiencing sudden right-sided
hemiparesis. Further questioning revealed that the patient experienced involuntary movements of
one-half of his body for 5 minutes at the beginning of the episode. On examination, the patient was
drowsy and had weakness in his right arm and leg. A brain CT scan revealed an early infarct in the
middle cerebral artery territory. A full blood count, clotting screen, and renal function tests were
within normal limits. His random plasma glucose was 23 mmol/L (3 - 8.5 mmol/L) while an ECG
showed a sinus rhythm. Furthermore, he has a significant past history of hypertension, type-2
diabetes mellitus, and a basal ganglia haemorrhage from 7 years ago.

What is the recommended initial management for this patient?

1. Aspirin only immediately


2. Dipyridamole orally
3. IV heparin infusion
4. Thrombolysis with tPA
5. Warfarin

1. Aspirin only immediately Correct answer selected


A 72-year-old woman presents with a one-hour history of weakness of the left face and arm. The
MAU team suspect a stroke and imaging is requested.

Which of the following are the features of acute stroke on neuroimaging?

1. CT angiography is the first-line investigation for all patients with suspected stroke
2. CT performed within 6 hrs of symptom onset has high sensitivity and specificity for acute stroke
3. Loss of volume and hypoattenuation are features of early acute stroke on CT
4. Standard MRI has better sensitivity and specificity at detecting acute stroke than CT
5. The presence of restricted diffusion on MRI is highly suggestive of ischaemic stroke rather than
stroke mimic

Explanation:

Understanding imaging in acute stroke is important to all stroke physicians, radiologists and acute
physicians.
Currently, non-contrasted CT is the primary imaging modality for acute stroke in the UK. The three
descriptions of stroke are acute (<24hrs); subacute 24 hrs to 5 days and chronic >5 days.

Acute changes are evident as loss of the grey-white matter interface and effacement of the cortical
sulci.

Subacute changes are illustrated by vasogenic oedema with greater mass effect, hypoattenuation and
well-delineated margins. Chronic changes are seen as loss of brain tissue and hypoattenuation. So, a
non-contrasted CT might reveal features of stroke but would reveal evidence of bleed.

There is a sensitivity of roughly 55% for acute stroke with CT but high specificity (>90%).

Subacute changes (>24hrs) are better detected by MRI than CT however, MR with diffusion (DWI)
improves sensitivity from 50% to 90% with restricted diffusion being highly suggestive of stroke
rather than stroke mimic in the acute setting.

Reference:

1. Birenbaum, D., Bancroft, L. W., & Felsberg, G. J. (2011). Imaging in acute stroke. Western Journal
of Emergency Medicine, 12(1), 67.
2. Xin, Y., & Han, F. G. (2016). Diagnostic accuracy of computed tomography perfusion in patients
with acute stroke: A meta-analysis. Journal of the neurological sciences, 360, 125-130.

3. The presence of restricted diffusion on MRI is highly suggestive of ischaemic stroke rather than
stroke mimic Correct answer selected

An 80-year-old man presented to hospital with an episode of impaired speech that lasted less than 10
minutes.

There was no upper or lower limb weakness. He has a past history of hypertension treated with
amlodipine.

Examination is entirely normal with a blood pressure of 130/75 mmHg.

What treatment would you consider in this man?

1. Aspirin
2. Aspirin + clopidogrel
3. Clopidogrel
4. No treatment needed
5. Warfarin
Explanation:

This man has an ABCD2 score of 2 (age and speech impairment for <10 minutes with controlled
high blood pressure). Recommendations for low-risk TIAs is to start aspirin 160–325 mg as
monotherapy.

UpToDate:

"Transient ischaemic attack — For patients with a transient ischaemic attack


(TIA) who do not have a known cardioembolic source at presentation, we start
antiplatelet therapy immediately while evaluating the ischaemic mechanism.
1. We start aspirin (162 to 325 mg/daily) alone for low-risk TIA, defined by an
ABCD2 score <4 (table 2).
2. For high-risk TIA, defined as an ABCD2 score of =4 (algorithm 2), we employ
dual antiplatelet therapy (DAPT) using aspirin (160 to 325 mg loading dose,
followed by 50 to 100 mg daily) plus clopidogrel (300 to 600 mg loading dose,
followed by 75 mg daily) for the first 21 days. This strategy reduces the risk of
recurrent ischaemic stroke with a possible small increase in the risk of moderate
or major bleeding and no apparent impact on mortality. (See 'Combination
antiplatelets' below.)
For patients on single antiplatelet therapy with aspirin or clopidogrel at the time
of TIA onset, we recommend switching to DAPT using aspirin plus clopidogrel
for the first 21 days for high-risk TIA (ie, an ABCD2 score of =4). For patients
on other antiplatelet agents, the decision should be individualised based on the
underlying indication.
For patients either on anticoagulation at the time of TIA onset, or with a clear
indication for anticoagulation (eg, atrial fibrillation, venous thromboembolism,
mechanical heart valve) we recommend anticoagulation rather than antiplatelet
therapy. In patients who are sub-therapeutically or not anticoagulated at
presentation, bridging anticoagulation with heparin, low molecular weight
heparin, or a direct oral anticoagulant (DOAC) should be considered. In patients
who are therapeutically anticoagulated at presentation, management should be
individualized based on the underlying mechanism of the TIA. In some
instances, such as when the TIA is more likely due to atherosclerosis than to
cardioembolism, it may be reasonable to add single antiplatelet therapy. Triple
therapy (ie, anticoagulation plus DAPT) is associated with a high risk of
haemorrhage and should be avoided. (See 'Anticoagulant failure' below.)
Once the ischaemic mechanism is determined, antithrombotic therapy can be
modified as necessary."
NICE guideline [NG128]:

“Initial management of suspected and confirmed TIA


1.1.4 Offer aspirin (300 mg daily), unless contraindicated, to people who have
had a suspected TIA, to be started immediately. [2019]
1.1.5 Refer immediately people who have had a suspected TIA for specialist
assessment and investigation, to be seen within 24 hours of onset of symptoms.
[2019]
1.1.6 Do not use scoring systems, such as ABCD2, to assess risk of subsequent
stroke or to inform urgency of referral for people who have had a suspected or
confirmed TIA. [2019]
1.1.7 Offer secondary prevention, in addition to aspirin, as soon as possible after
the diagnosis of TIA is confirmed. [2008, amended 2019]”

Reference:
3. Filho, J.O. and Mullen, M.T. (updated 2020). Antithrombotic treatment of acute ischemic stroke and transient ischemic attack.
UpToDate.
4. NICE (2019). NICE guideline [NG128]: Stroke and transient ischaemic attack in over 16s: diagnosis and initial management.

5. Aspirin Correct answer selected

An 80-year-old man presented to hospital with an episode of impaired speech and left-sided
weakness that lasted less than around 30 minutes.

He has a past history of hypertension treated with amlodipine. Examination is entirely normal with a
blood pressure of 150/90 mmHg.

What initial treatment would you consider in this man?

1. Aspirin
2. Aspirin + clopidogrel
3. Clopidogrel
4. No treatment needed
5. Warfarin
Explanation:

The following details the NICE guidelines on such cases. In this case, the ABCD2 score is 6 so he
would be classed as high risk and some authorities would recommend dual antiplatelet therapy.

However, the exam is bound by NICE guidance so the correct approach here is aspirin initially.

NICE guideline [NG128]:

“Initial management of suspected and confirmed TIA


1. 1.1.4 Offer aspirin (300 mg daily), unless contraindicated, to people who have
had a suspected TIA, to be started immediately. [2019]
2. 1.1.5 Refer immediately people who have had a suspected TIA for specialist
assessment and investigation, to be seen within 24 hours of onset of symptoms.
[2019]
3. 1.1.6 Do not use scoring systems, such as ABCD2, to assess risk of subsequent
stroke or to inform urgency of referral for people who have had a suspected or
confirmed TIA. [2019]
4. 1.1.7 Offer secondary prevention, in addition to aspirin, as soon as possible after
the diagnosis of TIA is confirmed. [2008, amended 2019]”
Reference:
5. Filho, J.O. and Mullen, M.T. (updated 2020). Antithrombotic treatment of acute ischemic stroke and transient ischemic attack.
UpToDate.
6. NICE (2019). NICE guideline [NG128]: Stroke and transient ischaemic attack in over 16s: diagnosis and initial management.

7. Aspirin Correct answer selected

A 75-year-old man has sustained a middle cerebral artery stroke. He was found to be in atrial
fibrillation with a rate of 95 bpm. Aspirin was started and the CT scan showed an ischaemic stroke
with no haemorrhagic transformation. After discussion with the patient, he opted to be started on
warfarin.

How long after the onset of symptoms should one wait before starting warfarin?
1. 7 days
2. 14 days
3. 21 days
4. 28 days
5. 90 days

Explanation:

Patients with atrial fibrillation who have had a stroke are advised to start warfarin unless there is a
contraindication to anticoagulation and the CT shows no haemorrhagic transformation.

It is advisable that one waits for 14 days before starting warfarin to reduce the risk of haemorrhagic
transformation. Should the patient already be on warfarin at the time of the stroke, one can continue
warfarin provided a haemorrhage has been excluded.

NICE guideline [NG128]:


"Anticoagulation treatment for other comorbidities
1. 1.4.17 Ensure that people with disabling ischaemic stroke who are in atrial
fibrillation are treated with aspirin 300 mg for the first 2 weeks before
anticoagulation treatment is considered. [2008]
2. 1.4.18 For people with prosthetic valves who have disabling cerebral infarction
and who are at significant risk of haemorrhagic transformation, stop
anticoagulation treatment for 1 week and substitute aspirin 300 mg. [2008]
3. 1.4.19 Ensure that people with ischaemic stroke and symptomatic proximal deep
vein thrombosis or pulmonary embolism receive anticoagulation treatment in
preference to treatment with aspirin unless there are other contraindications to
anticoagulation. [2008]
4. 1.4.20 Treat people who have haemorrhagic stroke and symptomatic deep vein
thrombosis or pulmonary embolism to prevent the development of further
pulmonary emboli using either anticoagulation or a caval filter. [2008]"
Reference:
5. NICE (2019). NICE guideline [NG128]: Stroke and transient ischaemic attack in over 16s: diagnosis and initial management.

6. 14 days Correct answer selected

Mr R. presents to the Accident and Emergency department with slurred speech and right arm
weakness of one-hour duration. His symptoms have neither improved nor worsened.

He is hypertensive with type 2 diabetes. He had a myocardial infarction three years ago.

He takes ramipril, amlodipine, aspirin and metformin.

Two months ago he attended the one-stop haematuria clinic after an episode of haematuria.

After a cystoscopy, he was told that nothing was found. Four months ago he had a knee replacement
and has recovered well.

He had a CT scan fifteen years ago after falling from a ladder. He suffered significant facial trauma
and required reconstructive surgery to the jaw.

A 4mm aneurysm was found on the circle of Willis. He was monitored by the neurosurgical team
with yearly CT scans and was discharged five years ago having been reassured that the aneurysm had
not grown over that time.

Which of those problems represents an absolute contraindication to thrombolysis?

1. 4mm intracerebral aneurysm


2. Currently taking an antiplatelet
3. Haematuria 2 months ago
4. History of significant facial trauma
5. Knee replacement 4 months ago

Explanation:

The only absolute contraindication in that list is the intracranial vascular lesion. Whilst all others can
be absolute or relative contraindications the timespan is not within the limits set to withhold
thrombolysis.

Reference:

1. Radcliffe Cardiology: Table 2: Contraindications To Thrombolytic Therapy

2. 4mm intracerebral aneurysm Correct answer selected


A 76-year-old man presents to the ER with sudden onset vertigo and imbalance while walking with
occasional coughing while eating.

On examination, the patient has ipsilateral cerebellar signs and Horner's syndrome with decreased
facial pain sensation. In addition, he has a loss of sensation on the opposite side of his trunk and
extremities.

The patient has a significant past history of hypertension, type-2 diabetes mellitus, and ischaemic
heart disease.

Which of the following arteries is most likely affected in this patient?

1. Anterior inferior cerebellar artery


2. Basilar artery
3. Posterior cerebral artery
4. Posterior communicating artery
5. Posterior inferior cerebellar artery
Explanation:

This is a case of lateral medullary syndrome, also known as Wallenberg's syndrome or posterior
inferior cerebellar artery (PICA) syndrome, or vertebral artery syndrome. Lateral medullary
syndrome is a neurological disorder that causes a variety of symptoms due to ischaemia in the
brainstem's lateral medulla oblongata. Ischaemia is caused by a blockage in the vertebral artery or
the posterior inferior cerebellar artery.

This syndrome is distinguished by sensory deficits in the trunk and extremities on the contralateral
(opposite) side of the lesion, as well as sensory deficits in the face and cranial nerves on the
ipsilateral side (same side as the lesion). If the lateral spinothalamic tract is involved, there will be a
loss of pain and temperature sensation. The main symptom from which a diagnosis can be made is
the cross-body finding.

Patients frequently have difficulty walking or maintaining balance (ataxia) caused by damage to the
cerebellum or the inferior cerebellar peduncle. The involvement of the nucleus ambiguus, which
supplies the vagus and glossopharyngeal nerves, can cause dysphagia, dysphonia, and slurred speech.
Damage to the hypothalamospinal fibres disrupts sympathetic nervous system relay, resulting in
symptoms similar to Horner's syndrome, such as miosis, anhidrosis, and partial ptosis.

Anterior cerebral artery occlusion (AICA) syndrome, also known as lateral pontine syndrome, is
typically the result of AICA occlusion. Its symptoms include sudden onset of vertigo and vomiting,
nystagmus, dysarthria, and falling to the side of the lesion due to the involvement of vestibular
nuclei. In addition, it presents with ipsilateral features such as hemiataxia, loss of all facial sensations
(due to damage to the primary sensory trigeminal nucleus), facial paralysis (due to damage to the
facial nucleus), hearing loss, and tinnitus.

Basilar artery occlusion is a result of the obliteration of blood supply to the posterior circulation or
vertebrobasilar system of arteries to the brain. Therefore, a variable degree of basilar artery occlusion
can cause ipsilateral cranial nerve deficits, contralateral hemiparesis, sensory impairment,
coordination deficits, quadriparesis, nausea, headache, vertigo, dizziness and vertigo, as well as
aphasia, dysarthria, dysphagia, loss of consciousness, coma, and cardiopulmonary compromise.

Posterior cerebral artery involvement can affect the blood supply of the occipital lobe, the
inferomedial temporal lobe, the thalamus, and part of the brainstem and midbrain. It can present as a
visual field defect, visual dysfunction, and cognitive and behavioural disturbances.

The posterior communicating artery is the third most common cause of Circle of Willis aneurysm,
and it can lead to compression of the oculomotor nerve.

Reference:

1. Saleem F, M Das J. Lateral Medullary Syndrome. [Updated 2021 Aug 11]. In: StatPearls [Internet].
Treasure Island (FL): StatPearls Publishing; 2022.
2. Kaye, V. What are the signs and symptoms of lateral medullary (Wallenberg) syndrome in
vertebrobasilar stroke? Medscape, 2021.

3. Posterior inferior cerebellar artery Correct answer selected

A 70-year-old female was admitted into the ward following a left total anterior circulation ischaemic
infarct.
What should be prescribed for the prevention of deep venous thrombosis?

1. Graduated compression stockings


2. Intermittent pneumatic compression sleeves
3. Low molecular weight heparin
4. None
5. Unfractionated heparin

Explanation:

Patients with immobility after acute stroke should be offered intermittent pneumatic compression
(IPC) within 3 days of admission to hospital for the prevention of deep vein thrombosis.
Treatment should be continuous for 30 days or until the patient is mobile or discharged, whichever is
sooner.

The CLOTS 3 trial showed that IPC using sequential compression with venous refill technology in
immobile patients in the first 30 days after stroke is an effective treatment for reducing proximal
DVT and improves survival but not functional outcomes.

Reference:

1. Intercollegiate Stroke Working Party (2016). National clinical guideline for stroke. 5th edn. London:
Royal College of Physicians.

https://www.nice.org.uk/guidance/ng158/chapter/Recommendations

2. Intermittent pneumatic compression sleeves Correct answer selected

Mr A was admitted into the hyperacute stroke unit with a severe right total anterior circulation stroke
within 2 hours and received intravenous thrombolysis.

He is medically stable but still has difficulty mobilising.

When can the healthcare professional team start mobilising Mr A?

1. Between 24 and 48 hours


2. Immediately
3. Not to mobilise until repeat CT brain available
4. Within 12 hours
5. Within 24 hours
1.

Explanation:

Patients without difficulty moving after stroke should be assessed as soon as possible within the first
24 hours of onset by an appropriately trained healthcare professional to determine the most
appropriate and safe methods of transfer and mobilisation.

Patients with difficulty moving early after stroke who are medically stable should be offered
frequent, short daily mobilisations (sitting out of bed, standing or walking) by appropriately trained
staff with access to appropriate equipment, typically beginning between 24 and 48 hours of stroke
onset.

Mobilisation within 24 hours of onset should only be for patients who require little or no assistance
to mobilise.

Reference:
2. Intercollegiate Stroke Working Party (2016). National clinical guideline for stroke. 5th edn. London:
Royal College of Physicians.

3. Between 24 and 48 hours Correct answer selected

Mrs B was seen in the acute receiving unit out-of-hours, with a transient right facial and arm
weakness lasting for 1 hour which occurred yesterday.

Her symptoms have completely resolved and she is eager to get home. She is on Edoxaban.

What is the preferred management for this patient?

1. Admit for neurological observation for 24 hours


2. Admit for neurological observation until CT brain during in-hours
3. Discharge without brain imaging, continue edoxaban and refer to urgent TIA clinic within 24 hours
4. Discharge without brain imaging, stop edoxaban and start aspirin 300 mg with referral to urgent TIA
clinic within 24 hours
5. Request CT brain out-of-hours
Explanation:

Patients with suspected TIA should be assessed by a specialist physician before a decision on brain
imaging is made, except when haemorrhage requires exclusion in patients taking an anticoagulant or
with a bleeding disorder when unenhanced CT should be performed urgently.

Patients with acute neurological symptoms that resolve completely within 24 hours (i.e. suspected
TIA) should be given aspirin 300 mg immediately and assessed urgently within 24 hours by a
specialist physician in a neurovascular clinic or an acute stroke unit.

In this case, Edoxaban is an anticoagulant and haemorrhage should be excluded.

Reference:

1. Intercollegiate Stroke Working Party (2016). National clinical guideline for stroke. 5th edn. London:
Royal College of Physicians.

2. Request CT brain out-of-hours Correct answer


A patient was admitted to the emergency department with an acute intracerebral haemorrhage
presented with 4 hours of onset time.

His blood pressure on admission is 200/90 mmHg and GCS on admission is 5.

What is the preferred management of his blood pressure?

1. Aim for a systolic blood pressure lower than 180 mmHg within 1 hour and maintain for 7 days
2. Aim for a systolic blood pressure within 130 to 140 mmHg within 1 hour and maintain for 48 hours
3. Aim for a systolic blood pressure within 130 to 140 mmHg within 1 hour and maintain for 7 days
4. Aim for a systolic blood pressure within 130 to 140 mmHg within 6 hours and maintain for 7 days
5. Do not offer rapid blood pressure monitoring
Explanation:

Do not offer rapid blood pressure lowering to people who:

1. have an underlying structural cause (for example, tumour, arteriovenous malformation or aneurysm)
1. have a score on the Glasgow Coma Scale of below 6
1. are going to have early neurosurgery to evacuate the haematoma
1. have a massive haematoma with a poor expected prognosis.
Offer rapid blood pressure lowering to people with acute intracerebral haemorrhage who do not have
any of the exclusions listed in the recommendation and who present within 6 hours of symptom onset
and have a systolic blood pressure between 150 and 220 mmHg.

Consider rapid blood pressure lowering for people with acute intracerebral haemorrhage who do not
have any of the exclusions listed in the recommendation and who present beyond 6 hours of
symptom onset or have systolic blood pressure greater than 220 mmHg.

Aim for a systolic blood pressure target of 130 to 140 mmHg within 1 hour of starting treatment and
maintain this blood pressure for at least 7 days.

Reference:

1. NICE guideline [NG128] (2019) Stroke and transient ischaemic attack over 16s: diagnosis and initial
management

2. Do not offer rapid blood pressure monitoring Correct answer

Mrs D is reviewed with a confirmed diagnosis of transient ischaemic attack 2 days ago with a 2-hour
history of left face and arm weakness. She went on to have carotid imaging which showed
symptomatic carotid stenosis in her right carotid artery (40% NASCET criteria). What should be the
next step in her management?

1. Do not consider surgery and give optimal medical treatment


2. Refer urgently for carotid endarterectomy and initiate optimal medical treatment after surgery
3. Refer urgently for carotid endarterectomy and receive optimal medical treatment
4. Refer urgently for carotid stenting and initiate optimal medical treatment after procedure
5. Refer urgently for carotid stenting and receive optimal medical treatment

Explanation:

Patients with TIA or an acute non-disabling stroke with stable neurological symptoms who have
symptomatic severe carotid stenosis of 50–99% (NASCET method) should be referred for carotid
endarterectomy to be performed as soon as possible within 7 days of the onset of symptoms in a
vascular surgical centre routinely participating in a national audit and receive optimal medical
treatment (control of blood pressure, antiplatelet treatment, cholesterol reduction through diet and
drugs, and lifestyle advice including smoking cessation).

Patients with TIA or an acute non-disabling stroke who have mild or moderate carotid stenosis of
less than 50% (NASCET method) should not undergo carotid intervention and receive optimal
medical treatment.

Patients who meet the criteria for carotid intervention but who are unsuitable for open surgery (e.g.
inaccessible carotid bifurcation, re-stenosis following endarterectomy, radiotherapy-associated
carotid stenosis) should be considered for carotid angioplasty and stenting.

People who have undergone carotid revascularisation should be reviewed post-operatively by a


stroke physician to optimise medical aspects of vascular secondary prevention.

Reference:

1. Intercollegiate Stroke Working Party (2016). National clinical guideline for stroke. 5th edn. London:
Royal College of Physicians.

2. Do not consider surgery and give optimal medical treatment Correct answer selected

A 58-year-old right-handed gentleman collapsed at his workplace at 8:30 a.m. while speaking to a
colleague. He is rushed to the emergency department where clinical examination reveals dense right
hemiparesis, global aphasia and right homonymous hemianopia.

NIHSS score is 20. CT scan of the brain is reported as normal.

CT angiogram of the neck and intracranial vessels shows the left middle cerebral artery is occluded
with a clot. His colleague confirms a witnessed onset time of 8:30 a.m.
No contraindication to thrombolytic therapy is identified after completing a safety checklist with the
patient’s family.

You are now 2 hours from the onset of symptoms.

What is the appropriate treatment choice for this patient?

1. Aspirin 300 mg stat and once daily


2. Intravenous thrombolysis with alteplase
3. Intravenous thrombolysis with alteplase and mechanical thrombectomy
4. Intravenous thrombolysis with tenecteplase
5. Mechanical thrombectomy alone

Explanation:

This patient has a clear onset time, measurable focal neurological deficit on the NIHSS scale and no
contraindications on the thrombolysis safety checklist. He is therefore a suitable candidate for
reperfusion therapy.

Mechanical thrombectomy in combination with intravenous thrombolysis is the treatment of choice


for patients with acute ischaemic stroke with a proximal intracranial arterial occlusion (Goyal et al.,
2016).
The National Clinical Guideline for Stroke recommends intravenous thrombolysis in combination
with mechanical thrombectomy as the treatment of choice for patients with an NIHSS score of 6 or
more and an identifiable proximal intracranial clot as long as the procedure can begin within 5 hours
from stroke onset (National Clinical Guideline for Stroke, 2016).

Reference:

1. Goyal, M., Menon, B., van Zwam, W. et al. (2016). Endovascular thrombectomy after large-vessel
ischaemic stroke: a meta-analysis of individual patient data from five randomised trials. The Lancet,
387(10029), pp. 1723-1731.
2. Royal College of Physicians (2016). National Clinical Guideline for Stroke, 5th edn, pp. 40 - 43.

3. Intravenous thrombolysis with alteplase and mechanical thrombectomy Correct answer

John is a 56-year-old gentleman who was noted to be slurring his speech at the local shop while
buying a newspaper at 8:45 a.m.

He was rushed to the local emergency department where clinical examination showed dense left
hemiparesis, severe inattention to the left, significant dysarthria and a partial gaze palsy to the left.

NIHSS is calculated at 14.

CT brain scan is normal and CT angiogram has shown a clot occluding the right middle cerebral
artery. His wife arrived and confirmed that he left home at 8:30 a.m. when he was absolutely normal.

She also stated that John is on a blood-thinning medication called apixaban for atrial fibrillation and
has taken his medication last night, but not this morning. No other contraindication to thrombolysis
is found on completing the safety checklist. It is now 10 a.m.

What is the appropriate treatment choice for this patient?

1. Aspirin 300 mg stat and once daily


2. Intravenous alteplase at a reduced dosage of 0.6 mg/kg body weight
3. Intravenous prothrombin complex concentrate followed by intravenous alteplase
4. Intravenous tenecteplase
5. Mechanical thrombectomy
1. Mechanical thrombectomy Correct answer selected

Explanation:

John has a clear witnessed time when he was last seen well and a measurable focal neurological
deficit on the NIHSS scale. He does have an absolute contraindication to thrombolytic therapy as he
is on the anticoagulant drug apixaban. Thrombolytic therapy in any form, alteplase or tenecteplase, is
contraindicated if an anticoagulant has been administered within the last 48 hours. Administering
prothrombin complex concentrate or other reversal agents to aid delivery of thrombolytic therapy is
not recommended (National Clinical Guideline for Stroke, 2016). The National Clinical Guideline
for Stroke recommends considering mechanical thrombectomy alone for patients who have a clear
contraindication to thrombolysis but not to thrombectomy, if they have a proximal intracranial large
vessel occlusion causing a disabling neurological deficit of NIHSS 6 or more, and the procedure can
begin within 5 hours of symptom onset (National Clinical Guideline for Stroke, 2016).

Reference:

2. Royal College of Physicians (2016). National Clinical Guideline for Stroke, 5th edn, pp. 40 -43.
[Online]
A 60-year-old lady with a background of hypertension and type 2 diabetes mellitus developed 3
episodes of right arm and leg weakness and numbness. Each episode lasted between five to ten
minutes and happened within a day. Each episode resolved spontaneously back to normal.

A clinical neurological examination confirmed no neurological deficit.

CT scan of the brain was reported as normal, carotid Dopplers showed atherosclerotic disease
without any significant stenosis and ECG showed normal sinus rhythm.

What driving advice would you give to this lady?

1. Continue to drive as normal


2. To not drive for one week
3. To not drive for one year
4. To not drive for three months
5. To not drive for twenty-eight days
1. To not drive for three months Correct answer
2. 48% To not drive for twenty-eight days Incorrect answer selected

Explanation:

The clinical situation described here is recurrent left-hemispheric TIA. As there are multiple events
within a time period of seven days, this will be classed as crescendo TIAs.

DVLA guidance for this is not to drive for 3 months and it is essential to inform the DVLA.

For a single TIA the DVLA guidance is not to drive for 1 month and there is no need to inform the
DVLA..

Reference:

3. DVLA (updated 2019). Guidance: Neurological disorders: assessing fitness to drive.


A 72-year-old man with a background of hypertension, type 2 diabetes mellitus, dyslipidaemia and
who is a current smoker developed an episode of the left arm and leg weakness and numbness.

The episode lasted between five to ten minutes and resolved spontaneously. A clinical neurological
examination confirmed no neurological deficit.

CT scan of the brain was reported as normal, carotid Dopplers showed atherosclerotic disease with
80% stenosis in the right internal carotid artery (NASCET criteria) and ECG showed normal sinus
rhythm.

What treatment strategy would you recommend for this gentleman?

1. Anticoagulation with warfarin or a direct oral anticoagulation agent


2. Carotid endarterectomy in right internal carotid artery with best medical treatment
3. Carotid stenting in right internal carotid artery with best medical treatment
4. Clopidogrel 300 mg stat and continue clopidogrel 75 mg once daily
5. Dual antiplatelet therapy with aspirin and clopidogrel 75 mg once daily each
1. Carotid endarterectomy in right internal carotid artery with best medical treatment Correct answer
selected

Explanation:

This gentleman has suffered a right hemispheric TIA for which carotid imaging is essential. There
are two valid ways to report carotid stenosis: NASCET (derived from North American Symptomatic
Carotid Endarterectomy Trial) and ECST (derived from European Carotid Surgery Trial).

The National Clinical Guideline for Stroke recommends carotid revascularisation for carotid stenosis
of 50-99% reported by NASCET criteria (National Clinical Guideline for Stroke, 2016).

Carotid angioplasty and stenting when compared with surgical endarterectomy is associated with an
increased risk of stroke of any severity or death (Bonati et al., 2012). However, this increased risk is
influenced by age and when the comparison is restricted to patients below the age of 70 years there is
no difference in adverse outcomes between the two procedures (International Carotid Stenting Study
investigators et al., 2010).

The National Clinical Guideline for Stroke recommends carotid endarterectomy for patients above
70 years of age, as in this case; however, for patients under 70 years of age both carotid
endarterectomy and carotid stenting should be offered (National Clinical Guideline for Stroke, 2016).

Reference:

2. Royal College of Physicians (2016). National Clinical Guideline for Stroke, 5th edn, pp. 37-38 & 91-
93.
3. Bonati, L. H., Lyrer, P., Ederle, J., et al. (2012). Percutaneous transluminal balloon angioplasty and
stenting for carotid artery stenosis. Cochrane Database of Systematic Reviews, 9, article number
CD000515.
4. International Carotid Stenting Study investigators (2010). Carotid artery stenting compared with
endarterectomy in patients with symptomatic carotid stenosis (International Carotid Stenting Study):
an interim analysis of a randomised controlled trial. The Lancet, 375(9719), 985-997.

A 72-year-old gentleman is admitted to the Acute Stroke Unit with left-sided weakness. His
background consists of hypertension, diabetes mellitus and a previous ischaemic stroke 1 year ago.

MRI brain shows an acute infarct and a gliotic area suggestive of an old infarct in the right frontal
lobe within the right anterior cerebral artery territory.

Carotid doppler shows no significant stenosis, ECG is in normal sinus rhythm.


CT angiogram of intracranial arteries showed significant stenosis of the right middle cerebral artery
and the right anterior cerebral artery, which was felt to be atherosclerotic in nature.

What treatment would you recommend for this gentleman?

1. Aspirin 300 mg once daily


2. Anticoagulation with Warfarin
3. Clopidogrel 75 mg once daily
4. Dual antiplatelet therapy - aspirin and clopidogrel
5. Right anterior cerebral artery angioplasty and stenting

Explanation:

This gentleman has recurrent stroke in the right anterior cerebral artery territory with symptomatic
atherosclerotic stenosis in this artery. The recurrent stroke rate in intracranial atherosclerotic arterial
stenosis is high and needs aggressive intervention. Medical management with initial 90 days of dual
antiplatelet therapy was found to be superior to intracranial arterial angioplasty and stenting in the
SAAMPRIS trial (Chimowitz et al., 2011). The National Clinical Guideline for Stroke recommends
best medical management with 90 days of dual antiplatelet therapy for intracranial stenosis of
atherosclerotic aetiology; angioplasty and stenting can only be offered in the context of a clinical
trial (National Clinical Guideline for Stroke, 2016).

Reference:

1. Chimowitz, M.I., Lynn, M.J., Derdeyn, C.P., et al. (2011). Stenting versus Aggressive Medical
Therapy for Intracranial Arterial Stenosis. New England Journal of Medicine, 365, pp. 993-1003.
2. Royal College of Physicians (2016). National Clinical Guideline for Stroke, 5th edn, p. 103.
3. Dual antiplatelet therapy - aspirin and clopidogrel Correct answer selected

Mr A was admitted to the accident and emergency department with a left total anterior circulation
stroke, NIHSS of 8 with a time onset of 8 hours.

He had a very good pre-functional status (modified Rankin scale 0). CT brain shows a hyperdense
vessel in the left middle cerebral artery with no haemorrhage.

There is a thrombectomy service with both CT perfusion and MRI available in the hospital.

Which investigation and management is best suited for Mr A for consideration of thrombectomy?

1. CT angiogram and CT perfusion scan


2. CT angiogram and MR diffusion-weighted imaging
3. CT angiogram only
4. CT brain is already sufficient
5. IV thrombolysis followed by CT angiogram
1.

Explanation:

Mr A’s presentation is outside the licensed time window of 4.5 hours.

The recommended selection of thrombectomy candidates is with a pre-stroke functional status of less
than 3 on the modified Rankin scale and a score of more than 5 on the National Institutes of Health
Stroke Scale (NIHSS).

Offer thrombectomy as soon as possible to people who were last known to be well between 6 hours
and 24 hours previously (including wake-up strokes) who have acute ischaemic stroke and confirmed
occlusion of the proximal anterior circulation demonstrated by CTA or MRA and if there is the
potential to salvage brain tissue, as shown by imaging such as CT perfusion or diffusion-weighted
MRI sequences showing limited infarct core volume.

In Mr A’s case, moving between a CT scanner and MRI scanner would cost valuable time.

The best investigation available is for CT angiogram and CT perfusion scan to consider
thrombectomy.

Reference:

2. NICE (2019). NICE guideline [NG128]: Stroke and transient ischaemic attack over 16s: diagnosis
and initial management.
3. CT angiogram and CT perfusion scan Correct answer selected

Mrs B was admitted out of hours with a 6-hour history of unexplained fluctuating symptoms of right
face, arm and leg weakness.

She is suspected to have an acute stroke. She is not on anticoagulants and has no known bleeding
tendencies.

Her current GCS is 13/15. When should a non-enhanced CT brain be performed?

1. 12 hours from symptom onset


2. 24 hours from symptom onset
3. 6 hours from symptom onset
4. Discuss the available options with radiology
5. Immediately
Explanation:

Perform brain imaging immediately with a non-enhanced CT for people with suspected acute stroke
if any of the following apply:

1. indications for thrombolysis or thrombectomy


2. on anticoagulant treatment
3. a known bleeding tendency
4. a depressed level of consciousness (Glasgow Coma Score below 13)
5. unexplained progressive or fluctuating symptoms
6. papilloedema, neck stiffness or fever
7. severe headache at onset of stroke symptoms.
Perform scanning as soon as possible and within 24 hours of symptom onset in everyone with
suspected acute stroke without indications for immediate brain imaging.

Reference:

8. NICE (2019). NICE guideline [NG128]: Stroke and transient ischaemic attack over 16s: diagnosis
and initial management.

9. Immediately Correct answer selected


A patient was found on the floor by a relative and was admitted with a left total anterior circulation
infarct.

He was dysphagic. He was last seen well 1 day ago and did not receive thrombolysis or
thrombectomy after initial radiological investigations.

He failed his initial swallow assessment and is unable to take adequate nutrition and fluids orally.

When should nasogastric tube feeding be initiated for this patient?

1. Keep NBM, reassess swallow the next day


2. Keep NBM, supplement with IV fluids and electrolytes until speech and language therapist
assessment
3. Keep NBM, supplement with IV fluids and electrolytes until speech and language therapist
assessment with dietician review
4. Within 24 hours of admission
5. Within 72 hours of admission
Explanation:

On admission, ensure that people with acute stroke have their swallowing screened by an
appropriately trained healthcare professional before being given any oral food, fluid or medication.

If the admission screen indicates problems with swallowing, ensure that the person has a specialist
assessment of swallowing, preferably within 24 hours of admission and not more than 72 hours
afterwards.

People with acute stroke who are unable to take adequate nutrition, fluids and medication orally
should:

1. receive tube feeding with a nasogastric tube within 24 hours of admission unless they have had
thrombolysis
2. be considered for a nasal bridle tube or gastrostomy if they are unable to tolerate a nasogastric tube
3. be referred to an appropriately trained healthcare professional for detailed nutritional assessment,
individualised advice and monitoring
4. have their oral medication reviewed to amend either the formulation or the route of administration.
Reference:

5. NICE (2019). NICE guideline [NG128]: Stroke and transient ischaemic attack over 16s: diagnosis
and initial management.

6. Within 24 hours of admission Correct answer selected

A 70-year-old man is admitted complaining of involuntary movements on the left side of the body.
These symptoms have deteriorated over the last 3 weeks. He has a 10-year history of type 2 diabetes
for which he takes ramipril 10mg/d, metformin 1g bd, sitagliptin 100mg /d and atorvastatin 20mg
daily.

On examination, his pulse is 80 bpm reg, blood pressure 136/88 mmHg and BMI 28 kg/m2. He has
writhing movements on the left limbs with some occasional left arm and leg flinging movements.

Neurological examination is otherwise normal with flexor plantars and normal reflexes.

Investigations:

Investigation Result Normal range


FBC Normal
U+Es Normal
Glucose 25.9 mmol/L 3.5-5.5 mmol/L
HbA1c 88 mmol/mol <48 (10.2%)
Blood ketones 0.5 mmol/L <0.6

CT Head Report Small vessel disease.


Mildly increased density in right basal
ganglia
Which of the following is the most appropriate approach to the management of this patient?

1. Commence tetrabenazine
2. Commence topiramate
3. Commence variable rate insulin infusion
4. Commence variable rate insulin infusion with IV diazepam
5. Stop sitagliptin; commence gliclazide

1.
Explanation:

This is a typical history of the rare but well-described non-ketotic hyperglycaemic (diabetic) hemi-
chorea hemi-ballism syndrome characterised by continuous, irregular, and involuntary jerky
movements of one side of the body, often the result of a focal lesion of the contralateral basal
ganglia.

It usually affects elderly women, and there is an Asian preponderance. The condition is associated
with poor glycaemic control, as in this case and, improved glycaemic control is associated with
improvement and resolution of symptoms.
There is no ketosis. The pathophysiology of the disorder is associated with relative ischaemia,
possibly a result of hyperviscosity within the striatum.

CT may be normal or reveal densities in the putamen/arcuate nucleus, so MRI is often required and
demonstrates T1 hyperintensity in the contralateral putamen.

Explanation:

2. Al Montasir, A. and Sadik, M.H., 2013. Hemichorea-hemiballism in a nonketotic diabetic patient.


Journal of family medicine and primary care, 2(3), p.296.

3. Commence variable rate insulin infusion Correct answer selected

A 72-year-old male with a history of poorly controlled diabetes is brought into the clinic by his wife
after she noted a change in his behaviour over an 8-month period.

According to the wife, the patient had been frequently misplacing items at home and frequently
irritated his other family members with repetitive questions. He also began to become lost when
driving alone in his car and could not be trusted with money anymore.

His appetite had also become smaller and he frequently cried to himself when left alone at home. He
slept most of the day and appeared less energetic than before.

He had lost around 5 kilograms of weight over this period.

What is the most likely explanation for his change in behaviour?

1. Fronto-temporal dementia
2. Generalised anxiety disorder
3. Mixed vascular and Alzheimer’s dementia
4. Possible underlying malignancy
5. Underlying major depressive disorder
Explanation:

A frequent mimic of dementia is depression in elderly patients.

Depression is frequently missed, and cognition tends to respond well with treatment of depression.
Though the patient appears to have problems with cognition, he has strong symptoms to suggest
depression which needs to be addressed first before assessing for dementia.

Reference:

1. Rodda, J., Walker, Z. and Carter, J. (2011). Depression in older adults. BMJ, 343, p. d5219.

2. Underlying major depressive disorder Correct answer selected


An 81-year-old woman attended the clinic claiming she could hear singing in her home.

She has not experienced any visual hallucinations or heard voices speaking. Neither the patient nor
her family expressed any concern about her memory or mood.

Other than a long-term hearing impairment that is treated with hearing aids, there were no other
neurological signs.

She has a history of atrial fibrillation and takes amlodipine, apixaban, atorvastatin, and metoprolol on
a daily basis.

She is fully self-sufficient and lives alone, and has demonstrated normal cognitive and psychological
functioning.

A complete neurological examination revealed bilateral hearing loss.

What is the most likely diagnosis?

1. Acoustic neuroma
2. Acute psychosis
3. Auditory Charles Bonnet syndrome
4. Multiple sclerosis
5. Schizophrenia
1.

Explanation:

Musical hallucinations are also known as Oliver Sacks syndrome or musical ear syndrome. These are
auditory hallucinations perceived occasionally or continuously by the patients, in the absence of an
external auditory stimulus.

The vast majority of patients have normal cognitive and mental functioning. As a result, they're
rarely linked to other sorts of hallucinations or worries about mood, memory, capability, or vision.
Psychiatric or neurologic causes of hallucinations should, however, be ruled out.

Treatment of the underlying cause, such as hearing aids, treatment of underlying psychological
illnesses, withdrawal of suspected triggering medicines, and treatment of epileptic seizures, typically
leads to the remission of symptoms.

Antipsychotics (olanzapine and quetiapine), as well as antidepressants, have been linked to musical
hallucinations.

Reference:

2. Kukstas, C. (2019). 'Auditory Charles Bonnet syndrome: case report'. BR J Gen Pract, July; 69
(684): 362-363.

3. Auditory Charles Bonnet syndrome Correct answer selected


A 76-year-old female presents to MAU with a 4-day history of deteriorating weakness in the legs and
paraesthesiae. Two weeks previously she had received the AZ COVID-19 vaccine.

In her past medical history, she had a 5-year history of hypertension for which she was treated with
losartan and bendrofluazide.

She also had a diagnosis of asthma and had been prescribed a steroid inhaler which she used once
daily.

On examination, she had a temperature of 36.7oC, a pulse of 88 bpm, a blood pressure of 128/82
mmHg and a respiratory rate of 18/min. She had weakness of the lower limbs 3-4/5, reduced tone,
absent plantar, ankle and knee reflexes but normal sensation to pinprick and light touch. There was
also weakness of 4/5 of the upper limbs and loss of biceps and triceps reflexes. Co-ordination
appeared intact and the cranial nerves were normal.

Investigations revealed the following:

Investigation Result Normal range


FBC Normal
Sodium 131 mmol/L 134 - 144 mmol/L
Potassium 3.2 mmol/L 3.5 - 5 mmol/L
Urea 8.8 mmol/L 3 - 8 mmol/L
CRP 6 mg/L <5
Glucose 6.1 mmol/L 3.5 - 5.5 mmol/L
CXR Normal
CT Head Normal
LP results reveal:

Investigation Result Normal range


Glucose Normal
Protein 0.9 g/L <0.4
Lymphocytes 1.1 0-5

Which of the following is the likely cause of this patient’s presentation?

1. COVID vaccination
2. Hypomagnesaemia
3. Hypophosphataemia
4. Treponema pallidum infection
5. Vitamin B12 deficiency
1.

Explanation:

The likely diagnosis with the dissociated ascending motor polyneuropathy with no features of
sensory loss is Guillain-Barre, likely due to the COVID-19 in the absence of any other obvious
cause. This has been rarely reported but the features here are typical of GBS as is the LP findings
with a dissociated elevation of protein.

Hypokalaemic periodic paralysis is unlikely as the history is very different with a much short
episodic weakness. The mild hypokalaemia with hyponatraemia is likely due to the bendrofluazide.

Reference:

2. McKean N, Chircop C. Guillain-Barré syndrome after COVID-19 vaccinationBMJ Case Reports CP


2021;14:e244125.

3. COVID vaccination Correct answer selected

A 69-year-old man with a longstanding history of bipolar disease and epilepsy complains of poor
physical health over the past 6 months.

He is experiencing nausea, diarrhoea, tiredness, and loss of appetite. He was recently admitted to the
hospital and was diagnosed with kidney stones.
His medications include topiramate 200 mg/day, atenolol 50 mg/day, gliclazide 60 mg/day,
empagliflozin 10 mg/day, and aspirin 150 mg/day.

His ABG analysis results are as follows:

Investigation Result Normal range


pH 7.15 7. 35 - 7.45
HCO3 15 mEq/L 23 - 27 mEq/L
PaO2 13 kPa 12 kPa
PCo2 4.9 kPa 4.7 - 5.9 kPa

What medication is the most likely cause of his symptoms?

1. Aspirin
2. Atenolol
3. Empagliflozin
4. Gliclazide
5. Topiramate

Explanation:

Topiramate, as Salek, et al. (2022) note, ‘is an antiepileptic drug which leads to mixed renal tubular
acidosis by the inhibition of carbonic anhydrase in renal tubules. This leads to systemic metabolic
acidosis with low plasma bicarbonate concentration and alkaline urine pH with low urine citrate
concentration. These metabolic changes result in calcium phosphate stone formation.’
Other medications that induce nephrolithiasis are antacids, protease inhibitors, vitamin D and
calcium supplements, laxatives, allopurinol, and antimicrobials like sulfonamides and quinolones.

The use of gliclazide, atenolol, aspirin, or acetylsalicylic acid does not lead to nephrolithiasis.

Patients taking empagliflozin have been noted to frequently experience vaginal candidiasis and
urinary tract infections, but not nephrolithiasis.

Reference:

1. NHS: How and when to take topiramate, 2022.


2. Salek, T., Andel, I. and Kurfurstova, I., 2022. Topiramate induced metabolic acidosis and kidney
stones – a case study, Biochemia medica, 27(2), 404–410.

3. Topiramate Correct answer selected

A 77-year-old woman is reviewed in clinic. She complains that she is receiving a large amount of
treatment and is keen to reduce the medication, if possible. Which of the following is considered an
implicit indicator of potentially inappropriate prescribing?

1.
2. Beta-blockers with ACE-inhibitors in hypertension
3. Co-prescription of the same class of drug
4. Digoxin use in heart failure with preserved systolic function
5. Dopamine agonist therapy for benign essential tremor
6. The use of a COX-2 inhibitor in the presence of cardiovascular disease Incorrect answer selected
Explanation:

The co-prescription of the same class of drugs in a patient is a good indicator of potentially
inappropriate prescribing. e.g. two classes of calcium channel in hypertension or omeprazole and
lansoprazole for instance. COX-2 inhibitors may be used with caution in the presence of
cardiovascular disease.

Inappropriate prescribing in the elderly is very common and STOPP-2 is a useful tool to guide
accurate prescribing. Eg prescribing of drugs where more harm than benefit exists, inappropriate
dose or duration; drug-drug or drug-disease interaction; under-prescribing and prescribing drugs of
similar action.

Reference:
7. Pérez, T., Moriarty, F., Wallace, E., McDowell, R., Redmond, P., & Fahey, T. (2018). Prevalence of
potentially inappropriate prescribing in older people in primary care and its association with hospital
admission: longitudinal study. bmj, 363, k4524.

8. Co-prescription of the same class of drug Correct answer

You intend to embark on a project to reduce hospital admissions due to preventable drug-related
admissions. You intend to hone your exercise to the commonest culprits.

Which four groups of medicines are linked to roughly 50% of preventable drug-related admissions?

1. Antiplatelets, anticoagulants, oral hypoglycaemics and NSAIDs


2. Antiplatelets, diuretics, NSAIDs and anticoagulants
3. NSAIDs, antiplatelets, insulin and anticoagulants
4. NSAIDs, opioids, anticoagulants and diuretics
5. Oral hypoglycaemics, anticoagulants, opioids and antiplatelets
Explanation:

Preventable drug-related admissions account for roughly 4% of hospital admissions. The four most
common culprits were antiplatelets (16%), Diuretics (16%), NSAIDS (11%) and anticoagulants
(8%).

Reference:

1. Howard, R. L., Avery, A. J., Slavenburg, S., Royal, S., Pipe, G., Lucassen, P., & Pirmohamed, M.
(2007). Which drugs cause preventable admissions to hospital? A systematic review. British journal
of clinical pharmacology, 63(2), 136-147.

2. Antiplatelets, diuretics, NSAIDs and anticoagulants Correct answer


You see a 75-year-old woman in clinic and find that she has a blood pressure of 162/82 mmHg. You
explain that you want to commence her on an antihypertensive and suggest amlodipine.

When you next see her one month later, you find that her blood pressure remains at 160/82 mmHg.
She explains that she often forgets to take her medication.

What is the average percentage of patients who are non-adherent to their new medicines?

1. 10%
2. 20%
3. 30%
4. 40%
5. 50%
Explanation:

On average, roughly 50% of patients are non-adherent to their medications after commencing a new
drug.

Reasons include lack of belief that there is anything wrong, lack of importance attached to the drug
or belief that the drug is producing a side effect. This is why it is so important to convey fully to the
patient the rationale for treatment and allow the patient to discuss and fully understand this.

Reference:

1. Jimmy, B., & Jose, J. (2011). Patient medication adherence: measures in daily practice. Oman
medical journal, 26(3), 155.

2. 50% Correct answer


A 75-year-old man of African-Caribbean origin was seen in your hypertension clinic. He has been on
amlodipine 10 mg daily, lisinopril 20 mg daily and simvastatin 40 mg daily for several years.

His blood pressure has been well controlled with measurements around the 130-145 systolic.

On this occasion, he mentions that one month prior to this appointment that he had developed
significant lip and tongue swelling and some tingling after having had a filling at the dentist.

The dentist was keen that he be seen in casualty but the patient refused and it all settled in about 2
hours or so. He has had no such episodes since.

Which of the following is the most appropriate approach regarding treatment?

1. Local anaesthetic was the most likely cause of the reported swelling
2. Reaction to the dentist’s latex gloves was the likely cause of this swelling
3. The patient's ethnicity has no bearing to the probability of this being an adverse drug reaction
4. This is most likely ACE-inhibitor induced angioedema but as it settled and he has been on treatment
for years without problem, the drug can be continued
5. This is most likely to be ACE-Inhibitor induced angioedema and the Lisinopril should be
discontinued
1.

Explanation:

This is a good description of angioedema and the most likely culprit is the ace-inhibitor, Lisinopril.
This is a serious potential side effect that should be reported via the Yellow Card Scheme and the
ACE-I should be discontinued with an alternative sought. 50% of ACE-inhibitor induced
angioedema occurs in the first few weeks of treatment but it can occur at any time, even after years
of treatment. It occurs in roughly 1%. It has been reported to occur even up to 3 months after
discontinuation of the drug. ACE-I induced angioedema is more common in individuals of afro-
Caribbean origin.

Reference:

2. Howarth, D. (2013). ACE inhibitor angioedema: a very late presentation. Australian family
physician, 42(12), 860.
3. Gibbs, C. R., Lip, G. Y. H., & Beevers, D. G. (1999). Angioedema due to ACE inhibitors: increased
risk in patients of African origin. British journal of clinical pharmacology, 48(6), 861.

4. This is most likely to be ACE-Inhibitor induced angioedema and the Lisinopril should be
discontinued Correct answer
A 79-year-old man was sent to the psychiatry outpatient clinic due to concerns about a potential
decline in his mental health over the last 7 months.

On examination, the patient is noted to have auditory hallucinations and his wife has noticed him
talking to people who are not there.

He also has a delusional belief that the devil has been tormenting him for many months now.

He remains anxious at home and is progressively isolating himself from everyone.

What is the most likely diagnosis?

1. Delusional disorder
2. Manic-depressive illness
3. Schizophrenia
4. Schizophreniform disorder
5. Schizotypal personality

Explanation:

This is a typical case of schizophrenia where the patient presents with psychotic symptoms,
delusions, and hallucinations. Delusional disorder is an illness characterised by at least 1 month of
delusions but no other psychotic symptoms.

In the case of typical manic-depressive disorder, the psychotic symptoms are consistent with either
mania or depression, and the patient experiences intervals of euthymia (normal mood) free from
psychotic symptoms.
Individuals with schizoid personality disorder find it difficult and are uninterested in developing
deep relationships with others and prefer to be alone. There are no additional signs of schizophrenia.

Schizotypal personality disorder is characterised by strange thoughts and behaviours in addition to


the pattern of uneasiness in close relationships with others. The oddness in this disorder is not as
extreme as that observed in schizophrenia.

In Schizophreniform disorders, symptoms persist for more than one month but less than 6 months.

Reference:

1. NHS: schizophrenia, 2019.


2. NICE Clinical Guideline CG178: Psychosis and schizophrenia in adults: prevention and
management, 2014.

3. Schizophrenia Correct answer selected

A 65-year-old man reported to the psychiatry clinic with a 15-year history of repetitive actions, such
as frequently washing his hands. These repetitive actions often make him anxious. His medical
history includes episodes of lightheadedness and fainting that occurred as soon as he stood from a
seated position. He also laments having a tough time at the start of micturition and he has a weak
urine stream. On further examination, the patient has blood pressure readings of 140/100 mmHg
while seated and 115/90 mmHg while standing.

Which of the following medications is the most appropriate initial therapy for this patient's
psychiatric disorder?

1. Clomipramine
2. Clonazepam
3. Phenelzine
4. Sertraline
5. Venlafaxine
Explanation:

This patient has obsessive-compulsive disorder, and Sertraline, an FDA-approved selective serotonin
reuptake inhibitor (SSRI), is being used to treat it.

Although temporary sleep issues, gastrointestinal issues, headaches, restlessness, and anxiety are
possible side effects of SSRIs, they are less severe than those brought on by other drugs, such as
clomipramine.

Although clomipramine was the first medication to be approved by the Food and Drug
Administration for the treatment of obsessive-compulsive disorder, its side effects, such as
orthostasis, gastrointestinal disturbance, and sedation, as well as its anticholinergic effects, which
include dry mouth and constipation, prevent it from being used as first-line therapy.

This patient has orthostatic hypotension and urinary symptoms likely due to benign hypertrophy of
the prostate. Hence, Clomipramine is not the first-line drug in this case.

The use of phenelzine, venlafaxine, and clonazepam for obsessive-compulsive disorder is not
recommended.

Reference:
1. NHS: obsessive-compulsive disorder, 2019.
2. Nice Clinical Guidelines CG31: Obsessive-compulsive disorder and body dysmorphic disorder:
treatment, 2005.

3. Sertraline Correct answer selected

An elderly lady comes to see you because of falls. She gives a very clear history that she has fallen
because of her long-standing arthritis in her left knee and confides in you that she has become
increasingly ‘jumpy’ over the last few years.

When asked to elaborate, she says that she has always been an anxious person, but since the death of
her best friend five years ago she has got increasingly anxious, being startled even at the creak of the
floorboards in the house. On the advice of a friend, she saw her GP and was referred to a
psychologist.

At the time it was helpful, and she saw the psychologist for approximately fifteen weeks attending
hour-long sessions, but as soon as the sessions stopped she could feel her anxieties creeping back
again.

She has checked out a self-help book from her local library, and although she recognises the
strategies it presents, she cannot seem to overcome her anxieties. She tells you that when she retired
from teaching ten years ago, she started to get anxious about her students and was prescribed
sertraline by her GP.

However, she felt that she did not get on with it at all and stopped it immediately when she started
feeling a little suicidal. She tells you that her anxiety has worsened recently and she gets worried
about even leaving the house.

She outright refuses to try another antidepressant after her previous experience.

She asks you if there is anything else she could try for her anxiety?

What would you recommend for this patient?


Amitriptyline 10mg to be taken at night

Diazepam 1mg to be taken as needed

Persuade her to try Sertraline 50mg daily a second time

Pregabalin 25mg twice daily

Quetiapine 12.5mg twice daily

Explanation:

This lady appears to have a generalised anxiety disorder.

The first-line treatments are psychological - self-help and psychological interventions. Second-line
treatments are medication. Normally, sertraline would be the recommended first line of treatment,
but this lady has had significant side effects from this.

An alternative SSRI or SNRI would be recommended, but she is unhappy to pursue this route.

NICE recommends pregabalin as third-line pharmacotherapy.

The other options would not be recommended. Benzodiazepines can be used in crisis management
but are fraught with hazards in the elderly, neither is the history one of an acute crisis.

Reference:

1. NICE: Generalised anxiety disorder and panic disorder in adults: management Clinical guideline
[CG113]Published date: 26 January 2011 Last updated: 26 July 2019

2. Pregabalin 25mg twice daily Correct answer selected


You have been asked to provide some teaching for a mixed group of health care professionals around
DNAR (‘Do Not Attempt Resuscitation’) decision making.

The group have been discussing the significance of the ‘Tracey judgement’ (relating to the 2014
Court of Appeal decision in the case of Tracey v Cambridge University Hospitals NHS), and how far
patients should be involved in DNAR decision making.

A participant has asked whether patients should always be directly involved in DNAR decisions, as a
result of this legal case.

What precedent did this legal case establish in relation to the legal framework in England and Wales
around DNAR decision making?

1. DNAR decisions must be discussed with patients, even if they request not to be involved
2. If the likelihood of success of DNAR is less than 5%, there is no requirement to discuss a DNAR
decision with the patient
3. Legally, it is the patient’s decision as to whether they should be resuscitated or not
4. That it is ‘good practice’ for patients to countersign their DNAR form
5. There must be a presumption of involvement of the patient in DNAR decisions unless it will cause
significant psychological harm
1.
2.
Explanation:

This was a significant and important legal case in relation to how and when DNAR orders should be
discussed with patients and others.

Whilst a DNAR decision remains ultimately a clinical one, there must be a presumption of
involvement of the patient in DNAR decisions unless it will cause significant ‘psychological harm’.

There is no specific requirement for patients to countersign DNAR forms.

Whilst there must be a presumption of involvement of patients in DNAR decisions, the General
Medical Council also suggests that the discussion should not be forced on a patient who has stated
that they do not wish to know about, or discuss, a DNAR decision.

References:

3. Etheridge Z and Gatland E. When and how to discuss “do not resuscitate” decisions with patients.
BMJ 2015;350:h2640.
4. General Medical Council (2010). Treatment and care towards the end of life: good practice in
decision making.

5. There must be a presumption of involvement of the patient in DNAR decisions unless it will cause
significant psychological harm Correct answer selected
Chronic disease and disability (Diagnosis and Management)

A 75-year-old presents with a history of weight loss, palpitations and heat intolerance of 8 weeks
duration. Examination reveals a resting tremor and atrial fibrillation with a ventricular rate of
110bpm.
He is haemodynamically stable and has a goitre on palpation of the neck.

An ultrasound of the neck confirms the presence of multiple nodules in the thyroid gland with the
largest of the nodules measuring 0.6 cm in size.

His TFTs show suppressed TSH of < 0.01 and free T4 which is mildly elevated.

What is the most appropriate modality which can be used for his definitive management?

1. Carbimazole
2. Observe
3. Propylthiouracil
4. Radioiodine ablation
5. Total thyroidectomy

Explanation:
The patient has underlying toxic multinodular goitre which should be initially rendered euthyroid by
using thioamide therapy (carbimazole or propylthiouracil).

These are usually effective in controlling the hyperthyroid state but not curative/definitive as the
question asks.

The treatment of choice in such patients remains radio-iodine ablation therapy. Surgical removal of
the gland can be offered but would be regarded as second-line to radio-iodine in this scenario.

Reference:

1. Wass, J A.H. Stewart, P, M. Amiel, S, A. Davies, M, J. Oxford Textbook of Endocrinology and


Diabetes, 2ed, 2011. DOI: 10.1093/med/9780199235292.001.1.

2. Radioiodine ablation Correct answer selected

A 74-year-old with a past medical history of life-threatening arrhythmias is on long term amiodarone
therapy and was fitted with a defibrillator 2 years ago. He has been clinically asymptomatic for the
last year as far as symptoms of palpitations are concerned. His recent routine blood tests show:

Investigation Result Normal range


TSH 15.5 mU/L 0.35-5.5 mU/L
Free T4 4.6 pmol/L 11.5-22.7 pmol/L
Anti TPO negative
What is the next therapeutic intervention you will consider?

1. Liothyronine 20 µg twice a day


2. Radioiodine ablation
3. Start levothyroxine 25 µg per day
4. Start levothyroxine 50 µg per day
5. Stop amiodarone

1.

Explanation:

This patient has amiodarone-induced hypothyroidism which is the commonest side effect associated
with amiodarone treatment in iodine replete areas (in contrast to amiodarone-induced thyrotoxicosis
more commonly seen in iodine depleted areas).

Discontinuation of amiodarone should be considered if possible.

In this case, because of a previous history of life-threatening arrhythmias, the safer option would be
to start with a low dose of levothyroxine.

Reference:
2. Newman, C, W. Price, A. Davies, D, W. Gray, T, A. Weetman, A, P. Amiodarone and the thyroid: a
practical guide to the management of thyroid dysfunction induced by amiodarone therapy. Doi:
http://dx.doi.org/10.1136/hrt.79.2.121

3. Start levothyroxine 25 µg per day Correct answer selected

A 71-year-old lady attends for routine review of her hypertension. She is fit and well with no other
medical history.

She takes perindopril 8 mg once daily for her hypertension, is a non-smoker and drinks about 10
units of alcohol per week.

Her mother is alive at the age of 91, is in a nursing home with Alzheimer’s disease, but has no
history of vascular disease.

Her father died at the age of 84 from carcinoma of the prostate.

He also had treated hypertension but had no history of vascular disease.

She does not know of any family history of hyperlipidaemia or premature cardiovascular disease.
Her blood pressure is well controlled.

She has superior corneal arcus but no other physical signs and has a BMI of 22 kg/m2.

As part of a cardiovascular risk assessment, she had a lipid profile requested 5 years ago and
repeated last month.

5 years ago:

Investigation Result Normal range


Total cholesterol 7.3 mmol/L
HDL cholesterol 1.9 mmol/L >1.0 mmol/L
Triglycerides 1.8 mmol/L <2.0 mmol/L
Calculated LDL 4.6 mmol/L
Fasting glucose 5.2 mmol/L 3.0-6.0 mmol/L
Thyroid profile normal
Last month:

Investigation Result Normal range


Total cholesterol 7.7 mmol/L
HDL cholesterol 1.8 mmol/L >1.0 mmol/L
Triglycerides 1.8 mmol/L <2.0 mmol/L
Calculated LDL 5.1 mmol/L
Fasting glucose 5.4 mmol/L 3.0-6.0 mmol/L
Thyroid profile normal
eGFR >90 >90
Urine protein: <30 mg/mmol <30 mg/mmol
creatinine
Whas is the most likely diagnosis for her lipid profile?

1. Drug-induced hyperlipidaemia
2. Familial combined hyperlipidaemia
3. Familial hypercholesterolaemia
4. Polygenic hypercholesterolaemia
5. Secondary hyperlipidaemia due to hypertension- induced nephropathy
Explanation:

Polygenic hypercholesterolaemia is a common finding, but unlike familial hypercholesterolaemia


(FH) the genetics of the condition are less distinct. The hypercholesterolaemia often deteriorates over
time and may be associated with cardiovascular disease, particularly in the context of other risk
factors. The impact of polygenic hypercholesterolaemia in terms of atherogenesis is likely to be less
significant than FH, bearing in mind that an individual with FH will have been exposed to high levels
of atherogenic lipoproteins from birth, rather than just later in life. The absence of a family history
makes FH and familial combined hyperlipidaemia less likely. ACE inhibitors do not cause a
dyslipidaemia and there is no evidence of nephropathy in this patient.

Reference:

1. Sharifi, M. Futema, M. Nair, D. Humphries, S, E. Polygenic Hypercholesterolemia and


Cardiovascular Disease Risk. Curr Cardiol Rep. 2019; 21(6): 43. Published online 2019 Apr 22. doi:
10.1007/s11886-019-1130-z.
2. General Practice Notebook Polygenic hypercholesterolaemia

3. Polygenic hypercholesterolaemia Correct answer selected


A 72-year-old man with a 12-year history of diabetes being treated with metformin is seen as he is
lethargic and the GP is concerned about his persistent hyperkalaemia.

His HbA1c is 58.5 mmol/mol (7.5%) (< 53 mmol/mol (7)).

He also has a history of hypertension for which he takes bisoprolol 5 mg daily, amlodipine 10 mg/d,
simvastatin 40 mg/d and aspirin 75 mg/d.

He has been re-referred to clinic by his GP as he has been noted to have a persistent hyperkalaemia.

Previously he was taking an ACE inhibitor but this was stopped one year ago.

Investigation Result Normal range


Sodium 132 mmol/L 134-144 mmol/L
Potassium 5.8 mmol/L 3.5 – 5 mmol/L
Urea 8.8 mmol/L 3-8 mmol/L
Creatinine 100 micromol/L 50-100 micromol/L
Glucose 14.1 mmol/L 3.5-5.5 mmol/L
Which of the following is the likely cause of this man’s hyperkalaemia?

1. Autoimmune hypoadrenalism
2. Bisoprolol
3. Chronic renal failure
4. Metformin
5. Renal artery stenosis
Explanation:

This patient has a long history of hypertension and hyperkalaemia. He has mild hyponatraemia
associated with hyperkalaemia and the most likely diagnosis is type 4 renal tubular acidosis or
hyporeninaemic hypoaldosteronism.

The most likely drug to cause this is the beta-blocker and this should be stopped. Other drugs
implicated include ACE inhibitors and NSAIDs.

Reference:

1. Hyporeninaemic hypoaldosteronism

2. Bisoprolol Correct answer selected

A 72-year-old man with a three-year history of diet-controlled type 2 diabetes mellitus and a
previous myocardial infarction (MI) 3 years ago attends for review.

He stopped smoking at the time of his MI. He has had difficulty tolerating a statin due to
troublesome muscle aches and pains.

His symptoms settle following discontinuation of therapy but return within a week of
recommencement. His serum creatine kinase has always been within the reference range.
You have tried to use the following drug therapies, using the minimum dose indicated in the BNF:
simvastatin, atorvastatin and rosuvastatin. He has tolerated pravastatin 40 mg once daily.

His current biochemistry:

Investigation Result Normal range


Total cholesterol 6 mmol/L
HDL cholesterol 1.0 mmol/L >1.0 mmol/L
Triglycerides 1.5 mmol/L 0.1-2.0 mmol/L
Calculated LDL 4.3 mmol/L
Creatinine 125 mmol/L 70-120 mmol/L
eGFR 54 mL/min/ 1.73 m2 >90

What would be the most appropriate next step to manage his lipid profile?

1. Add ezetimibe 10 mg once daily


2. Add fenofibrate 160 mg once daily
3. Add omega 3 fatty acids
4. Continue current therapy
5. Increase pravastatin to 80 mg once daily

Explanation:

This patient is at high risk of cardiovascular events due to diabetes mellitus, previous MI and
smoking history.

In this patient we are dealing with secondary prevention and the aim should be to reduce his LDL
cholesterol, ideally to <2 mmol/L.

The maximum recommended dose of pravastatin is 40 mg once daily. But even at maximum dose
pravastatin is a low-intensity statin, i.e. reduces LDL cholesterol by 29% from baseline.

Ezetimibe can effectively reduce LDL cholesterol by blocking intestinal absorption of dietary and
biliary cholesterol, particularly in conjunction with a statin.
Both omega 3 fatty acids and fenofibrate have a limited effect in reducing LDL cholesterol.
Fenofibrate should be used with caution in renal dysfunction, particularly in conjunction with a
statin.

NICE CG181 does not recommend fibrates or omega 3 fatty acids for the prevention of
cardiovascular disease.

Reference:

1. NICE Guidance: TA385


2. NICE Guidance: NG28

3. Add ezetimibe 10 mg once daily Correct answer selected

A 75-year-old man is reviewed 3 months following admission with a myocardial infarction. He


required coronary artery bypass surgery but has made a good recovery and is asymptomatic.

He is a lifelong non-smoker, has had treated hypertension for 10 years and plans to return back to
work on a part-time basis.

His lipid profile prior to his MI reveals:

Investigation Result Normal range


Total cholesterol 5.7 mmol/L
HDL cholesterol 1.0 mmol/L >1.0 mmol/L
Triglycerides 0.8 mmol/L 0.1-2.0 mmol/L
Calculated LDL 4.3 mmol/L
Fasting glucose 4.8 mmol/L 3.5-5.5 mmol/L
He has never taken a statin because of concerns about side effects and has not started the atorvastatin
recommended by his cardiac surgeon.

Which of the following actions is appropriate?

1. Advise that dietary measures alone are appropriate and refer to a dietitian
2. Advise that statin therapy is safe in nearly all people and is recommended for all patients following
CABG
3. Recommend a bile acid sequestrant
4. Recommend starting a fibrate
5. Recommend starting ezetimibe

1.
Explanation:

Lipid lowering with a statin is recommended for all patients who have established cardiovascular
disease (secondary prevention).

In an individual who has never experienced side effects, he should be reassured that statins are safe
and effective at reducing the risk of recurrent cardiovascular events. Risk factor calculators are not
appropriate for use in secondary prevention.

Dietary advice is an important adjunct to drug therapy in this situation, but not as the only
intervention. Drug therapies, other than statins should not be offered at this stage as monotherapy.

Reference:

2. NICE: Lipid-modifying drugs


3. NICE CG 181

Advise that statin therapy is safe in nearly all people and is recommended for all patients
A 72-year-old male is referred by his GP who was concerned about an elevated prolactin
concentration.

He had been well up until 3 years ago when he was diagnosed with prostate cancer and was treated
with GnRH analogues.

Since then he has been tired but his prostate cancer is well controlled, being on 6 monthly
appointments with urology.

His GP had checked a few results and found that his prolactin was 980 mu/l (50-450), precipitating
referral.

He currently receives amlodipine, ramipril, simvastatin and domperidone

He appears hypogonadal in appearance and his blood pressure is 134/74 mmHg with a BMI of 32
kg/m2.

Investigations reveal:

Investigation Result Normal range


Testosterone 1.2 nmol/L 8-30 nmol/L
Prolactin 800 mu/L 50-450 mu/L
LH 1.2 mu/L 1-10 mu/L
FSH 1.1 mu/L 1-10 mu/L
Free T4 9.1 nmol/L 9-23 nmol/L
TSH 4 mu/L 0.5-5 mu/L
IGF-1 18 nmol/L 15-30 nmol/L
Which of the following is the likely cause for the hyperprolactinemia?

1. Drug-induced
2. Metastatic prostatic disease
3. Microprolactinoma
4. Non-functioning pituitary tumour
5. Primary hypothyroidism
Explanation:

This patient is receiving a dopamine antagonist - domperidone. This is the most likely cause for his
hyperprolactinemia. The thyroid function tests are entirely normal and a microprolactinoma would
be very unusual in a man of this age. The suppression of LH, FSH and testosterone indicates
adequate treatment with GnRH analogues.

Reference:

1. Thapa, S, Bhusal, K. Hyperprolactinemia. StatPearls, 2021.

2. Drug-induced Correct answer selected


A 70-year-old man is being seen regarding erectile dysfunction.

He has previously been seen and diagnosed with late-onset hypogonadism associated with obesity.

He has received three months of treatment with a testosterone gel formulation and currently uses 60
mg daily. His most recent result reveals:

Investigation Result Normal range


Testosterone 18.1 nmol/L 9-30 nmol/L

He has noticed an improvement in his energy levels and vitality as well as improved libido, but
despite these improvements, his erectile dysfunction persists.

Which of the following would be the most appropriate approach to his further management?

1. Add intracavernosal alprostidil (Caverject)


2. Add intraurethral alprostidil (Muse)
3. Add PDE-5 inhibitor
4. Titrate the dose of testosterone to 70mg daily
5. Switch testosterone to intramuscular testosterone

Explanation:

This man has been treated for weight-related hypogonadism and is receiving an adequate
replacement dose of testosterone, as reflected by the normal testosterone concentration.
Despite this, his erectile dysfunction persists and the most appropriate first-line approach would be to
try a PDE-5 inhibitor such as sildenafil, tadalafil or vardenafil. Success rates of up to 80% with
appropriate use of PDE-5 inhibitors may be expected (1).

Reference:

1. NICE: erectile dysfunction

2. Add PDE-5 inhibitor Correct answer selected

//////////////////////////////////////

You might also like